Sunteți pe pagina 1din 115

1

Department of English

Verbal and Quantitative


Reasoning
(Course Material)

Course Code: 15 EN 3105


Semester-V
2

SESSION -1
INTRODUCTION
The mention of Verbal Ability typically evokes two polemical responses.
“There’s nothing difficult about Verbal questions. RCs or paragraphs are given. You read them and answer
the questions given.”
“You just can’t figure out what is correct in Verbal. The most bizarre option tends to be correct.”
And the answer to this is not a middle ground between these two responses. Verbal Aptitude can be simple,
if you get your perspective about Verbal right. Below are various Verbal Ability question-types and the skills
that one needs to tackle these questions.
Vocabulary:
Vocabulary-based questions can range from Sentence Completion and Cloze Tests to Synonyms, Antonyms,
Analogies, Spellings, Words Often Confused and Odd Words. These questions require a combination of
vocabulary knowledge, reading skills and reasoning powers. Further, vocabulary knowledge is not merely
knowing what a word means but understanding what contexts the word can fit into.
For example, a Sentence Completion question can present the four answer options as: mitigate, alleviate,
ameliorate, assuage. At first glance, all four words may be defined generally as, ‘to lessen the intensity or
severity of something unpleasant’, but specifically mitigate, alleviate and assuage are used to refer to
lessening pain, grief, suffering, anger. Assuage is particularly used with hunger and anger. But ameliorate is
‘to make something more bearable’. Difficult conditions or states are ameliorated.
Take another example, the difference between the words incredible and incredulous. An unbelieving person
is incredulous. An unbelievable thing is incredible.
How to improve Vocabulary?
Vocabulary can be approached through etymology - understanding root meanings. Knowing the meaning of
a Latin root like ‘loqui’ will enable one to understand words like eloquent, loquacious, soliloquy,
ventriloquize, circumlocution, grandiloquent, magniloquent etc. Add to your understanding of roots, the
meanings of prefixes and suffixes and you have considerable control over vocabulary.
However, at the next level, you must seek a thorough mastery of the connotations and collocations of words
by encountering words in their appropriate contexts. Contextual word learning also allows you to retain the
words through the associations that the context creates for you.
Grammar:
There are two approaches to solving grammar based question – one is the classical approach wherein you
go by rules and learn the various parts of speech and their usages and the rules that apply, or you practice
the kind of questions that are asked and you take an approach of learning the kind of errors which lead to
questions based on error detection or sentence correction. Once there is familiarity with the error types, a
lot of practice helps you cross the line. Understanding Grammar does have a considerable impact on solving
3

other Verbal questions like Para-jumbles, Logical Continuation, Summary and even RC questions because
some of the option eliminations can be done through grammar rules as well.
Reading Comprehension:
Reading skills can make or mar the verbal score!
Reading Skills can be developed in three phases:
1. Habit cultivation: First, cultivate the reading habit. Read, read and read. Start with simple fiction,
newspapers and blogs. The components of reading include sustained concentration and retention of
information while reading.
2. Speed Building: Speed of reading being a critical component of any aptitude exam, you must cultivate speed.
Target nothing less than a reading speed of about 300 words per minute depending on the level of the text.
3. Diverse Reading: Read more but read diverse. Choose to read those topics that are out of your comfort
zone. RCs in aptitude exams can be drawn from any field.
Logic Based Questions (Logical Continuation, Logical Discontinuation, Critical Reasoning, Para jumbles
etc.) :
1. A piece of writing is as logical as any piece of logic can be!
2. A big learning for a student of Verbal Aptitude is the realization that writing follows logical patterns. This is
why a reader must learn to read critically. Understanding the technicalities of Critical Reasoning is
fundamental. Critical Reasoning not only prepares you to address the whole gamut of CR questions-
Assumptions, Strengthen, Weaken, Method of Reasoning, Flaw in the Reasoning, Syllogisms but also
enables you to understand structuring of ideas thus, facilitating critical reading. With Critical Reasoning,
you are ready to take on questions like Logical Continuation, Logical Discontinuity, Para-jumble etc.
3. A practice of a huge number of questions of all question types is the suggested preparation plan. Ensure that
you get an exposure of diverse topics.
4. Overall, developing Verbal Aptitude has multi-layered benefits. The structuring of thought that results from
good Verbal Aptitude is fundamental to being a successful manager.
4

COMPETENCY 1
LEXICAL RELATIONS
Verbal Ability -Session-2
Common Antonyms- List-1
1. Adulation X Criticism 49. Cessation X Gravity
2. Advocate X Oppose 50. Chaffing X Capitalistic
3. Affable X Rude 51. Dank X Dry
4. Affected X Unfeigned 52. Dapper X Untidy
5. Affluence X Poverty 53. Dauntless X Cowardly
6. Agility X Awkwardness 54. Dearth X Bundance
7. Alacrity X Slowness 55. Debacle X Progress
8. Alleviate X Worsen 56. Debilitate X Strengthen
9. Allure X Repel 57. Debonair X Awkward
10. Aloof X Gregarious 58. Derogatory X Praising
11. Amalgamate X Separate 59. Desecrate X Consecrate
12. Ambiguous X Clear 60. Destitute X Affluent
13. Amble X Hasten. 61. Devoid X Full Of
14. Ambulatory X Bedridden 62. Devout X Impious
15. Ameliorate X Make Worse 63. Diabolical X Seraphic
16. Analogous X Not Comparable 64. Diatribe X Eulogy
17. Anathematize X Bless 65. Diffidence X Boldness
18. Anomaly X Regularity 66. Dilate X Contrast
19. Antipathy X Fondness 67. Dilatory X Prompt
20. Antithesis X Similarity 68. Diminution X Appreciation
21. Baroque X Simple 69. Din X Silence
22. Beatific X Dreadful 70. Disabuse X Deceive
23. Belittle X Extol 71. Disconsolate X Joyous
24. Bellicose X Peaceful 72. Enervate X Strengthen
25. Benign X Malignant 73. Enhance X Degrade
26. Benison X Curse 74. Ennui X Excitement.
27. Berate X Praise 75. Enunciate X Mumble
28. Bestial X Noble 76. Ephemeral X Eternal
29. Bigotry X Tolerance 77. Equable X Stormy
30. Bizarre X Normal 78. Equanimity X Agitation
31. Blanch X Darken 79. Equilibrium X Imbalance
32. Blend X Soft 80. Equitable X Unfair
33. Blesse X Ardent 81. Equivocal X Clear
34. Blithe X Cheerless 82. Erratic X Steady
35. Bleak X Cheerful 83. Erroneous X Accurate
36. Capacious X Not Spacious 84. Erudite X Ignorant
37. Capricious X Steadfast 85. Ethereal X Earthy
38. Captious X Tolerant 86. Eulogistic X Critical
39. Carnal X Spiritual 87. Grandiose X Simple
40. Carnivorous X Herbivorous 88. Gratuitous X Warranted
41. Carping X Uncritical 89. Gregarious X Antisocial
42. Castigation X Commendation 90. Grisly X Antisocial
43. Categorical X Qualified 91. Gullible X Incredulous
44. Catholic X Narrow 92. Gusto X Distaste
45. Celerity X Delay 93. Gusty X Calm
46. Celibate X Married 94. Hackneyed X Original
47. Censure X Praise 95. Haggard X Plump
48. Centrifugal X Centripetal 96. Halcyon X Martial
5

97. Haphazard X Deliberate 151. Omnipotent X Weak


98. Ignoble X Worthy 152. Omniscient X Ignorant
99. Illusive X Not Deceptive 153. Opiate X Stimulant
100. Irksome X Interesting 154. Opportune X Awkward
101. Irrelevant X Pertinent 155. Opportunist X Man of Principle
102. Irreparable X Correctable 156. Opprobrium X Praise
103. Irreverent X Pious 157. Optimum X Worst
104. Jaded X Stimulated 158. Opulence X Poverty
105. Jaundiced X Unbiased 159. Ostentatious X Unassuming
106. Jaunty X Sedate 160. Precipitate X Stationary.
107. Jeopardy X Safety 161. Permeable X Impenetrable
108. Jettison X Salvage 162. Pernicious X Harmless
109. Jocular X Serious 163. Perpetual X Momentary
110. Judicious Unwise 164. Perspicuity X Vagueness
111. Kindle X Extinguish 165. Pertinacious X Superficial
112. Keen X Dull 166. Petulant X Uncomplaining
113. Knowledge X Ignorance 167. Precipitate X Cautious
114. Kill X Animate 168. Prefatory X Conclusive
115. Lachrymose X Cheering 169. Quell X Incite
116. Lackadaisical X Ambitious 170. Quintessence X Impure
117. Laconic X Verbose 171. queue X Curve, Zigzag
118. Lampoon X Praise 172. Ratify X Denounce
119. Languor X Vitality 173. Resilient X Unyielding
120. Latent X Obvious 174. Restive X Placid
121. Lavish X Frugal 175. Retentive X Forgetful
122. Laudatory X Defamatory 176. Reticence X Loquaciousness
123. Lax X Strict 177. Retrograde X Progressing
124. Lechery X Purity 178. Reverie X Dishonor
125. Lethargic X Invigorating 179. Rife X Scant
126. Limpid X Turbid 180. Robust X Weak
127. Lithe X Stiff 181. Rotundity X slimness
128. Loath X Eager 182. Rubble X Unbroken stone
129. Loquacious X Taciturn 183. Ruddy X War
130. Lugubrious X Cheerful 184. Rudimentary X Developed
131. Lurid X Dull 185. Rueful X Content
132. Malign x Eulogize 186. Rustic X Urban
133. Malleable X Brittle 187. Ruthless X Merciful
134. Manacle X Sane 188. Seldom x Frequently
135. Manifest X Obscure 189. Wreck X Restore
136. Manumit X Enslave 190. Wonder X Expectations
137. Martial X Bellicose 191. Wane X Prosper
138. Nebulous X Clear 192. Weary x Energetic
139. Nefarious X Bergin 193. Well-round x Scattered
140. Negation X Affirmation 194. Wrath X Delight
141. Neophyte X Veteran 195. Within X Beyond
142. Niggardly X Prodigal 196. Worried x Cheerful
143. Nocturnal X Daily 197. Watch x Neglect
144. Obdurate X Fleeting 198. Zany X Sane
145. Obese X Cadaverous 199. Zeal X Apathy
146. Objective X Emotionally Involved 200. Zaftig x Petite
147. Obligatory X Optional
148. Obloquy X Praise
149. Obsequious X Supercilious
150. Odium X Liking
6

SESSION -3
List-2
1. Ancient – Modern 40. Passive – Active
2. Arrive – Depart 41. Permanent – Unstable
3. Arrogant – Humble 42. Plentiful – Sparse
4. Ascend – Descend 43. Positive – Negative
5. Attack – Defend 44. Powerful – Weak
6. Blunt – Sharp 45. Praise – Criticism
7. Brave – Cowardly 46. Private – Public
8. Cautious – Careless 47. Problem – Solution
9. Complex – Simple 48. Professional – Amateur
10. Compliment – Insult 49. Profit – Loss
11. Crazy – Sane 50. Quality – Inferiority
12. Crooked – Straight 51. Random – Specific
13. Decrease – Increase 52. Rigid – Flexible
14. Demand – Supply 53. Segregate – Integrate
15. Destroy – Create 54. Shame – Honor
16. Divide – Unite 55. Simple - Complicated
17. Drunk – Sober 56. Single – Married
18. Expand – Contract 57. Stiff – Flexible
19. Freeze - Boil 58. Strength – Weakness
20. Full – Empty 59. Sturdy – Weak
21. Generous – Stingy 60. Sunny - Cloudy
22. Giant – Dwarf 61. Superb – Inferior
23. Gloomy – Cheerful 62. Temporary – Permanent
24. Guilty – Innocent 63. Timid – Bold
25. Hire – Fire 64. Toward – Away
26. Include – Exclude 65. Tragic – Comic
27. Individual – Group 66. Transparent - Opaque
28. Innocent – Guilty 67. Triumph – Defeat
29. Knowledge – Ignorance 68. Union – Separation
30. Liquid – Solid 69. Unique – Common
31. Lonely – Crowded 70. Upset – Relaxed
32. Major – Minor 71. Urge – Deter
33. Marvelous – Terrible 72. Vacant – Occupied
34. Mature – Immature 73. Vague – Definite
35. Maximum - Minimum 74. Vertical – Horizontal
36. Noisy – Quiet 75. Villain – Hero
37. Optimist - Pessimist 76. Visible - Invisible
38. Ordinary – Extraordinary 77. Wax – Wane
39. Partial – Complete 78. Wealth – Poverty
7

ANTONYMS
Directions to Solve: In the following questions choose the word which is the exact OPPOSITE of the given words.
1. ENORMOUS ( )
A. Soft B. Average C. Tiny D. Weak
2. COMMISSIONED ( )
A. Started B. Closed C. Finished D. Terminated
3. ARTIFICIAL ( )
A. Red B. Natural C. Truthful D. Solid
4. EXODUS ( )
A. Influx B. Home-coming C. Return D. Restoration
5. RELINQUISH ( )
A. Abdicate B. Renounce C. Possess D. Deny
6. EXPAND ( )
A. Convert B. Condense C. Congest D. Conclude
7. MORTAL ( )
A. Divine B. Immortal C. Spiritual D. Eternal
8. QUIESCENT ( )
A. ACTIVE B. Dormant C. Weak D. Unconcerned
9. OBEYING ( )
A. Ordering B. Following C. Refusing D. Contradicting
10. FRAUDULENT ( )
A. Candid B. Direct C. Forthright D. Genuine
11. FLAGITIOUS ( )
A. Innocent B. Vapid C. Ignorant D. Frivolous
12. BELITTLE ( )
A. Criticize B. Flatter C. Exaggerate D. Adore
13. STARTLED ( )
A. Amused B. Relaxed C. Endless D. Astonished
14. BUSY ( )
A. Occupied B. Engrossed C. Relaxed D. Engaged
15. FRESH ( )
A. Faulty B. Sluggish C. Disgraceful D. Stale
16. CULPABLE ( )
A. Defendable B. Blameless C. Careless D. Irresponsible
8

17. EVASIVE ( )
A. Free B. Honest C. Liberal D. Frank
18. GREGARIOUS ( )
A. Antisocial B. Glorious C. Horrendous D. Similar
19. AWARE ( )
A. Uncertain B. Ignorant C. Sure D. Doubtful
20. HIRSUTE ( )
A. Scaly B. Bald C. Erudite D. Quiet
21. SHRINK ( )
A. Contract B. Spoil C. Expand D. Stretch
22. COMMON ( )
A. Rare B. Small C. Petty D. Poor
23. COMFORT ( )
A. Uncomfort B. Miscomfort C. Discomfort D. None Of These
24. DEAR ( )
A. Priceless B. Free C. Worthless D. Cheap
25. ARROGANT ( )
A. Humble B. Cowardly C. Egotistic D. Gentlemanly
26. VICTORIOUS ( )
A. Defeated B. Annexed C. Destroyed D. Vanquished
27. GRACEFUL ( )
A. Rough B. Expert C. Miserable D. Awkward
28. NADIR ( )
A. Modernity B. Zenith C. Liberty D. Progress
29. EXTRAVAGANCE ( )
A. Luxury B. Poverty C. Economical D. Cheapness
30. PERTINENT ( )
A. Irrational B. Irregular C. Insistent D. Irrelevant
31. OBSCURE ( )
A. Implicit B. Obnoxious C. Explicit D. Pedantic
32. URBANE ( )
A. Illiterate B. Backward C. Discourteous D. Orthodo
33. VANITY ( )
A. Pride B. Humility C. Conceit D. Ostentatious
9

34. RARELY ( )
A. Hardly B. Definitely C. Frequently D. Periodically
35. MALICIOUS ( )
A. Kind B. Boastful C. Generous D.Indifferent
36. EPILOGUE ( )
A. Dialogue B. Prelude C. Post script D.Epigram
37. CAPACIOUS ( )
A. Limited B. Caring C. Foolish D.Changeable
38. CONDENSE ( )
A. Expand B. Distribute C. Interpret D. Lengthen
39. ADAPTABLE ( )
A. Adoptable B. Flexible C. Yielding D. Rigid
40. SACROSANCT ( )
A. Irreligious B. Unethical C. Irreverent D. Unholy
41. INDISCREET ( )
A. Reliable B. Honest C. Prudent D. Stupid
42. FAMILIAR ( )
A. Unpleasant B. Dangerous C. Friendly D. Strange
43. TANGIBLE ( )
A. Ethereal B. Concrete C. Actual D. Solid
44. LOVE ( )
A. Villainy B. Hatred C. Compulsion D. Force
45. FAMOUS ( )
A. Disgraced B. Unknown C. Evil D. Popular
46. ABSOLUTE ( )
A. Deficient B. Faulty C. Limited D. Scarce
47. FRUGAL ( )
A. Copious B. Extravagant C. Generous D. Ostentatious
48. INSIPID ( )
A. Tasty B. Stupid C. Discreet D. Feast
49. ABLE ( )
A. Disable B. Inable C. Unable D. Misable
50. HOSTILITY ( )
A. Courtesy B. Hospitality C. Relationship D. Friendliness
10

51. CROWDED ( )
A. Busy B. Congested C. Quiet D. Deserted
52. COMIC ( )
A. Emotional B. Tragic C. Fearful D. Painful
53. HAPLESS ( )
A. Cheerful B. Consistent C. Fortunate D. Shapely
54. FLIMSY ( )
A. Frail B. Filthy C. Firm D. Flippant
55. EQUANIMITY ( )
A. Resentment B. Dubiousness C. Duplicity D. Excitement
56. ADDITION ( )
A. Division B. Enumeration C. Subtraction D. Multiplication
57. ZENITH ( )
A. Acme B. Top C. Nadir D. Pinnacle
58. DOUBTFUL ( )
A. Famous B. Certain C. Fixed D. Important
59. PERENNIAL ( )
A. Frequent B. Regular C. Lasting D. Rare
60. BENIGN ( )
A. Malevolent B. Soft C. Friendly D. Unwise
61. HINDRANCE ( )
A. Aid B. Persuasion C. Cooperation D. Agreement
62. EXTRICATE ( )
A. Manifest B. Palpable C. Release D. Entangle
63. REPRESS ( )
A. Inhibit B. Liberate C. Curb D. Quell
64. ACQUITTED ( )
A. Freed B. Burdened C. Convicted D. Entrusted
65. PROVOCATION ( )
A. Vocation B. Pacification C. Peace D. Destruction
66. SUBSERVIENT ( )
A. Aggressive B. Straightforward C. Dignified D. Supercilious
67. LEND ( )
A. Borrow B. Cheat C. Pawn D. Hire
11

68. FAINT-HEARTED ( )
A. Warm-hearted B. Full-blooded C. Hot-blooded D.Stout-hearted
69. REMISS ( )
A. Forgetful B.Watchful C. Dutiful D. Harmful
70. TRANSPARENT ( )
A. Semi-transparent B.Muddy C. Opaque D. Dark
71. HONORARY ( )
A. Dishonorable B. Reputed C. Paid D. Official
72. METICULOUS ( )
A. Mutual B. Shaggy C. Meretricious D. Slovenly
73. LOQUACIOUS ( )
A. Reticent B.Talkative C. Garrulous D. Verbose
74. CONFESS ( )
A. Deny B.Refuse C. Contest D. Contend
75. ANNOY ( )
A. Praise B.Rejoice C. Please D. Reward
SESSION-3
SYNONYMS

Annihilation, destruction, carnage, extinction


Benefit, profit, revenue, yield
Cunning, keen, sharp, slick
Destitute, poor, bankrupt, impoverished
Deterioration, pollution, defilement, adulteration
Enormous, huge, gigantic, massive
Fertile, fruitful, abundant, productive
House, dwelling, abode, domicile
Intelligent, clever, brilliant, knowledgeable
Loyal, faithful, ardent, devoted
Organization, institution, management
Partner, associate, colleague, companion
Polite, courteous, cordial, gracious
Risky, dangerous, perilous, treacherous
Sleepy, drowsy, listless, sluggish
Vacant, empty, deserted, uninhabited
12

Synonyms Exercises

Directions: Choose and mention in the space provided the option that is nearest in meaning to the head word.

1. Jaunt ( )
a) irresistible force b) solitary c) short drive d) refuse
2. Fostering ( )
a) Safeguaring b) Ignoring c) Neglecting d) Nurturing
3. Indelible ( )
a) Track b) Enforce c) Revolt d) Permanent
4. Hematology ( )
a) study of humans b) study of birds c) study of nervous system d) study of blood
5. Kindred ( )
a) liberal b) related c) cliff d) enemy
6. Hebetude ( )
a) foolishness b) intelligence c) sharpness d) numbness
7. Impediment ( )
a) obstacle b) supplement c) to insult d) Peace
8. Imminent ( )
a) Impure b) Unsteady c) upcoming d) Proud
9. Kindle ( )
a) desire b) creditable c) old person d) to ignite
10. Kernel ( )
a) expose b) hungry c) core d) impose
11. Frugal ( )
a) invention b) economical c) explore d) to whisper
12. Grisly ( )
a) ornate b) earthly c) remote d) horrible
13. Jargon ( )
a) Fate b) Simple c) Decisive d) Complex
14. Milieus ( )
a) apprehensions b) surroundings c) arrangements d) circular
15. Gyrate ( )
a) spin b) explain c) decayed d) float
16. Hallowed ( )
a) old b) sacred c) inactive d) effective
13

17. Gelid ( )
a) talkative b) hard headed c) soft d) extremely cold
18. Fecund ( )
a) fertile b) entangle c) immature d) hostile
19. Jeer ( )
a) taunt b) mild c) friend d) sluggish
20. Gall ( )
a) irritate b) blessed c) sullen d) necessary
21. Ravage ( )
a) annoy b) ruin c) to move back d) popular
22. Torpid ( )
a) retire b) lacking vigor c) reside d) boredom
23. Veer ( )
a) great courage b) shoot c) to shift d) pride
24. Verbatim ( )
a) sudden change of fortune b) abusive language c) a turmoil d) in the same words
25. Rile ( )
a) to enrage b) to hit c) indulge in riots d) optimistic
26. Surmount ( )
a) to overcome b) to be defeated c) to wait d) to hurry
27. Sanguine ( )
a) to be disappointed b) rehash c) to be hopeful d) to cleanse
28. Vapid ( )
a) charity b) lacking zest c) empty d) stagnate
30. Unequivocal ( )
a) definite b) uncertain c) rein d) sedate
31. Untoward ( )
a) peaceful b) retrace c) rebate d) troublesome
32. Vicious ( )
a) wicked b) involve c) hostile d) neglect
33. Quell ( )
a) to suppress b) to hesitate c) to plead d) to review
34. Ululate ( )
a) to laugh b) to wail c) to protest d) to sing
14

35. Unobtrusive ( )
a) coming forward b) to retain c) not coming forward d) to lean to
36. Turbulent ( )
a) agitated b) proud c) abuse d) wicked
37. Sagacity ( )
a) relief b) to be shrewd c) sedate d) revision
38. Slue ( )
a) reeve b) rebind c) rob d) to twist
39. Restive ( )
a) at rest b) at motion c) relay d) uneasy
40. Remonstrate ( )
a) defeat b) familiar c) to protest d) to approve
41. Quaver ( )
a) raze b) residual c) negligent d) to tremble
42. Malice ( )
a) court b) friend c) ill will d) desire
43. Mirth ( )
a) joviality b) break c) fright d) immortal
44. Oblique ( )
a) slanted b) straight c) follower d) not clear
45. Ornithology ( )
a) branch of chemistry b) branch of botany dealing with onions
c) branch of zoology dealing with birds d) branch of science
46. Putative ( )
a) Commonly accepted b) outdated c) Commonly rejected d) historical
47. Pique ( )
a) act of greediness b) to he happy c) to be irritated d) act of homage
48. Melange ( )
a) scheme b) rugs c) tiles d) mixture of various things
49. Palliative ( )
a) nasty b) dubious c) obey d) to make excuses
50. Nugatory ( )
a) precious b) motivational c) worthless d) gait
51. Livid ( )
a) calm b) soothing c) happy d) enrage
15

52. Oblivion ( )
a) lack of vitamins b) lack of sleep c) lack of air d) lack of memory
53. Lucrative ( )
a) profitable b) chaos c) deceive d) decoration
54. Nostrum ( )
a) fruit b) medicine c) herb d) plant
55. Nettlesome ( )
a) something old b) something pleasant c) something new d) something irritating
56. Ocular ( )
a) hazy b) audible c) visual d) sensual
57. Polemic ( )
a) discouraging b) not appropriate c) controversial d) encouraging
58. Nectarine ( )
a) delicious b) pungent c) freedom d) honey
59. Qualm ( )
a) trend b) manner c) unimaginative d) a sudden feeling of fear
60. Quarry ( )
a) indite b) flatter c) prey d) divine
61. Maudlin ( )
a) to be very emotional b) advance c) worthy d) fantasy
62. Exacerbate ( )
a) to deny b) relevant c) to make worse d) to confer
63. Effusive ( )
a) confused b) impartial c) decorate d) lifeless
64. Benign ( )
a) superb b) harmless c) muster d) amplify
65. Decry ( )
a) unlucky b) trouble c) praise d) criticize
66. Carrion ( )
a) investigate b) penetrate c) dead body d) march
67. Callow ( )
a) deadly b) monument c) hard d) inexperienced
68. Desultory ( )
a) discontinuous b) rejoin c) thoughtful d) wide spread
16

69. Complacent ( )
a) self-satisfied b) stupor c) exhaustion d) unruly
70. Discard ( )
a) perfect b) hostile c) reject d) fact
71. Bland ( )
a) harmony b) delight c) gently soothing d) sublime
72. Ameliorate ( )
a) frequent b) precious c) to improve d) keen
73. Bicameral ( )
a) revolting b) development c) two branches d) reunion
74. Equable ( )
a) disgraceful b) weaken c) effects d) unvarying
75. Cellulite ( )
a) lumpy fat under the skin b) plant c) outline d) solvent

Session-4
ONE WORD SUBSTITUTION WITH MEANING
1. Altruist – one, who considers the happiness and well-being of others first
A person unselfishly concerned for or devoted to the welfare of others (opposed to egoist).
Ex: If I had your altruist emotional temperament, I should not hesitate for a moment.
2. Amputate – to cut off a part of a person's body which is infected
To cut off (all or part of a limb or digit of the body), as by surgery.
Ex: It was necessary for old Doc Robbins to amputate both at the shoulders.
3. Arsenal – a place for ammunition and weapons.
A place of storage or a magazine containing arms and military equipment for land or naval service.
Ex: The arsenal was the main employer in Woolwich, but it was shut down after WWII when the Empire
disappeared and the army shrunk.
4. Archives – a place where government or public records are kept
usually, archives.
Documents or records relating to the activities, business dealings, etc., of a person, family, corporation,
association, community, or nation.
Ex: In this weekly feature, we dig into the NEWSWEEK archives to see how times have changed—or in some cases,
haven't.
17

5. Amateur – a man who does a thing for pleasure and not as a profession.
A person who engages in a study, sport, or other activity for pleasure rather than for financial benefit or
professional reasons, compare professional.
Ex: The entire thing was built by some amateurs with screwdrivers and plywood
6. Ambidexterity – one, who can use either hand with ease
A person who uses both hands with equal facility.
Ex: Being an ambidexterity is a gift to few people.
7. Alimony – allowance paid to wife on legal separation
An allowance made to a wife out of her husband's estate or income for her support, upon her divorce or
legal separation from him, or during a suit for the same.
Ex: Her suit was to recover unpaid alimony which was to continue until her remarriage.
8. Anthology – a collection of poems
A collection of flowers of literature, that is, beautiful passages from authors; a collection of poems or
epigrams; -- particularly applied to a collection of ancient Greek epigrams.
Ex: Barely more than a year ago I was sitting at your table and dallying with the thought of publishing an anthology
of your works.
9. Abdication – voluntary giving up of throne in favor of someone
The act of abdicating; the renunciation of a high office, dignity, or trust, by its holder; commonly the
voluntary renunciation of sovereign power; as, abdication of the throne, government, power, authority.
Ex: Barely more than a year ago I was sitting at your table and dallying with the thought of publishing an
anthology of your works.
10. Arbitrator – a person, appointed by two parties to solve a dispute
A person who decides some point at issue between others; one who formally hears and decides a disputed
cause submitted by common consent of the parties to arbitration.
Ex: want of union, want of mutual assistance, want of a common arbitrator to resort to in their differences.
11. Axiom – a statement which is accepted as true without proof
A proposition that is not susceptible of proof or disproof; its truth is assumed to be self-evident
Ex: The true principle may be stated in the form of an axiom.
12. Almanac – an annual calendar with positions of stars
A book or table, containing a calendar of days, and months, to which astronomical data and various statistics
are often added, such as the times of the rising and setting of the sun and moon, eclipses, hours of full tide, etc.
Ex: he is a good maker of almanacs, but no good judge of the weather.
13. Belligerent – a person, nation that is involved in war
A nation or state recognized as carrying on war; a person engaged in warfare.
Ex: All the belligerent powers, excepting Maria Theresa, weary of the long war, were anxious for peace.
18

14. Biennial – an event which happens once in two years


Something which takes place or appears once in two years; esp. a biennial examination.
Ex: The members of each branch of the Legislature are chosen biennially.
15. Blasphemy – the act of speaking disrespectfully about sacred things
An indignity offered to God in words, writing, or signs; impiously irreverent words or signs addressed to, or
used in reference to, God; speaking evil of God; also, the act of claiming the attributes or prerogatives of deity.
Ex: With curses and blasphemy the crowd hurled itself down the stairs to the floor below.
16. Cynosure – centre of attraction
Something that strongly attracts attention and admiration
Ex: Other eyes were regarding him as well; he was the cynosure of many.
17. Connoisseur – a critical judge of any art and craft
A critical judge of any art, particularly of painting, sculpture, or music; one competent to pass a critical
judgment: as, a connoisseur of carvings; a connoisseur of lace.
Ex: He was a connoisseur in every department of art and life, and took care that he was well served.
18. Crusade – a religious war
To go on a crusade; fight a holy war
Ex: The religious crusades to the Holy Land commenced in the year 1095 and lasted to 1270.
19. Cacographist – a person, who is bad in spellings or writing
Pertaining to, or characterized by, cacography; badly written or spelled.
20. Cynic – one, who sneers at the aims and beliefs of his fellow men
Someone who is critical of the motives of others
Ex: He became a cynic in order the more intimately to know the masses.
21. Convalescent – one, who is recovering health
22. Cartographer – one, who draws maps
23. Elegy – a poem of lamentation
24. Epitaph – words which are inscribed on the grave or the tomb in the memory of the buried
25. Ephemeral – lasting one day
26. Effeminate – a person who is womanish
27. Egotism – practice of talking too much about oneself
28. Epicure – one, who is devoted to the pleasure of eating and drinking
29. Fastidious – one, who is very -selective in one's taste
30. Fanatic or Bigot – one, who is filled with excessive and mistaken enthusiasm in religious matters
31. Facsimile – an exact copy of handwriting, printing etc
32. Fratricide – murder of brother
19

33. Fugitive – one, who runs away from justice or the law
34. Granary – a place for grains
35. Gregarious – animals which live in flocks
36. Hangar – a place for housing aero planes
37. Hive – a place for bees
38. Hearse – a vehicle which is used to carry a dead body
39. Hedonist – one, who believes that pleasure is the chief good (sensual)
40. Horizon – a line at which the earth and the sky seem to meet
41. Honorary – holding office without any remuneration
42. Heretic – one, who acts against religion
43. Incorrigible – incapable of being corrected
44. Invincible – one, too strong to be overcome
45. Indelible – that cannot be erased
46. Incognito – travelling under another name than one's own
47. Indefatigable – one, who does not tire easily
48. Infallible – one, who is free from all mistakes and failures
49. Itinerant – one, who journeys from place to place
50. Infirmary – a home or room used for ill or injured people
51. Inflammable – liable to catch fire easily
52. Interregnum – a period of interval between two reigns or governments
53. Loquacious – one, who talks continuously
54. Lapidist – one, who cuts precious stones
55. Novice or Tyro – one, new to anything, inexperienced
56. Obstetrician – one, who is skilled in midwifery
57. Ostler – one, who looks after horses at an inn
58. Potable – fit to drink
59. Philatelist – one, who collects stamps
60. Plagiarism – literary theft or passing off an author's original work as one's own
61. Philogynist – a lover of womankind
62. Plebiscite – (a decision made by) votes of all qualified citizens
63. Philanderer – one, who amuses himself by love making
64. Philistine – one who does not care for art and literature
65. Pseudonym – an imaginary name assumed by an author for disguise
66. Panacea – a remedy for all diseases
67. Platitude – ordinary remarks often repeated
20

68. Pedant – one, who makes a vain display of his knowledge


69. Paleography – the study of ancient writing
70. Posse – a number of policemen called to quell a riot
71. Parole – pledge given by a prisoner for temporary release, not to escape
72. Quarantine – an act of separation from other persons to avoid infection
73. Rhetoric – the art of elegant speech or writing
74. Sacrilege – violating or profaning religious things/places
75. Somnambulist – a person, who walks in sleep
76. Somniloquist – a person, who talks in sleep
77. Souvenir – a thing kept as a reminder of a person, place or event
78. Swan song – the last work (literary) of a writer
79. Sinecure – a job with high salary but little responsibility
80. Sanatorium – a place for the sick to recover health

SESSION-5
WORD CLASSIFICATION

Exercise
Directions to Solve
In each of the following questions, five words have been given out of which four are alike in some manner, while
the fifth one is different. Choose the word which is different from the rest.
I. Choose the word which is different from the rest
1. A. Chicken B. Snake C. Swan D. Crocodile E. Frog ( )
2. A. Cap B. Turban C. Helmet D. Veil E. Hat ( )
3. A. Kiwi B. Eagle C. Emu D. Ostrich ( )
4. A. Rigveda B. Yajurveda C. Atharvaveda D. Ayurveda E.Samveda ( )
5. A. Curd B. Butter C. Oil D. Cheese E.Cream ( )
6. A. Chicken B. Snake C. Swan D. Crocodile E.Frog ( )
7. A. Cap B. Turban C. Helmet D. Veil E. Hat ( )
8. A. Potassium B. Silicon C. Zirconium D. Gallium E. Germanium ( )
9. A. Tea B. Cinchona C. Rubber D. Cardamom E.Chalk ( )
10.A. Hangar B. Platform C. Dock D. Park E.Bus-stand ( )
11.A. Sparrow B. Swan C. Parrot D. Koel ( )
21

12.A. Tall B. Huge C. Thin D. Sharp E.Small ( )


13. A. Pear B. Apple C. Litchi D. Guava E.Orange ( )
14.A. Dagger B. Hammer C. Knife D. Sword E.Blade ( )
15.A. Kanpur B. Allahabad C. Varanasi D. Mathura ( )
16.A. Oyster B. Clam C.Scallop D. Mussel ( )
17.A. Deck B. Quay C. Stern D. Bow E.Mast ( )
18.A. Producer B. Director C. Investor D. Financier E.Entrepreneur ( )
19.A. Tricycle B. Trident C. Trifle D. Tricolour E.Trilogy ( )
20.A. Chameleon B. Crocodile C. Alligator D. Locust E.Salamander ( )
21.A. Calendar B. Year C. Date D. Month E. Day ( )
22.A. Mumbai B. Cochin C. Kandla D. Mysore E.Vishakhapatnam ( )
23.A. Cumin B. Groundnut C. Cinnamon D. Pepper E.Clove ( )
24.A. Biscuits B. Chocolate C. Cake D. Bread E.Pastry ( )
25. A. Sweep B. Wipe C. Scrub D. Wash E.Stain ( )
26. A. Coat B. Shirt C. Blouse D. Trousers E.Sweater ( )
27.A. Asia B. Argentina C. Africa D. Australia E.Antarctica ( )
28.A. Japan B. India C. Sri Lanka D. New Zealand ( )
29.A. Spectacles B. Goggles C. Binoculars D. Microphone E.Telescope ( )
30.A. Shehnai B. Bagpipe C. Flute D. Sitar E. Harmonica ( )
31.A. Cheetah B. Lion C. Bear D. Tiger E. Leopard ( )
32A. Sheep B. Gazelle C. Ibex D. Shrew E. Tapir ( )
33.A. Flood B. Hurricane C. Avalanche D. Earthquake E.Explosion ( )
34.A. Ant B. Bee C. Moth D. Midge E. Spider ( )
35.A. Flute B. Guitar C. Sitar D. Violin E. Veena ( )
36.A. Treachery B. Fraud C. Deceit D. Swindle E. Morbid ( )
37.A. Lima B. Algiers C. New York D. Tokyo E. Beijing ( )
38.A. Reader B. Writer C. Printer D. Publisher E. Reporter ( )
39.A. Arrow B. Axe C. Knife D. Dagger E. Sword ( )
22

40.A. Feathers B. Tentacles C. Scales D. Pseudopodia E.Flagella ( )


41.A. Dog B. Horse C. Goat D. Cat E. Fox ( )
42.A. House B. Cottage C. School D. Palace E. Hut ( )
3.A. Physics B. Chemistry C. Geography D. Botany E. Zoology ( )
44.A. Football B. Volleyball C. Cricket D. Chess E. Hockey ( )
45.A. Trunk B. Tree C. Fruit D. Leaf E. Flower ( )
46.A. Giraffe B. Hyena C. Deer D. Rhinoceros E. Zebra ( )
47.A. Poland B. Greece C. Spain D. Italy E. Korea ( )
48.A. Bajra B. Mustard C. Rice D. Wheat E. Barley ( )
49.A. Cot B. Sheet C. Quilt D. Pillow E. Blanket ( )
50.A. Assassinate B. Kill C. Kidnap D. Stab E. Murder ( )

SESSION-5

LOGOTHERAPY
Attitude --- Success --- Purpose in life ---- Failure, how to counter it

ATTITUDE
Attitude is a psychological construct, it is a mental and emotional entity that inheres in, or characterizes a
person. They are complex and an acquired state through experiences. It is an individual's predisposed state of
mind regarding a value and it is precipitated through a responsive expression toward a person, place, thing, or
event (the attitude object) which in turn influences the individual's thought and action. Prominent
psychologist Gordon Allport once described attitudes as "the most distinctive and indispensable concept in
contemporary social psychology."
An attitude can be as a positive or negative evaluation of people, objects, events, activities, and ideas. It could be
concrete, abstract or just about anything in your environment, but there is a debate about precise definitions.
Eagly and Chaiken, for example, define an attitude as "a psychological tendency that is expressed by evaluating a
particular entity with some degree of favor or disfavor.”
23

Factors affecting formation of attitudes


Family
Family plays a significant role in the primary stage of attitudes held by individuals. Initially, a person develops
certain attitudes from his parents, brothers, sister, and elders in the family. There is a high degree of relationship
between parent and children in attitudes found in them.
Society
Societies play an important role in formatting the attitudes of an individual. The culture, the tradition, the
language, etc., influence a person's attitudes. Society, tradition, and the culture teach individuals what is and what
is not acceptable.
Political
Political factors such as ideologies of political parties, political leaders and political stability affect the attitudes of
people.
Economic
A person's attitude also depends on issues such as his salary, status, work as such, et
3 Components of Attitude
Cognitive component The cognitive component of attitudes refer to the beliefs, thoughts, and attributes
that we would associate with an object. Many times a person's attitude might be based on the negative and
positive attributes they associate with an object.
Affective component The affective component of attitudes refer to your feelings or emotions linked to an
attitude object. Affective responses influence attitudes in a number of ways. For example, many people are
afraid/scared of spiders. So this negative affective response is likely to cause you to have a negative attitude
towards spiders.
Behavioural component The behavioural component of attitudes refer to past behaviours or experiences
regarding an attitude object. The idea that people might infer their attitudes from their previous actions. This
idea was best articulated by Bem.
Frankl and LogoTherapy
Viktor Emil Frankl (26 March 1905 – 2 September 1997) was an Austrian neurologist and psychiatrist as well as
a Holocaust survivor. Frankl was the founder of logo therapy, which is a form of existential analysis, the "Third
Viennese School of Psychotherapy". His best- selling book Man's Search for Meaning(published under a different
title in 1959: From Death-Camp to Existentialism, and originally published in 1946 as Trotzdem Ja Zum Leben
Sagen: Ein Psychologe erlebt das Konzentrationslager, meaning Nevertheless, Say "Yes" to Life: A Psychologist
Experiences the Concentration Camp) chronicles his experiences as a concentration camp inmate, which led him to
discover the importance of finding meaning in all forms of existence, even the most brutal ones, and thus, a
reason to continue living. Frankl became one of the key figures in existential therapy and a prominent source of
inspiration for humanistic psychologists.
24

From 1933 to 1937, Frankl completed his residency in neurology and psychiatry at the Steinhof Psychiatric
Hospital in Vienna. He was responsible for the so-called Selbstmörderpavillon, or "suicide pavilion". Here, he
treated more than 30,000 women who had suicidal tendencies. In 1937, he established an independent private
practice in neurology and psychiatry at Alser Strasse 32/12 in Vienna.
Beginning with the Nazi takeover of Austria in 1938, he was prohibited from treating "Aryan" patients due to his
Jewish identity. In 1940 he started working at the Rothschild Hospital, where he headed its neurological
department. This hospital was the only one in Vienna to which Jews were still admitted. His medical opinions
saved several patients from being euthanised via the Nazi euthanasia program.

On 25 September 1942, Frankl, his wife, and his parents were deported to the Nazi Theresienstadt Ghetto. There
Frankl worked as a general practitioner in a clinic. When his skills in psychiatry were noticed, he was assigned to
the psychiatric care ward in block B IV, establishing a camp service of "psychohygiene" or mental health care. He
organized a unit to help camp newcomers to overcome shock and grief. Later he set up a suicide watch, assisted
by Regina Jonas.
On 29 July 1943, Frankl organized a closed event for the Scientific Society at Theresienstadt, and with the help
of Leo Baeck, offered a series of open lectures, including "Sleep and Sleep Disturbances", "Body and Soul",
"Medical Care of the Soul", "Psychology of Mountaineering", "How to keep my nerves healthy?", "Medical
ministry", "Existential Problems in Psychotherapy", and "Social Psychotherapy".His father Gabriel died of
pulmonary edema and pneumonia at Theresienstadt.
On 19 October 1944, Frankl and his wife Tilly were transported to the Auschwitz concentration camp, where he
was processed. He was moved to Kaufering, a camp affiliated with Dachau, on 25 October, where he spent five
months working as a slave laborer. In March 1945, he was offered a move to the so-called rest camp, Türkheim,
also affiliated with Dachau, where he worked as a physician until 27 April 1945, when the camp was liberated by
American soldiers.
Frankl's mother Elsa and brother Walter died at Auschwitz. His wife was moved to Bergen-Belsen, where she died.
The only other survivor of the Holocaust among Frankl's immediate family was his sister, Stella, who had
emigrated from Austria to Australia.
After enduring the suffering in these camps, Frankl concluded that even in the most absurd, painful, and
dehumanized situation, life has potential meaning and that, therefore, even suffering is meaningful.
This conclusion served as a basis for his logotherapy and existential analysis, which Frankl had described before
World War II. He said, "What is to give light must endure burning."
An example of Frankl's idea of finding meaning in the midst of extreme suffering is found in his account of an
experience he had while working in the harsh conditions of the Nazi concentration camps:
25

We stumbled on in the darkness, over big stones and through large puddles, along the one road leading from the
camp. The accompanying guards kept shouting at us and driving us with the butts of their rifles. Anyone with very
sore feet supported himself on his neighbor's arm. Hardly a word was spoken; the icy wind did not encourage talk.
Hiding his mouth behind his upturned collar, the man marching next to me whispered suddenly: "If our wives
could see us now! I do hope they are better off in their camps and don't know what is happening to us."

That brought thoughts of my own wife to mind. And as we stumbled on for miles, slipping on icy spots, supporting
each other time and again, dragging one another up and onward, nothing was said, but we both knew: each of us
was thinking of his wife. Occasionally I looked at the sky, where the stars were fading and the pink light of the
morning was beginning to spread behind a dark bank of clouds. But my mind clung to my wife's image, imagining it
with an uncanny acuteness. I heard her answering me, saw her smile, her frank and encouraging look. Real or not,
her look was then more luminous than the sun which was beginning to rise.
A thought transfixed me: for the first time in my life I saw the truth as it is set into song by so many poets,
proclaimed as the final wisdom by so many thinkers. The truth – that love is the ultimate and the highest
goal to which Man can aspire. Then I grasped the meaning of the greatest secret that human poetry and
human thought and belief have to impart: The salvation of Man is through love and in love. I understood
how a man who has nothing left in this world still

may know bliss, be it only for a brief moment, in the contemplation of his beloved. In a position of utter
desolation, when Man cannot express himself in positive action, when his only achievement may consist in
enduring his sufferings in the right way – an honorable way – in such a position Man can, through loving
contemplation of the image he carries of his beloved, achieve fulfillment. For the first time in my life I was
able to understand the meaning of the words, "The angels are lost in perpetual contemplation of an
infinite glory."

LOGOTHERAPY
Logotherapy is a term derived from “logos,” a Greek word that translates as “meaning,” and therapy, which is
defined as treatment of a condition, illness, or maladjustment.
Logotherapy was developed by neurologist and psychiatrist Viktor Frankl. It is considered the "Third
Viennese School of Psychotherapy" along
with Freud's psychoanalysis and Adler's individual psychology.
Developed by Viktor Frankl, the theory is founded on the belief that human nature is motivated by the
search for a life purpose; logotherapy is the pursuit of that meaning for one's life. Frankl's theories were
heavily influenced by his personal experiences of suffering and loss in Nazi concentration camps
26

Logotherapy is based on an existential analysis focusing on Kierkegaard's will to meaning as opposed to


Adler's Nietzschean doctrine of will to power or Freud's will to pleasure. Rather than power or pleasure,
logotherapy is founded upon the belief that it is the striving to find a meaning in one's life that is the primary,
most powerful motivating and driving force in humans. A short introduction to this system is given in Frankl's
most famous book, Man's Search for Meaning, in which he outlines how his theories helped him to survive
his Holocaust experience and how that experience further developed and reinforced his theories. Presently, there
are a number of logotherapy institutes around the world.
The notion of Logotherapy was created with the Greek word logos ("word"). Frankl’s concept is based on the
premise that the primary motivational force of an individual is to find a meaning in life. The following list of tenets
represents basic principles of logotherapy:
Life has meaning under all circumstances, even the most miserable ones.
Our main motivation for living is our will to find meaning in life.
We have freedom to find meaning in what we do, and what we experience, or at least in the stance we take
when faced with a situation of unchangeable suffering.
Components of Franklian Philosophy
There are three main components that are at the heart of the Franklian philosophy:
1. Each person has a healthy core.
2. The primary focus is to enlighten a person to their own internal resources and provide them with the tools to
use their inner core.
3. Life offers you purpose and meaning; it does not owe you a sense of fulfillment or happiness.

Finding Meaning with Logotherapy


Logotherapy is based on the premise that humans are driven to find a sense of meaning and purpose in life.
According to Frankl, life’s meaning can be discovered in three different ways:
1. By creating a work or accomplishing some task
2. By experiencing something fully or loving somebody
3. By the attitude that one adopts toward unavoidable suffering

Frankl believed that suffering is a part of life, and that man’s ultimate freedom is his ability to choose how to
respond to any set of given circumstances, even the most painful ones. Additionally, people can find meaning in
their lives by identifying the unique roles that only they can fulfill. For example, when a man consulted with Frankl
due to severe depression following the death of his wife, Frankl asked him to consider what would have happened
if he had died first and his wife had been forced to mourn his death. The man was able to recognize that his own
suffering spared his wife from having that experience, which served as a curative factor and helped relieve his
depression.
27

The human spirit is referred to in several of the assumptions of logotherapy, but the use of the term spirit is not
"spiritual" or "religious". In Frankl's view, the spirit is the will of the human being. The emphasis, therefore, is on
the search for meaning, which is not necessarily the search for God or any other supernatural being. Frankl also
noted the barriers to humanity's quest for meaning in life. He warns against "...affluence, hedonism,
[and] materialism..." in the search for meaning.

Purpose in life and meaning in life constructs appeared in Frankl's logotherapy writings with relation to existential
vacuum and will to meaning, as well as others who have theorized about and defined positive
psychological functioning. Frankl observed that it may be psychologically damaging when a person's search for
meaning is blocked. Positive life purpose and meaning was associated with strong religious beliefs, membership in
groups, and dedication to a cause, life values, and clear goals. Adult development and maturity theories include
the purpose in life concept. Maturity emphasizes a clear comprehension of life's purpose, directedness, and
intentionality which contributes to the feeling that life is meaningful.

LOGOTHERAPY TECHNIQUES
The three main techniques of logotherapy are:
1. Dereflection: Dereflection is used when a person is overly self-absorbed on an issue or attainment of a goal.
By redirecting the attention, or dereflecting the attention away from the self, the person can become whole
by thinking about others rather than themselves.
2. Paradoxical intention: Paradoxical intention involves asking for the thing we fear the most. For people who
experience anxiety or phobias, fear can paralyze them. But by using humor and ridicule, they can wish for the
thing they fear the most, thus removing the fear from their intention and relieving the anxious symptoms
associated with it.
3. Socratic dialogue: Socratic dialogue is a technique in which the logotherapist uses the own person's words as
a method of self-discovery. By listening intently to what the person says, the therapist can point out specific
patterns of words, or word solutions to the client, and let the client see new meaning in them. This process
allows a person to realize that the answer lies within and is just waiting to be discovered.
References:
1. Biography. (n.d). Victor Frankl Institut. Retrieved from
2. https://www.viktorfrankl.org/e/chronology.html
3. Bulka, R.P. (1978). Is logotherapy authoritarian? Journal of Humanistic Psychology,18(4), 45-54.
4. Delavari, H., Nasirian, M., & Baezegar bafrooei, K. (2014). Logo therapy effect on anxiety and depression in
mothers of children with cancer. Iranian Journal of Pediatric Hematology Oncology, 4(2), 42-48.
5. Frankl, V.E. (1959). Man’s search for meaning. Boston: Beacon Press.
6. Smith, A.J. (2013). Logotherapy to treat substance abuse as a result of military-related PTSD. Journal of
Military and Government Counseling, 1(1), 61-74.
28

Case Study 1
KELLOGG: Is the Two-Tier System Ethically Problematic

The 226 workers at Kellogg’s Memphis plant have been “locked-out” from their jobs producing Frosted Flakes and
Froot Loops for over 3 months. Company management and the union representing the workers — the Bakery,
Confectionery, Tobacco Workers and Grain Millers International Union — reached a stalemate in negotiations in
October, resulting in the lockout. The primary issue is Kellogg’s demand of dramatically increasing the amount of
temporary workers, who would earn $6 less and be entitled to much fewer benefits: effectively creating a two-tier
system at the plant. Under the current agreement, Kellogg has the right to use temporary workers for up to 30%
of the workforce, but the union claims Kellogg is now pushing for 100%. The workers, who have had their health
insurance suspended by Kellogg, fear that their jobs will either be replaced entirely by temporary workers, or they
will be forced to take lower wages. Kellogg, in the midst of a 4-year cost reduction effort labeled, “Project K,”
claims that the change is necessary to remain competitive and that current employees will be unaffected by the
change. Are two-tier systems ethically problematic?
Kirk: Kellogg has ignored lessons learned by the airline industry that dividing employees into two classes of citizens
won’t work for very long. American Airlines, with a host of others, started a plan in 1983 that instituted a two-tier
system separating current employees from future hires into different pay scales. By 1987, the company had to
significantly overhaul the program. Two-tier systems create tension between the employees, resentment of
management, higher turnover, and further complicate union relations. In the long run, these programs are not
sustainable. They undermine the concept of shared sacrifice, shared reward, and make development of a strong
corporate culture exponentially more difficult.
Patrick: I don’t think tiered systems are inherently unethical, although it is largely a matter of fairness. In the case
of American Airlines, and most likely the Kellogg lockout, new hires will be doing the same job as existing
employees but will get paid significantly less. “Treat similar cases similarly” goes a long way here. Anything else
will create an imbalance and undermine the company in the long run. On the flip side, Google famously uses a
tiered system, assigning different color “badges” for full-time employees, contractors, and interns. Yes, they
create divisions between the groups, but they also strengthen the group identity of the subgroups and incentivize
employees to “climb the ladder.” It’s Darwinian, but fair.
Labor Battle at Kellogg Plant in Memphis Drags On (NY Times)
A Framework for Thinking Ethically (Markkula Center for Applied Ethics)
29

Case Study 2
HOLDING THE CEO'S FEET TO THE FIRE: Should Chief Execs Be Penalized for Failing to Appoint Women to Senior
Positions?

“Chief executives should be challenged for explanations and even have their pay cut if they fail to appoint women
to senior positions,” said the Business Council of Australia in a letter this week to its members. The BCA, the
representative body of the chief executives of Australia’s 100 largest companies, is urging its members to adopt a
“checklist of reforms” aimed at addressing the underrepresentation of women in senior positions, and to consider
docking CEO pay if they do not
implement the reforms. In Australia, women have been outpacing men in earning college degrees since 1985 and
make up 46% of the workforce, but hold only 16% of board positions and 3.5% of chief executive roles. The BCA
aims to double the number of women in senior positions in the next 10 years, and claims this isn’t just an equality
issue, but also an economic issue: “We risk not getting the best talent for the job.” Even if we take for granted that
equal opportunity for women in senior positions is a laudable goal, is tying executive compensation to the
promotion of women ethically problematic? (Here in Silicon Valley, the Silicon Valley Business Journal reports that
only 4% of executive positions and 9% of board positions in Silicon Valley are held by women.)
Kirk: Unfortunately, progress in cracking the glass ceiling, in Australia or in Silicon Valley, has been glacial without
an effective "stick" to prod it along - be it regulation or board-imposed pay cuts. The Australian business council
needs the cooperation of corporate boards - but of course there are so few women on those boards the issue may
be ignored. Reluctantly, I have to conclude that only government regulation and requirements, in some form, can
bring about the needed change. In the interim, certainly corporate boards should set specific goals for their CEOs -
and for themselves. The CEOs pay should be cut if he or she does not make progress; the boards themselves
should admit they are failing if they don't make progress.
Patrick: People respond to incentives. Whether they are social, ethical, or in this case, economic, incentives bring
about change. We’d like to think that this is a problem that figures itself out over time, but it’s clear that there are
biases in play that prevent this from happening. Tying executive pay to fair hiring practices will ensure that this
problem is addressed, despite making it an issue of compliance instead of conscience. Hopefully measures like this
are only needed for a period of time, like a ladder you throw away after you have climbed up it.
Business Council.

Case Study 3
When Extra Attention Crosses the Line
Annie has been recently hired full time at a major tech company where she interned for two summers during her
college career. Annie loves her job and has established many strong relationships with her co-workers over the
time she has worked there. The company encourages the interns and new hires to interact with VPs and upper
management in order to create an open and friendly atmosphere.
30

During her time as an intern, Annie began to notice that one of the VPs paid her extra attention. When
he was around he would always make an extra effort to stop by Annie's cubicle and chat: something he did not do
with any of the other interns. He reached out to her over social networking sites and even invited her to a
gathering at his house. Some of her co-workers began to make offhand comments to Annie about the extra
attention.
Now that she was in a full time position, Annie began to dread that she would soon have to work with this VP
directly. While he has not done or said anything explicitly inappropriate, the extra attention—and the fact that her
co-workers noticed it—made her very uncomfortable and undermined her concentration on work. When she was
hired, she was told that she should always speak to her manager if she was uncomfortable or had issues with the
work environment. While at the same time, she is afraid to come across like a tattletale since the VP hasn't
explicitly done anything wrong.
What course of action should Annie take?

Case Study 4
Quality Management: Signing Off on a Substandard Product

Lauren's first job after graduation from Santa Clara University was working as a quality engineer with a highly
respected technology company. She had to monitor the manufacturing process and make sure that all products
met customer specifications. Just three months into her position, the company booked a very large deal with a
strategic customer, helping establish the company's dominance in the industry.
Specifically, Lauren's company was designing a device that would be integrated into another company's product.
The customer contracted out this work because they were experiencing rapid growth and cannot meet demand
otherwise. They picked Lauren's company because of its good reputation and fast turnaround time. Lauren's role
was to test the new device and make sure it met technical and environmental specifications, particularly
functionality under extreme conditions, such as high humidity.
The test results showed that the products did not meet the quality standards agreed upon, but only by a very
small margin. Her general manager instructed her to push it through anyway, stating that the risk of failure was
not great enough to delay mass production. Moreover, the likelihood of the product ever being placed in such
extreme situations was so small that the manager did not feel jeopardizing the contract was worth it.
Lauren spoke to her immediate boss, who worked under her general manager, and he also advocated pushing the
product through to production. She was faced with the choice of ignoring company protocols or going against
management. Sweeping the problem under the rug would require Lauren to sign off on a report that she knew to
be fraudulent. She also knew that if she went to upper management her working relationships with her immediate
bosses would be strained, maybe even preventing her success in the company. Not to mention, the company
would have to delay production and possibly lose the contract. What should Lauren Do?
31

Case Study 5
Protected Class: Externalities of Age Discrimination Safeguards
Lindsey worked as a top manager at a struggling technology company in Silicon Valley. As part of a companywide
initiative, she had the task of downsizing her department by a considerable margin. Among the most troubling
decisions involved eliminating a position within her department's most productive teams: eight people for seven
jobs. As she considered each team members'; contributions and merits, there were two employees whose
performance reviews were far behind the rest of the team.
Dianne was a 38-year-old woman, an employee at the company for 12 years, and an average performer. She
worked hard and did a decent job overall, but failed to thrive at the company. She worked for a mediocre manager
and Lindsey thought Diane's performance would improve if she worked for a more competent manager. Lindsey
felt that Diane had more potential than Ron, but up until now it had not been realized.
Ron was a 42-year-old male with tenure and experience in the firm similar to Diane. Like Diane, he was an average
performer but was not a rising star in the organization. He did not show as much potential as Dianne. However,
because Ron was over 40, he was considered a member of a "protected class," giving him special protections
against discrimination based on age. If Lindsey fired him, he could, and most certainly would, sue the company
with a claim that he was being let go because of age discrimination.
Lindsey felt that Dianne was the slightly better candidate, given her potential to grow into a top contributor. On
the other hand, eliminating Ron's position would expose the company to a lawsuit and the expenses associated
with it, perhaps outweighing any benefit the company would gain by choosing Diane over Ron. What should
Lindsey do?

Case Study 6
Financial Reporting: Do Small Errors Need to be Reported?
Ben is a recent Santa Clara University graduate who has just started his first job in the finance department of a
publicly traded Silicon Valley company. One of his main responsibilities is to create and distribute extensive
Microsoft Excel reports that analyze costs and revenues for different divisions. Ben sends completed reports to his
direct supervisor and the CFO. The CFO then uses the information to create the company's financial reports, in
addition to the strategy and forecasting formulation.

While Ben considers himself to be detailed-oriented, the complicated nature of and the sheer volume of data
sometimes overwhelm him, which is exacerbated by their strict deadlines. While Ben works hard to prepare the
reports as accurately as possible, he often finds errors after he has submitted his final report. When the errors are
critical, he revises the reports and resends them. However, some of the errors are minor, in Ben's estimation, and
he doubts that the CFO will use or look at these figures. Ben is ambitious and wants to be promoted, but worries
32

that if he frequently sends out revised reports he will appear unreliable and unqualified. At the same time, the
potential consequences from inaccurate financial reports put the company, the CFO and CEO, and Ben himself at
risk. What actions should Ben take when he catches a mistake? Is he obligated to report every error, particularly
since he works for a publicly traded company? Is there such a thing as a small error in this context?

1. Receiving a Holiday Gift:


A supplier sends a basket of expensive foodstuffs to your home at Christmas with a card: "We hope you and your
family enjoy the 'goodies.'". What action(s) might you want to take?
2. Sales Expense:
The purchasing manager for a large company agrees to give you an order (their first), expecting you agree to make
a $200 donation to his favorite charity, a local youth sports team. How do you respond?
3. Sales Expense Reimbursement:
A customer executive from Southeast Asia will visit your HQ facility and meet with your executive team. Your
independent Southeast Asian agent requests that you reimburse the customer for his expenses, including
expenses that could violate your company's policies. The agent will reimburse you. How do you proceed?
4. References:
A large, prospective client calls you and asks about a competitor's reputation. One of your long time customers
had a very bad experience with this competitor. What information do you share with the prospect? How do you
respond to the prospect call?
5. Gratuities:
A customer has a large sailing yacht on a vessel that your company will be discharging. The customer is present
and is watching the off-loading operation.
The five stevedores you manage pull off a very tricky maneuver, safely transferring the yacht to the trailer. The
customer is elated, and reaches into his pocket, pulling out a big wad of $50 bills. What do you do?
6. Conflict of Interest:
As department manager, you are hosting an informal celebration in the office. The food budget is $200. Your next
door neighbor has just started her own catering business and asks to supply the food. Since she is just starting out,
she'll do it at cost and provide extra items at no charge. What might you want to consider?
7. More Competition:
You are in a head-to-head battle with your arch competitor, Evil Enterprises. One of your co-workers approaches
you. He has recently joined your company after having worked for a second competitor for several years.
He suggests, "I made notes on all of Evil's bids when I could get the data. They use some clear cost standards.
Would you like me to bring my notes to the office tomorrow and let you look through them?" How do you
respond?
33

7. The Case of the Plant Relocation


By Karen Musalo
Production costs are rising. Your company can make more money for shareholders by relocating your plants to a
country with lower labor costs and fewer regulations. Using this case, Stan Raggio, senior vice president for
sourcing and logistics at The Gap, and Karen Musalo, then director of the Markkula Center for Applied Ethics
International Human Rights and Migration Project, discussed the ethical issues companies should consider at an
Ethics Roundtable for Executives.
You are the chief executive of Electrocorp, an electronics company, which makes the onboard computer
components for automobiles. In your production plants, complex hydrocarbon solvents are used to clean the chips
and other parts that go into the computer components. Some of the solvents used are carcinogens and must be
handled with extreme care. Until recently, all of your production plants were located in the United States.
However, the cost of production has risen, causing profits to decline.
A number of factors have increased production costs. First, the union representing the workers in your plant
waged a successful strike resulting in increased salary and benefits. The pay and benefits package for beginning
employees is around $15/hour. A second factor has been stringent safety regulations. These safety procedures,
which apply inside the plant, have been expensive in both time and money. Finally, environmental regulations
have made Electrocorp's operations more costly. Electrocorp is required to put its waste through an expensive
process before depositing it at a special disposal facility.
Shareholders have been complaining to you about the declining fortunes of the company. Many of Electrocorp's
competitors have moved their operations to less-developed countries, where their operating costs are less than in
the United States, and you have begun to consider whether to relocate a number of plants to offshore sites.
Electrocorp is a major employer in each of the U.S. cities where it is located, and you know that a plant closure will
cause economic dislocation in these communities. You know that the employees who will be laid off because of
plant closures will have difficulty finding equivalent positions and that increased unemployment, with its
attendant social costs, will result. However, you are aware of many other corporations, including your
competitors, that have shut down their U.S. operations, and it is something that you are willing to consider.
You have hired a consultant, Martha Smith, to investigate the sites for possible plant relocation. Ms. Smith has
years of experience working with companies that have moved their operations to less-developed countries to
reduce their operating costs. Based on your own research, you have asked Ms. Smith to more fully investigate the
possibility of operations in Mexico, the Philippines, and South Africa. A summary of her report and
recommendation for each country follows:
34

Mexico
A number of border cities in Mexico would be cost-efficient relocation sites based on both labor, and health and
safety/environmental factors. Workers in production plants comparable to Electrocorp's earn about $3 per day,
which is the prevailing wage. There is frequent worker turnover because employees complain that they cannot live
on $3/day, and they head north to work illegally in the United States. However, a ready supply of workers takes
their place.
Mexican health and environmental laws are also favorable to production. Exposure to toxic chemicals in the
workplace is permitted at higher levels than in the United States, allowing corporations to dispense to some
degree with costly procedures and equipment. Mexico's environmental laws are less strict than those of the
United States, and a solvent recovery system, used to reduce the toxicity of the waste before dumping, is not
required.
The only identifiable business risk is possible bad publicity. The rate of birth defects has been high in many
Mexican border towns where similar plants are in operation. Citizen health groups have begun protests, accusing
the companies of contamination leading to illness.
Philippines
Conditions in the Philippines are more favorable than those in Mexico in terms of labor and health and
safety/environmental factors.
The prevailing wage in the Philippines is about $1/day, and young workers (under 16) may be paid even less. As in
Mexico, the workers complain that the rate of pay is not a living wage, but it is the present market rate.
The health and safety and environmental regulations are equivalent to those in Mexico, but there have been no
public complaints or opposition regarding birth defects, cancers, or other illnesses.
South Africa
Conditions in South Africa are positive in some respects, but not as favorable in economic terms as in Mexico or
the Philippines. The prevailing wage in South Africa is about $10/day. Furthermore, there is a strong union
movement, meaning that there may be future demands for increases in wages and benefits.
The unions and the government have been working together on health and safety issues and environmental
protections. Exposure to toxic chemicals in the workplace is not permitted at as high a level as in Mexico and the
Philippines. Although the equipment necessary to reduce toxic chemicals to an acceptable level is not as costly as
in the United States, this expense would not be incurred in the other two countries. Furthermore, there are
requirements for a solvent recovery system, which also increases operation expenses.
You have to decide how you would like to proceed. Your options are to further investigate one or more of the
overseas sites or to simply continue all operations within the United States. Examine each possibility and the
factors you will consider in weighing the pros and cons.
35

8. The Case of Nutritional Foods


By Kirk O. Hanson
What do we do when products go wrong? That question was explored by the Ethics Center's Ethics Roundtable for
Executives at a September meeting featuring Greg Steltenpohl, chair of Odwalla Inc.,
and Kirk O. Hanson, director of Stanford University's Sloan Program at the Graduate School of Business. To
facilitate discussion of the issue, Hanson created the following fictitious case. It does not represent a real event,
but it does provide a framework for looking at questions of product responsibility. The case is presented in four
parts to mimic how such a scenario might evolve in real time. At each break in the case, stop and ask yourself what
you would do given the information you have.
First Warnings
Fred James, chief executive of Nutritional Foods Inc., a $50 million manufacturer of healthful foods, listened with
concern as John Healy, his vice president for production, described reports that had come in during the past hour.
The reports came from two county health departments, one in Seattle and the other in Southern California. In
each case, the health department official reported a possible link between acute food poisoning of a child and an
unpasteurized apple product produced by Nutritional Foods and distributed throughout the Western United
States. The health departments had not yet ruled out all other possible causes. Additional information was not yet
available, and Healy did not have batch numbers for the products in question.
Nutritional Foods was rapidly becoming the best-known brand of natural or nonpasteurized foods in the Western
United States. It made its products in two facilities, one in California's Central Valley and the other in a coastal city
of Central California. Fresh fruit and vegetable products were shipped from growing regions throughout the West
to these two facilities for processing and canning or bottling. The handling of nonpasteurized products was critical
as contamination could occur in picking, transporting, or processing the fresh product.
Distribution was also critical to the freshness and safety of the company's products. Daily distribution from the
company's processing facilities in company-owned refrigerated trucks ensured freshness.
Unpasteurized products had been popular in the health-food market for many years, but Nutritional Foods was
the most successful of several companies seeking to appeal to the mainstream market as well as to the niche
consumer. The company's success had led to its rapid growth and the construction of its new processing facility in
the Central Valley.
"OK, John," said James, "what's our response? Do two 'maybes' mean we should do something immediately? We
have had an occasional report, perhaps one every couple of months, during the past two years. None of those
turned out to be traceable to our product. Do two reports represent anything other than a statistical quirk? Should
we be doing anything but waiting for the final reports from the health departments in a couple of days?"
Concern Deepens
Healy dispatched company managers to the two counties where initial reports indicated there might be acute
36

food poisonings related to one of the company's unpasteurized products. He was startled a short time later to
receive a third and fourth report similar to the first two.
Although also not conclusive, the new reports made Healy wonder if something was terribly wrong. Healy
immediately dispatched company managers to the two new counties, urging all four to get the batch numbers of
the products in question. He also asked for an immediate meeting with James.
"Now what should we do?" asked Healy. "Should we warn the retailers, asking them to stop selling the product?
Should we also warn the public? Such a move could devastate the company's reputation and its stock price at a
critical moment. Don't we have an obligation to think long and hard before we take that step? How much certainty
must we have and how serious does a problem have to be for us to proceed?" Time to Act?

9. Product Safety Case Studies


by John McBain and James Balassone
All employees are expected to act according to their organization's Code of Ethics or Conduct, based upon the
values of the organization.
Furthermore, product safety engineers are asked to:
(a) Determine "safety" of products
(b) Obtain various non-governmental agency certifications for products
(c) Confirm that products comply with government regulations
(d) Examine and test products according to various standards
They are required to do this using the minimum time, money, and number of product samples - usually at the end
of the product development process, when changes are more difficult and everyone wants to ship products.

Scenario 1:
Some agencies authorize companies to test products, provide the data to the agency, and ship the product bearing
the agency mark. The company's capability has been evaluated by the agency and a contract signed to allow this.
A new high-end computer is ready to ship - except for one test that you will not complete for another three
weeks. The probability of failure is low - and even if the test fails, corrections can be made and sent out later to
customers. Marketing is VERY anxious to ship because the end of the fiscal quarter is next week.
Should you put on the agency mark and ship while finishing the test?
Your boss tells you that this has occurred before; the company shipped the product, and there was no problem.
He also says that if you do not want to sign off, then he will do so.
What should you do?
Products were shipped before this test was completed - but it happened when you were on a business trip. The
production manager apologizes, but doesn't want to take any action. What should you do?
37

Scenario 2:
The company records-retention policy instructs employees to discard development records and test results for
products five years after End of Life is declared. This policy is in compliance with local legal requirements.
Because of the press of work you have not disposed of some old records, and they are a couple of years over the
limit for the company policy. You finally get time to clean out your files, but you receive a legal request for any
information about the old product that is involved in an injury case. Your records may or may not be applicable to
the case. Should you destroy the records?

Scenario 3:
You have just discovered that a country in the Far East has new regulations that apply to your product. The
requirement is to submit a report and get a file number to apply to your product - after the government
department has given its OK. However, you know from industry contacts that there is no enforcement of the law
at this time.
Should you delay shipping products until they are compliant or take other action?
Products have been held at customs in this country, and you ask a local agent to investigate. Later you hear the
products were released without any changes and without certification.
Should you look further into this event? Why or why not?
You make the identical product at two factories, but only one is authorized to apply a certain certification mark.
Unfortunately, the wrong factory shipped the product to the country that requires the certification mark.
Returning the product and shipping out the identical product with a different mark on the label would be very
expensive and time-consuming, and your customer would be very unhappy. What are your options?

Scenario 4:
Your company's product uses some supplementary circuit protection in larger units. While visiting the factory for
another reason, you tour the production line and notice that the protectors are different from the ones you
originally evaluated. They seem to have the same ratings, but you suspect they may not be suitable as a
substitute.
This product is not your responsibility, and you would have to do some research to figure out if there is a problem.
What course(s) of action should you take to investigate the potential problem?
The production line supervisor tells you the substitution has been approved by the factory safety engineer, but
you are positive these protectors are not suitable.
What should or could you do?
38

Scenario 5:
The latest edition of the standard that applies to your products now has three pages of "safety" markings and
warnings specified. So many warnings about very unlikely situations greatly reduce the impact of warnings that
might prevent dangerous events. You have actually surveyed customers and found that to be true.
Should you reduce the warning labels to only to the important ones or just follow the standards of the certification
agencies?
Your marketing department wants you to color-coordinate and reduce in size the warning labels. The new version
still would comply with the standard, but it would not stand out on the machine.
Should you resist the change?
The factory has a lot of old inventory with silk-screened markings that do not comply with the new requirements,
although they did comply with the previous edition. To change them would cost thousands of dollars. Should you
let the company use up the old stock, although it is technically not in compliance?

Self Study
5 PSYCHOLOGICAL THEORIES OF MOTIVATION TO INCREASE PRODUCTIVITY
Everyone wants to be more productive but getting motivated enough to actually get things done can seem
impossible. Social scientists have been studying motivation for decades, trying to find out what motivates our
behaviour, how and why. Dozens of theories of motivation have been proposed over the years. Here are 5 popular
theories of motivation that can help you increase workplace productivity.

1. Hertzberg’s Two-Factor Theory


The Two-Factor Theory of motivation (otherwise known as dual-factor theory or motivation-hygiene theory) was
developed by psychologist Frederick Herzberg in the 1950s.
Analysing the responses of 200 accountants and engineers who were asked about their positive and negative
feelings about their work, Herzberg found 2 factors that influence employee motivation and satisfaction…

1. Motivator factors – Simply put, these are factors that lead to satisfaction and motivate employees to work
harder. Examples might include enjoying your work, feeling recognised and career progression.
2. Hygiene factors – These factors can lead to dissatisfaction and a lack of motivation if they are absent. Examples
include salary, company policies, benefits, relationships with managers and co-workers.
According to Herzberg’s findings, while motivator and hygiene factors both influenced motivation, they appeared
to work completely independently of each other…
39

While motivator factors increased employee satisfaction and motivation, the absence of these factors didn’t
necessarily cause dissatisfaction. Likewise, the presence of hygiene factors didn’t appear to increase satisfaction
and motivation but their absence caused an increase in dissatisfaction.

How to apply it to the workplace


This theory implies that for the happiest and most productive workforce, you need to work on improving both
motivator and hygiene factors.
To help motivate your employees, make sure they feel appreciated and supported. Give plenty of feedback and
make sure your employees understand how they can grow and progress through the company.
To prevent job dissatisfaction, make sure that your employees feel that they are treated right by offering them the
best possible working conditions and fair pay. Make sure you pay attention to your team and form supportive
relationships with them.
Don’t forget that all of your employees are different and what motivates one person might not motivate
another. Paul Hebert of Symbolist believes that benefits packages should not be one-size-fits all… “For true
engagement to occur in a company you must first remove the issues that cause dissatisfaction – the baseline
benefits offered by the company that satisfy the hygiene needs of the employee. Then you must focus on the
individual and what they want out of their association with your enterprise”.

2. Maslow’s Hierarchy of Needs


The Hierarchy of Needs theory was coined by psychologist Abraham Maslow in his 1943 paper “A Theory of
Human Motivation”.
The crux of the theory is that individuals’ most basic needs must be met before they become motivated to achieve
higher level needs. The hierarchy is made up of 5 levels:
1. Physiological – these needs must be met in order for a person to survive, such as food, water and shelter.
2. Safety – including personal and financial security and health and wellbeing.
3. Love/belonging – the need for friendships, relationships and family.
4. Esteem – the need to feel confident and be respected by others.
5. Self-actualisation – the desire to achieve everything you possibly can and become the most that you can be.
According to the hierarchy of needs, you must be in good health, safe and secure with meaningful relationships
and confidence before you are able to be the most that you can be.

How to apply it to the workplace


Chip Conley, founder of the Joie de Vivre hotel chain and Head of Hospitality at Airbnb, used the Hierarchy of
Needs pyramid to transform his business. According to Chip, many managers struggle with the abstract concept of
40

self actualization and so focus on lower levels of the pyramid instead. Conley found one way of helping with higher
levels was to help his employees understand the meaning of their roles during a staff retreat…” In one exercise,
we got groups of eight housekeepers at a table and asked an abstract question: if someone from Mars came down
and saw what you were doing as a housekeeper in a hotel, what name would they call you? They came up with
“The Serenity Sisters,” “The Clutter Busters,” and “The Peace of Mind Police.” There was a sense that people were
doing more than just cleaning a room. They were creating a space for a traveler who was far away from home to
feel safe and protected”.

Conley’s team were able to realise the importance of their job to the company and to the people they were
helping. By showing them the value of their roles, the team were able to feel respected and motivated to work
harder. In order to get the most out of your team, you should also make sure you support them in other aspects of
their lives outside work. Perhaps you could offer flexible working hours to give employees time to focus on their
families and make sure they are paid fairly to help them feel financially stable.

3. Hawthorne Effect
The Hawthorne Effect was first described by Henry A. Landsberger in 1950 who noticed a tendency for some
people to work harder and perform better when they were being observed by researchers. The Hawthorne Effect
is named after a series of social experiments on the influence of physical conditions on productivity at Western
Electric’s factory at Hawthorne, Chicago in the 1920s and 30s.

The researchers changed a number of physical conditions over the course of the experiments including lighting,
working hours and breaks. In all cases, employee productivity increased when a change was made. The
researchers concluded that employees became motivated to work harder as a response to the attention being
paid to them, rather than the actual physical changes themselves.

How to apply it to the workplace


The Hawthorne Effect studies suggest that employees will work harder if they know they’re being observed. While
I don’t recommend hovering over your employees watching them all day, you could try providing regular
feedback, letting your team know that you know what they’re up to and how they’re doing.
Showing your employees that you care about them and their working conditions may also motivate them to work
harder. Encourage your team to give you feedback and suggestions about their workspace and development.

4. Expectancy Theory
Expectancy Theory proposes that people will choose how to behave depending on the outcomes they expect as a
result of their behaviour. In other words, we decide what to do based on what we expect the outcome to be. At
41

work, it might be that we work longer hours because we expect a pay rise. However, Expectancy Theory also
suggests that the process by which we decide our behaviours is also influenced by how likely we perceive those
rewards to be. In this instance, workers may be more likely to work harder if they had been promised a pay rise
(and thus perceived that outcome as very likely) than if they had only assumed they might get one (and perceived
the outcome as possible but not likely)

Expectancy Theory is based on THREE ELEMENTS:


1. Expectancy – the belief that your effort will result in your desired goal. This is based on your past experience,
your self confidence and how difficult you think the goal is to achieve.
2. Instrumentality – the belief that you will receive a reward if you meet performance expectations.
3. Valence – the value you place on the reward.
Therefore, according to Expectancy Theory, people are most motivated if they believe that they will receive a
desired reward if they hit an achievable target. They are least motivated if they don’t want the reward or they
don’t believe that their efforts will result in the reward.

How to apply it to the workplace


The key here is to set achievable goals for your employees and provide rewards that they actually want.
Rewards don’t have to come in the formof pay rises, bonuses or all-expenses paid nights out (although I find these
are usually welcomed!) Praise, opportunities for progression and “employee of the month” style rewards can all
go a long way in motivating your employees.

5. Three-Dimensional Theory of Attribution


Attribution Theory explains how we attach meaning to our own, and other people’s, behaviour. There are a
number of theories about attribution.

Bernard Weiner’s Three-Dimensional theory of attribution assumes that people try to determine why we do what
we do. According to Weiner, the reasons we attribute to our behaviour can influence how we behave in the
future. For example, a student who fails an exam could attribute their failure to a number of factors and it’s this
attribution that will affect their motivation in the future.

Weiner theorized that specific attributions (e.g. bad luck, not studying hard enough) were less important than the
characteristics of that attribution. According to Weiner, there are three main characteristics of attributions that
can affect future motivation.
42

1. Stability – how stable is the attribution? For example, if the student believes they failed the exam because they
weren’t smart enough, this is a stable factor. An unstable factor is less permanent, such as being ill. According to
Weiner, stable attributions for successful achievements, such as passing exams, can lead to positive expectations,
and thus higher motivation, for success in the future. However, in negative situations, such as failing the exam,
stable attributions can lead to lower expectations in the future.

2. Locus of control – was the event caused by an internal or an external factor?


For example, if the student believes it’s their own fault they failed the exam, because they are innately not smart
enough (an internal cause), they may be less motivated in the future. If they believed an external factor was to
blame, such as poor teaching, they may not experience such a drop in motivation.

3. Controllability – how controllable was the situation? If an individual believes they could have performed better,
they may be less motivated to try again in the future than someone who believes they failed because of factors
outside of their control.

How to apply it to the workplace


Weiner’s Three-Dimensional theory of attribution has implications for employee feedback.
Make sure you give your employees specific feedback, letting them know that you know they can
improve and how they can about it. This, in theory, will help prevent them from attributing their failure to an
innate lack of skill and see that success is controllable if they work harder or use different strategies.
You could also praise your employees for showing an improvement, even if the outcome was still not correct. For
example, you might praise someone for using the correct methodology even though the results weren’t what you
wanted. This way, you are encouraging employees to attribute the failure to controllable factors, which again, can
be improved upon in the future.
43

SESSION-6
JOBS EXERCISE

In the left-hand blanks, list up to five jobs that you have never held but that you believe might be
interesting to you. In the right-hand column, list up to three aspects of the job you think you would enjoy the
most.

EXAMPLE: YOUR TURN…


JOBS VALUES JOBS VALUES
computer work ORDER ____________________ ___________________

KNOWLEDGE ___________________
PATIENCE ___________________
astronaut THRILL ____________________ ___________________

RECOGNITION__ ___________________
SECURITY ___________________
artist CREATIVITY ____________________ ___________________

FREEDOM ___________________

BEAUTY ___________________

social worker HELPING ____________________ ___________________

PEOPLE ___________________
___________________
mail carrier OUTDOORS ____________________ ___________________
EXERCISE ___________________
SECURITY ___________________
44

HOBBIES EXERCISE

In the left-hand blanks, list up to five hobbies that you have or that you believe you would find
interesting. In the right-hand blanks, write the three things you personally find most interesting about each
hobby.

EXAMPLE: YOUR TURN…


HOBBIES VALUES HOBBIES VALUES
golf EXERCISE ________________ ________________

COMPETITION ________________

OUTDOORS ________________

hiking NATURE ________________ ________________

PEACE ________________

EXERCISE ________________

reading ALONE ________________ ________________

AMUSEMENT ________________
KNOWLEDGE ________________

ceramics ACCOMPLISH. ________________ ________________

RELAXATION ________________
PRIDE ________________

photography SKILL ________________ ________________

ENTERTAINMENT ________________
EXCITEMENT ________________
45

RECENT EVENTS EXERCISE

In the left-hand blanks, list up to five events that you attended by your own choice. In the right-hand blanks,
list three values you fulfilled by attending.

EXAMPLE: YOUR TURN…


EVENTS VALUES EVENTS VALUES
concert FRIENDSHIP ________________ ________________

RELAXATION ________________
MUSIC ________________
ball game EXCITEMENT ________________ ________________
FRIENDSHIP ________________
COMPETITION ________________
movie EXCITEMENT ________________ ________________
ADVENTURE ________________
INSIGHT ________________
dinner FAMILY ________________ ________________

LOVE ________________
RELAXATION ________________
cookout FAMILY ________________ ________________

PRIDE ________________

ACCOMPLISHMENT
46

SIGNIFICANT PEOPLE

In the left-hand column, name five persons who have had a significant, positive effect upon your life. In the
right-hand column, list up to three aspects or qualities you most appreciate about that person.

EXAMPLE: YOUR TURN…


PERSONS VALUES PERSONS VALUES
M. T. – teacher RESPECT

ACCEPTANCE ________________

SELF-CONFIDENCE ________________
Abraham Lincoln PATRIOTISM ________________

SERVICE ________________

CHALLENGE ________________
j. j. – spouse LOVE

BELONGING ________________

SUPPORTIVE ________________

R. T.–grandparent LOVE ________________

FAMILY ________________

EXCITEMENT

Huckleberry Finn CHALLENGE ________________ ________________


ADVENTURE ________________
INDEPENDENCE ________________
47

ORGANIZATIONS/CAUSES

In the left-hand column, list up to five organizations or causes in which you have enlisted voluntarily. After
each, list up to three of your underlying values that led you to become involved.

EXAMPLE: YOUR TURN…


CAUSES VALUES CAUSES VALUES
Scouts KNOWLEDGE ________________ ________________

LEADERSHIP

IDEALISM ________________
Red Cross HELPING

HEALTH ________________

GIVING ________________

AA SELF-GROWTH ________________ ________________

PEACE ________________

SPIRITUALITY ________________

Church SPIRITUALITY ________________ ________________

FAITH ________________
________________
$ to United Way HELPING ________________ ________________
________________
________________
48

A Shopping List of Values

The values that you surface in the above worksheets come from this list

acceptance excitement morality self-reliance


accomplishment exercise nature sensibility
achievement exploration neatness sentimentality
adventure fame nonconformity serenity
affection family observance seriousness
alertness femininity order service
ambition fitting in originality sex
amusement food outgoing sharing
animal life forgiveness outdoors sincerity
approval frankness patience skill
attention freedom patriotism sobriety
attractiveness freshness peace socialization
authenticity friendship people social welfare
beauty generosity perfection solitude
belonging genuineness persistence speed
boldness growth persuasiveness spirituality
calmness happiness physical ability sports
carefulness hard work physical exercise stability
caring health plant life status
caution helpfulness pleasantness strength
challenge history politeness surprise
cheerfulness honesty power talent
children humor practicality tangibility
cleverness idealism pride taste
communication imagination productivity thoughtfulness
companionship independence promptness thrift
competence inquisitiveness punctuality thrill
competition insight quiet tidiness
completion intelligence recognition tolerance
confidence intensity relationships Travel
49

conformity intuition relevance trust


contributing kindness relaxation truth
control knowledge reliability trustworthiness
cooperation leadership religion understanding
courtesy learning respect unselfishness
creativity life responsibility variety
curiosity logic risking vitality
dependability love roots warmth
discovery loyalty safety wisdom
education masculinity security wittiness
efficiency mastery self-assurance women
energy mechanics self-confidence work
entertainment men self-esteem
enthusiasm mental health self-expression
escape modesty self-growth

SESSION- 7

ASSERTIVENESS - AN INTRODUCTION

Assertiveness is a skill regularly referred to in social and communication skills training.


Being assertive means being able to stand up for your own or other people’s rights in a calm and positive way,
without being either aggressive, or passively accepting ‘wrong’.
Assertive individuals are able to get their point across without upsetting others, or becoming upset themselves.
Although everyone acts in passive and aggressive ways from time to time, such ways of responding often result from
a lack of self-confidence and are, therefore, inappropriate ways of interacting with others.
What is Assertiveness?
The Concise Oxford Dictionary defines assertiveness as: “Forthright, positive, insistence on the recognition of one's
rights”
In other words: Assertiveness means standing up for your personal rights - expressing thoughts, feelings and beliefs
in direct, honest and appropriate ways. It is important to note also that:
50

By being assertive we should always respect the thoughts, feelings and beliefs of other people.
Those who behave assertively always respect the thoughts, feelings and beliefs of other people as well as their own.
Assertiveness concerns being able to express feelings, wishes, wants and desires appropriately and is an important
personal and interpersonal skill. It enables individuals to act in their own best interests, to stand up for themselves
without undue anxiety, to express honest feelings comfortably and to express personal rights without denying the
rights of others.
Passive, Aggressive and Assertive
Assertiveness is often seen as the balance point between passive and aggressive behaviour, but it’s probably easier
to think of the three as points of a triangle.
Being Assertive
Being assertive involves taking into consideration your own and other people’s rights, wishes, wants, needs and
desires.
Assertiveness means encouraging others to be open and honest about their views, wishes and feelings, so that both
parties act appropriately.
Assertive behaviour includes:
Being open in expressing wishes, thoughts and feelings and encouraging others to do likewise.
Listening to the views of others and responding appropriately, whether in agreement with those views or not.
Accepting responsibilities and being able to delegate to others. See our page on Delegation Skills for more.
Regularly expressing appreciation of others for what they have done or are doing. Being able to admit to mistakes
and apologise.
Maintaining self-control.
Behaving as an equal to others.
Some people may struggle to behave assertively for a number of reasons, and find that they behave either
aggressively or passively instead.
Being Passive
Responding in a passive or non-assertive way tends to mean compliance with the wishes of others and can
undermine individual rights and self-confidence.
Many people adopt a passive response because they have a strong need to be liked by others. Such people do not
regard themselves as equals because they place greater weight on the rights, wishes and feelings of others. Being
passive results in failure to communicate thoughts or feelings and results in people doing things they really do not
want to do in the hope that they might please others. This also means that they allow others to take responsibility,
to lead and make decisions for them. A classic passive response is offered by those who say 'yes' to requests when
they actually want to say 'no'.
51

For example:
“Do you think you can find the time to wash the car today?”
A typical passive reply might be:
“Yes, I'll do it after I've done the shopping, made an important telephone call, finished the filing, cleaned the
windows and made lunch for the kids!”
A far more appropriate response would have been:
“No, I can't do it today as I've got lots of other things I need to do.”
The person responding passively really does not have the time, but their answer does not convey this message. The
second response is assertive as the person has considered the implications of the request in the light of the other
tasks they have to do.
Assertiveness is equally important at work as at home.
If you become known as a person who cannot say no, you will be loaded up with tasks by your colleagues and
managers, and you could even make yourself ill.
When you respond passively, you present yourself in a less positive light or put yourself down in some way. If you
constantly belittle yourself in this way, you will come to feel inferior to others. While the underlying causes of
passive behaviour are often poor self-confidence and self-esteem, in itself it can further reduce feelings of self-
worth, creating a vicious circle.
Being Aggressive
By being aggressive towards someone else, their rights and self-esteem are undermined.
Aggressive behaviour fails to consider the views or feelings of other individuals. Those behaving aggressively will
rarely show praise or appreciation of others and an aggressive response tends to put others down. Aggressive
responses encourage the other person to respond in a non-assertive way, either aggressively or passively.
There is a wide range of aggressive behaviours, including rushing someone unnecessarily, telling rather than asking,
ignoring someone, or not considering another's feelings.
Good interpersonal skills mean you need to be aware of the different ways of communicating and the different
response each approach might provoke. The use of either passive or aggressive behaviour in interpersonal
relationships can have undesirable consequences for those you are communicating with and it may well hinder
positive moves forward.
It can be a frightening or distressing experience to be spoken to aggressively and the receiver can be left wondering
what instigated such behaviour or what he or she has done to deserve the aggression.
If thoughts and feelings are not stated clearly, this can lead to individuals manipulating others into meeting their
wishes and desires. Manipulation can be seen as a covert form of aggression whilst humour can also be used
aggressively.
52

Different Situations Call for Different Measures – or do they?


One may find that one responds differently — whether passively, assertively or aggressively — when one are
communicating in different situations.
It is important to remember that any interaction is always a two-way process and therefore your reactions may
differ, depending upon your relationship with the other person in the communication.
For example It may be easier to be assertive to one’s partner than with one’s boss or vice versa. However, whether
it is easy or not, an assertive response is always going to be better for one’s relationship with the other person.

GENERAL TECHNIQUES OF ASSERTIVENESS


Two key techniques that can aid assertiveness are known as "Fogging" and the "Stuck Record" technique.
Fogging
Fogging is a useful technique if people are behaving in a manipulative or aggressive way.
Rather than arguing back, fogging aims to give a minimal, calm response using terms that are placating but not
defensive, while at the same time not agreeing to meet demands.
Fogging involves agreeing with any truth that may be contained within statements, even if critical. By not responding
in the expected way, in other words by being defensive or argumentative, the other person will cease confrontation
as the desired effect is not being achieved. When the atmosphere is less heated, it will be possible to discuss the
issues more reasonably.
Fogging is so termed because the individual acts like a 'wall of fog' into which arguments are thrown, but not
returned.
Example Situation
“What time do you call this? You're nearly half an hour late, I'm fed up with you letting me down all the time.”
Fogging response:
“Yes, I am later than I hoped to be and I can see this has annoyed you.”
“Annoyed? Of course I'm annoyed, I've been waiting for ages. You really should try to think about other people a bit
more.”
Fogging response:
“Yes, I was concerned that you would be left waiting for almost half an hour.”
“Well... why were you late?”
The Stuck Record Technique
The Stuck Record technique employs the key assertive skill of 'calm persistence'.
53

It involves repeating what you want, time and time again, without raising the tone of your voice, becoming angry,
irritated, or involved in side issues.
Example Situation
Imagine that you are returning something that is faulty to a store. The conversation may go as follows.
“I bought these shoes last week and the heels have fallen off. I would like a refund please.”
“It looks like they've been worn a lot and these shoes were only designed for occasional wear.”
Stuck Record technique response:
“I have only had them a week and they are faulty. I would like a refund please.”
“You cannot expect me to give you your money back after you've worn them out.”
Stuck Record technique response:
“The heels have fallen off after only a week and I would like a refund please.”
... and so on.
Continually repeating a request will ensure the discussion does not become side-tracked and involved in irrelevant
argument. The key is to stay calm, be very clear in what you want, stick to the point and not give up.
Accept a compromise only if you are happy with the outcome.
Positive and Negative Enquiry
Positive Enquiry
Positive enquiry is a simple technique for handling positive comments such as praise and compliments.
People often struggle with responding to praise and compliments, especially those with lower self-esteem as they
may feel inadequate or that the positive comments are not justified. It is important to give positive feedback to
others when appropriate but also to react appropriately to positive feedback that you receive.
Positive enquiry is used to find out more details about the compliment or praise given, and agree with it:
Example Situation
Sender:
“You made an excellent meal tonight, it was delicious!”
Receiver:
“Thanks. Yes, it was good. What did you like about it in particular?”
This is different from a passive response that may have been:
"It was no effort" or "It was just a standard recipe"
Negative Enquiry
The opposite of positive enquiry is negative enquiry. Negative enquiry is a way to respond to more negative
exchanges such as receiving criticism.
54

Dealing with criticism can be difficult, remember that any criticism received is just somebody's opinionNegative
enquiry is used to find out more about critical comments and is a good alternative to more aggressive or angry
responses to criticism.
Example Situation
Sender:
“That meal was practically inedible, I can't remember the last time I ate something so awful”
Receiver:
“It wasn't the best, exactly what didn’t you like about it?”
This is different from an aggressive response that may have been:
"How dare you, I spent all afternoon preparing that meal" or "Well that's the last time I cook for you"

THE BENEFITS OF BEING ASSERTIVE


One of the main benefits of being assertive is that it can help you to become more self-confident, as you gain a
better understanding of who you are and the value that you offer.
Assertiveness provides several other benefits that can help you both in your workplace and in other areas of your
life. In general, assertive people:

Make great managers. They get things done by treating people with fairness and respect, and are treated by
others the same way in return. This means that they are often well-liked and seen as leaders that
people want to work with.

Negotiate successful "win-win" solutions. They are able to recognize the value of their opponent's position and
can quickly find common ground with him.

Are better doers and problem solvers. They feel empowered to do whatever it takes to find the best solution to
the problems that they encounter.

Are less anxious and stressed. They are self-assured and don't feel threatened or victimized when things don't
go as planned or as expected.
55

COMPETENCY-2
SYNTACTIC RELATIONS
Session -8 & 9

Tricks for Reading Comprehension passages

Preparing for Reading Comprehension is intimidating, especially the long RC passages and it is natural to prefer short
passages. One has to concentrate on long passages and become quite adept at handling them.

Here are a few tricks to help in better preparation for reading passages.

1. Do an initial reading and take notes

Short passages are structured into either one or two paragraphs. If they are one paragraph, you can often divide the
passage by searching for structural words. First, do an initial reading to map the text out. The idea in a one-
paragraph text is to read the first couple of sentences and take notes.

2. Look for structural words and other text markers

Next, you should start skimming through the rest and paying attention to text markers, such as the structural words.
For example, a structural word used could be “however, or “yet”. Focusing on such words will help you to
understand the overall structure. In addition, you should also pay attention to the very end, as it may provide a
summary, information about the author’s opinion, or a conclusion.

3. A final tip for two-paragraph short passages

The same procedure should be used with a two-paragraph text. In addition to what you have done with the first
paragraph, look closely at the first two sentences of the second paragraph. You should extend this to the third
sentence as well if there is an opposition conjunction. Of course, pay close attention to the end of the second
paragraph, as well.

This concept of active reading is very important in all RC passages. Think about the structure of the text. But don’t
forget to do so when handling the short passages as well.

Eliminate extreme answers in Reading Comprehension

Today we will discuss a particular type of incorrect answer choice: the extreme answer. Extreme answer choices can
be found in several types of Verbal questions, including tone, main idea and inference questions. Here’s a hint -- you
can almost never go wrong if you eliminate an extreme answer choice.

Students may come across extreme answer choices in Reading Comprehension (RC) questions about tone. The
question is probably a tone question if it includes a phrase like “the author’s attitude towards ‘X’ can best be
described as…”
56

In RC passages, the tone is usually quite dry, similar to that of an academic journal. The authors tend to be more
neutral than very excited about a topic. He or she will rarely hate or love something and will never rave about or
deeply detest an idea. In other words, the less extreme, the better.

Here’s the take-home idea: if you need to quickly eliminate answer choices in a tone question, find the ones that
contain words that seem extreme to you. Consider what words an academic author would or would not use in his or
her writing. You can probably eliminate any extreme answer choice.

Strategies for Reading Comprehension

The good news is that most passages in the Reading Comprehension section are somewhat formulaic. Typically, the
writer uses the first paragraph to establish the topic they’ll be discussing in the entire passage. The second
paragraph is used to question someone else’s stance on that topic. Finally, the writer uses the third paragraph to
establish his or her own stance on that topic.

However, after reading the entire passage, ask yourself a few questions when reading the passages. Firstly,
determine what the issue at hand is. You should be able to narrow this down a little more quickly. Also consider
which theories the author discusses within that issue, and whether or not he or she agrees with them. Many
questions in the Reading Comprehension section ask whether or not the author agrees or disagrees with a
statement. Nailing down how the author feels about the issues will help you answer these more quickly and
accurately.

Also, don’t be tempted to read over a sentence that doesn’t click right away. Doing so multiple times will only take
you out of the flow of reading and digesting the entire passage. Even worse, you’ll be inclined to stop asking yourself
the questions we discussed above, which will make it even more difficult to answer the following questions. In this
case, re-reading will only be a detriment to you.

Directions

The questions in this group are based on the content of a passage. After reading the passage, choose the best
answer to each question. Answer all questions following the passage on the basis of what is stated or implied in the
passage.

Question

Schools expect textbooks to be a valuable source of information for students. My research suggests, however, that
textbooks that address the place of Native Americans within the history of the United States distort history to suit a
particular cultural value system. In some textbooks, for example, settlers are pictured as more humane, complex,
skillful, and wise than Native Americans. In essence, textbooks stereotype and depreciate the numerous Native
American cultures while reinforcing the attitude that the European conquest of the New World denotes the
superiority of European cultures. Although textbooks evaluate Native American architecture, political systems, and
homemaking, I contend that they do it from an ethnocentric, European perspective without recognizing that other
perspectives are possible.

One argument against my contention asserts that, by nature, textbooks are culturally biased and that I am simply
underestimating children's ability to see through these biases. Some researchers even claim that by the time
students are in high school, they know they cannot take textbooks literally. Yet substantial evidence exists to the
57

contrary. Two researchers, for example, have conducted studies that suggest that children's attitudes about
particular cultures are strongly influenced by the textbooks used in schools. Given this, an ongoing, careful review of
how school textbooks depict Native Americans is certainly warranted.

Which of the following would most logically be the topic of the paragraph immediately following the passage?

(A) specific ways to evaluate the biases of United States history textbooks
(B) the centrality of the teacher's role in United States history courses
(C) nontraditional methods of teaching United States history
(D) the contributions of European immigrants to the development of the United States
(E) ways in which parents influence children's political attitudes

Exercises
Practice -1

Passage for Question 1 to 9


"Most economists in the United states seem captivated by spell of the free market. Consequently, nothing seems
good ornormal that does not accord with the requirements of the free market. A price that is determined by the
seller or for that matter, established by anyone other than the aggregate of consumers seems pernicious,
Accordingly, it requires a major act of will to think of price – fixing (the determination of prices by the seller) as both
“normal” and having a valuable economic function. In fact, price-fixing is normal in all industrialized societies
because the industrial system itself provides, as an effortless consequence of its own development, the price-fixing
that requires, Modern industrial planning requires and rewards great size. Hence a comparatively small number of
large firms will be competing for the same group of consumers. That each large firm will act with consideration of its
own needs and thus avoid selling its products for more than its competitors charge is commonly recognized by
advocates of free-markets economic theories. But each large firms will also act with full consideration of the needs
that it has in common with the other large firms competing for the same customers. Each large firm will thus avoid
significant price cutting, because price cutting would be prejudicial to the common interest in a stable demand for
products. Most economists do not see price-fixing when it occurs because they expect it to be brought about by a
number of explicit agreements among large firms; it is not. More over those economists who argue that allowing the
free market to operate without interference is the most efficient method of establishing prices have not considered
the economies of non socialist countries other than the United States. These economies employ intentional price-
fixing usually in an overt fashion. Formal price fixing by cartel and informal price fixing by agreements covering the
members of an industry are common place. Were there something peculiarly efficient about the free market and
inefficient about price fixing, the countries that have avoided the first and used the second would have suffered
drastically in their economic development. There is no indication that they have. Socialist industry also works within
a frame work of controlled prices. In early 1970’s, the soviet union began to give firms and industries some of the
flexibility in adjusting prices that a more informal evolution has accorded the capitalist system. Economists in the
United States have hailed the change as a return to the free market.But Soviet firms are no more subject to prices
established by free market over which they exercise little influenced than are capitalist firms.

Question 1
The primary purpose of the passage is to
1. refute the theory that the free market plays a useful role in the development of industrialized societies.
2. suggest methods by which economist and members of the government of the United States can recognize and
combat price-fixing by large firms.
3. explain the various ways in which industrialized societies can fix in order to stabilized the free market
58

4. argue that price-fixing, in one form or another, is an inevitable part of and benefit to the economy of any
industrialized society.
5. Analysis of free markets in different economies

Question 2
The passage provides information that would answer which of the following questions about price-fixing?

I. What are some of the ways in which prices can be fixed?


II. For what products is price-fixing likely to be more profitable than the operation of the free market?
III. Is price-fixing more common in socialist industrialized societies or in nonsocialist industrialized societies?

1. I only2. III only 3. I and II only 4. II and III only 5. I, II and III

Question 3
The author’s attitude toward “Most economists in the United States” can best be described as
1. spiteful and envious 2. scornful and denunciatory
3. critical and condescending 4. ambivalent but deferential e. uncertain but interested

Question 4
It can be inferred from the author’s argument that a price fixed by the seller “seems pernicious” because
1. people do not have confidence in large firms
2. people do not expect the government to regulate prices
3. most economists believe that consumers as a group should determine prices.
4. most economists associate fixed prices with communist and socialist economies.
5. Most economists believe that no one group should determine prices.

Question 5
The suggestion in the passage that price-fixing in industrialized societies is normal arises from the author’s
statement that price-fixing is
1. a profitable result of economic development
2. an inevitable result of the industrial system
3. the result of a number of carefully organized decisions.
4. a phenomenon common to industrialized and to industrialized societies.
5. a phenomenon best achieved cooperatively by government and industry.

Question 6
According to the author, priced-fixing in nonsocialist countries is often.
1. accidental but productive 2. illegal but useful
3. legal and innovative 4. traditional and rigid
5. intentional and widespread.

Question 7
According to the author, what is the result of the Soviet Union’s change in economic policy in the 1970’s?
1. Soviet firms show greater profit 2. Soviet firms have less control over the free market
3. Soviet firms are able to abject to technological advances.
59

4. Soviet firms have some authority to fix prices. 5. Soviet firms are more responsive to the free market.

Question 8
8. With which of the following statements regarding the behavior of large firms in industrialized societies would
the author be most likely to agree.
1. The directors of large firms will continue to anticipate the demand for products
2. The directors of large firms are less interested in achieving a predictable level of profit tan in achieving a large
profit.
3. The directors of large firms will strive to reduce the costs of their products.
4. Many directors of large firms believe that the government should establish the prices that will be charged for
products.
5. Many directors of large firms believe that the price charged for products is likely to increase annually.

Question 9
In the passage, the author is primarily concerned with
1. predicting the consequences of a practice 2. criticizing a point of view
3. calling attention to recent discoveries. 4. proposing a topic for research.
5. summarizing conflicting opinions.

Passage for Question 10 to 15


The discoveries of the white dwarf, the neutron star, and the black hole, coming well after the discovery of the red
giant are among eh most exciting developments in decades because they may be well present physicists with their
greatest challenge since thefailure of classical mechanics. In the life cycle of the star, after all of the hydrogen and
helium fuel has been burned, the delicate balance between the outer nuclear radiation.pressure and the stable
gravitational force becomes disturbed and slow contraction begins. As compression increases, a very dense plasma
forms. If the initial star had mass of less than 1.4 solar masses (1.4 times the mass of our sun), the process ceases at
the density of 1,000 tons per cubic inch, and the star becomes the white dwarf. However, if the star was originally
more massive, the white dwarf plasma can’t resist the gravitations pressures, and in rapid collapse, all nuclei of lthe
star are converted to a gas of free neutrons. Gravitational attraction compresses this neutron gas rapidly until a
density of 10 tons per cubic inch is reached; at this point the strong nuclear force resists further contraction. If the
mass of the star was between 1.4 and a few solar masses, the process stops here, and we have a neutron star. But if
the original star was more massive than a few solar masses, even the strong nuclear forces cannot resist the
gravitational orunch. The neutrons are forced into one another to form heavier hadrons and these in turn coalesce
to form heavier entities, of which we as yet know nothing. At this point, a complete collapse of the stellar mass
occurs; existing theories predict a collapse to infinite density and infinitely small dimensions Well before this,
however, the surface gravitational force would become so strong that no signal could ever leave the star - any
photon emitted would fall back under gravitational attraction – and the star would become black hole in space. This
gravitational collapse poses a fundamental challenge to physics. When the most widely accepted theories predict
such improbable things as infinite density and infinitely small dimensions, it simply means that we are missing some
vital insight. This last happened in physics in the 1930’s, when we faced the fundamental paradox concerning atomic
structure. At that time, it was recognized that electrons moved in table orbits about nuclei in atoms. However, it was
also recognized that if charge is accelerated, as it must be to remain in orbit, it radiates energy; so, theoretically, the
electron would be expected eventually to spiral into the nucleus and destroy the atom. Studies centered around this
paradox led to the development of quantum mechanics. It may well be that an equivalent t advance awaits us in
investigating the theoretical problems presented by the phenomenon of gravitational collapse.
60

Question 10
The primary purpose of the passage is to
1. offer new explanations for the collapse of stars.
2. explain the origins of black holes, neutron stars, and white dwarfs.
3. compare the structure of atoms with the structure of the solar system.
4. explain how the collapse of stars challenges accepted theories of physics.
5. describe the imbalance between radiation pressure and gravitational force.

Question 11
According to the passage, in the final stages of its devedlopment our own sun is likely to take the form of a
1. white dwarf 2. neutron star 3. red giant 4. gas of free neutrons 5. black hole

Question 12
According to the passage, an imbalance arises between nuclear radiation pressure and gravitational force in stars
because
1. the density of a star increases as it ages. 2. radiation pressure increases as a star increases in mass
3. radiation pressure decreases when a star’s fuel has been consumed
4. the collapse of a star increases its gravitational force.
5. a dense plasma decreases the star’s gravitational force.

Question 13
The author asserts that the discoveries of the white dwarf, the neutron star, and the black hole are significant
because these discoveries.
1. demonstrate the probability of infinite density and infinitely small dimensions
2. pose the most comprehensive and fundamental problem faced by physicists in decades
3. clarify the paradox suggested by the collapse of electrons into atomic nuclei.
4. establish the relationship between the mass and gravitational pressure.
5. assist in establishing the age of the universe by tracing the life histories of stars.

Question 14
The passage contains information that answers which of the following questions?
1. I only 2. III only 3. I and II only 4. II and III only 5. I, II and III

Question 15
The author introduces the discussion of the paradox concerning atomic structures in order to
1. Show why it was necessary to develop quantum mechanics
2. Compare the structure of an atom with the structure of star
3. Demonstrate by analogy that a vital insight in astrophysics is missing
4. Illustrate the contention that improbable things do happen in astrophysics
5. Argue that atoms can collapse if their electrons do not remain in orbit.
61

Practice -2
Passage for Question 1 to 9
Recent years have brought minority-owned businesses in the United States unprecedented opportunities-as well as
new and significant risks. Civil rights activists have long argued that one of the principal reasons why Blacks,
Hispanics and the other minority groups have difficulty establishing themselves in business is that they lack access to
the sizable orders and subcontracts that are generated by large companies. Now congress, in apparent agreement,
has required by law that businesses awarded federal contracts of more than $500,000 do their best to find minority
subcontractors and record their efforts to do so on forms field with the government. Indeed, some federal and local
agencies have gone so far as to set specific percentage goals for apportioning parts of public works contracts to
minority enterprises. Corporate response appears to have been substantial. Accoring to figures collected in 1977,
the total of corporate contracts with minority business rose from $77 to $1. 1 billion in 1977. The projected total of
corporate contracts with minority business for the early 1980’s is estimated to be over $3 billion per year with no
letup anticipated in the next decade. Promising as it is for minority businesses, this increased patronage poses
dangers for them, too. First, minority firms risk expanding too fast and overextending themselves financially, since
most are small concerns and, unlike large businesses they often need to make substantial investments in new plants,
staff, equipment, and the like in order to perform work subcontracted to them. If, thereafter, their subcontracts are
for some reason reduced, such firms can face potentially crippling fixed expenses. The world of corporate
purchasing can be frustrating for small entrepreneurs who get requests for elaborate formal estimates and bids.
Both consume valuable time and resources and a small company’s efforts must soon result in orders, or both the
morale and the financial health of the business will suffer. A second risk is that White-owned companies may-seek to
cash inon the increasing apportion-ments through formation of joint ventures with minority-owned concerns, of
course, in many instances there are legitimate reasons for joint ventures; clearly, white and minority enterprises can
team up to acquire business that neither could Third, a minority enterprise that secures the business of one large
corporate customer often runs the danger of becoming – and remaining dependent. Even in the best of
circumstances, fierce competition from larger, more established companies makes it difficult for small concerns to
broaden their customer bases; when such firms have nearly guaranteed orders from a single corporate benefactor,
they may truly have to struggle against complacency arising from their current success.
Question 1
The primary purpose of the passage is to
1. present a commonplace idea and its inaccuracies 2. describe a situation and its potential drawbacks
3. propose a temporary solution to a problem 4. analyze a frequent source to a problem
5. explore the implications of a findings.

Question 2
The passage supplies information that would answer which of the following questions?
1. What federal agencies have set percentage goals for the use of minority owned businesses in public works
contracts?
62

2. To which governments agencies must businesses awarded federal contracts report their efforts to find minority
subcontractors?
3. How widespread is the use of minority-owned concerns as “fronts; by White backers seeking to obtain
subcontracts?
4. How many more minority owned businesses were there in 1977 than in 1972?
5. What is one set of conditions under which a small business might find itself financially overextended?

Question 3
According to the passage, civil rights activists maintain that one disadvantage under which minority owned
businesses have traditionally had to labor is that they have
1. been specially vulnerable to governmental
2. been denied bank loans at rates comparable to those afforded larger competitors
3. not had sufficient opportunity to secure businesses created by large corporations
4. not been able to advertise in those media that reach large numbers of potential customers
5. not had adequate representation in the centers of government power.

Question 4
The passage suggests that the failure of a large business to have its bids for subcontracts results quickly in order
might cause it to
1. experience frustrations but not serious financial harm2. face potentially crippling fixed expenses
3. have to record its efforts on forms filed with the government
4. increase its spending with minority subcontractors
5. revise its procedure for making bids for federal contracts and subcontracts

Question 5
The authors implied that the minority owned concern that does the greater part of its business with one large
corporate customer should
1. avoid competition with the larger, more established concerns by not expanding
2. concentrate on securing even more business from that corporation
3. try to expands its customers base to avoid becoming dependent on the corporation
4. pass on some of the work to be done for the corporation to other minority owned concerns.
5. use its influence with the other corporation to promote subcontracting with other minority concerns.

Question 6
It can be inferred from the passage that, compared with the requirements of law, the percentage goals set by
“some federal and local agencies” are
1. more popular with large corporations2. more specific
3. less controversial 4. less expensive to enforce 5. easier to comply with

Question 7
Which of the following if true, would most weaken the author’s assertion that, in 1970’s, corporate response to
federal requirements (lines 18-19) was substantial?
1. Corporate contracts with minority owned business totaled about $2 billion in 1979
2. Between 1970 and 1972, corporate contracts with minority owned businesses declined by 25 percent
3. The figures collected 1977 underrepreented the extent of corporate contracts with minority owned businesses.
63

4. The estimate of corporate spending with minority owned businesses in 1980 is approximately $10 million too
high.
5. The $1.1 billion represented the same percentage of total corporate spending in 1977 as did $77 million in 1972.

Question 8
The passage most likely appeared in
1. a business magazine 2. an encyclopedia of black history to 1945
3. a dictionary of financial terms 4. a yearbook of business statistics 5. an accounting textbook

Question 9
The author would most likely agree with which of the following statements about corporate response to working
with minority subcontractors?
1. Annoyed by the proliferations of “front” organizations, corporates are likely to reduce their efforts to work with
minority owned subcontractors in the near future.
2. Although corporations showed considerable interest in working with minority businesses in the 1970’s their
aversion to government paperwork made them reluctant to pursue many government contracts.
3. The significant response of corporation in the 1970’s is likely to be sustained and conceivably be increased
throughout the 1980’s
4. Although corporations re eager to co-operate with minority owned businesses, a shortage of capital in the
1970’s made substantial response impossible.
5. The enormous corporate response has all but eliminated the dangers of over expansion that used to plague
small minority owned businesses.

Passage for Question 10 to 15


In strongly territorial birds such as the indigo bunting, song is the main mechanism for securing g, defining, and
defending an adequate breeding are. When population density is high, only the strongest males can retain a suitable
area. The weakest males do not breed or are forced to nest on poor or marginal territories. During the breeding
season, the male indigo bunting sings in his territory; each song lasts two or three seconds with a very short pause
between songs, Melodic and rhythmic characteristics are produced by rapid changes in sound frequency and some
regularity of silent periods between sounds. These modulated sounds form recognizable units, called figures, each of
which is reproduced again and again with remarkable consistency. Despite the large frequency range of these
sounds and the rapid frequency changes that the birds makes, the n umber of figures is very limited. Further,
although we found some unique figures in different geographical populations, more than 90 percent of all Indigo
bunting figures are extremely stable on the geographic basis . In our studies of isolated buntings we found that male
indigo buntings are capable of singing many more types of figures than they usually do. Thus, it would seem that
they copy their figures from other buntings they hear signing. Realizing that the ability to distinguish the songs of
one species from those of another could be an important factor in the volition of the figures, we tested species
recognition of a song. When we played a tape recording of a lazuli bunting or a painted bunting, male indigo bunting
did not respond; Even when a dummy of male indigo bunting was placed near the tape recorder. Playing an indigo
bunting song, however, usually brought an immediate response, making it clear that a male indigo bunting can
readily distinguished songs of its own species from those of other species. The role of the songs figures in
interspecies recognition was then examined. We created experimental songs composed of new figures by playing a
normal song backwards, which changed the detailed forms of the figures without altering frequency ranges or gross
temporal features. Since the male indigos gave almost a full response to the backward song, we concluded that a
wide range of figures shapes can evoke positive responses. It seems likely, therefore, that a specific configuration is
not essential for interspecies recognition, but it is clear that song figures must confirm to a particular frequency
64

range, must be within narrow limits of duration, and must be spaced at particular intervals. There is evident that
new figures may arise within a population through a slow process of change and selection. This variety is probably a
valuable adaptation for survival: if every bird sang only a few types of figures, in dense woods or underbrush a
female might have difficulty recognizing her mate’s song and a male might not be able to distinguish a neighbor
from a stranger. Our studies led us to conclude that there must be a balance between song stability and
conservatism, which lead to clear-cut species recognition, and song variation, which leads to individual recognition.

Question 10
The primary purpose of passage is to
1. raise new issues 2. explain an enigma 3. refute misconceptions
4. reconcile differing theories 5. analyze a phenomenon

Question 11
According to the passage, which of the following is true about the number and general nature of figures sung by
the indigo bunting?
1. They are established at birth 2. They evolve slowly as the bird learns
3. They are learned from other indigo buntings.
2. They develop after the bird has been forced onto marginal breeding areas.
3. The gradually develop through contact with prospective mates

Question 12
It can be inferred that the investigation that determined the similarly among more than 90 percent of all the
figures produced by birds living in different regions was undertaken to answer which of the following questions?
I.How much variations, if any, is there in the figure types produced by indigo buntings in different locales?
II.Do local populations of indigo buntings develop their own dialects of figure types?
III.Do figure similarities among indigo buntings decline with increasing geographic
separation?
1. II only 2. III only 3. I and II only 4. II and III only 5. I, II and III

Question 13
It can be inferred from the passage that the existence of only a limited number of indigo bunting figures servers
primarily to
1. ensure species survival by increasing competition among the fittest males for the females
2. increase population density by eliminating ambiguity in the figures to which the females must respond
3. maintain the integrity of the species by restricting the degree of figure variation and change
4. enhance species recognition by decreasing the number of figure patterns to which the bird must respond
5. avoid confusion between species by clearly demarcating the figure patterns of each specials

Question 14
It can be inferred that a dummy of a male indigo bunting was placed near the tape recorder that played the songs
of different species in order to try to
1. simulate the conditions in nature.
2. Rule out visual cues as a factor in species recognition
3. Supply an additional clue to species recognition for the indigo bunting
4. Provide data on the habits of bunting species other than then indigo bunting
65

5. Confound the indigo buntings in the experiment

Question 15
According to the passage, the authors played a normal indigo bunting song backwards in order to determine
which of the following?
1. What are the limits of the frequency range that will provide recognition by the indigo bunting.
2. What is the time duration necessary for recognition by the indigo bunting?
3. How specific must a figure shape be for it to be recognized by the indigo bunting?
4. How does variation in the pacing of song figures?
5. Is the indigo bunting responding to cues other than those in the song figures?

SESSION – 10
ANALYSIS OF AN ISSUE
Guidelines
1. One is expected to explain one’s position on an issue.
2. One must state and justify one’s opinion of the topic under discussion.
3. Every issue topics has two sides.
4. There is no "right" side: One has to decide one’s position on the topic after consideration of the pros and
cons.
5. Generally the position is 80 or 90% in favor of one side.
6. Always spend about 5 minutes thinking and planning. (Draw up a table of points before deciding which side
will make the most persuasive essay.)
7. Always use specific examples to support your point of view.

ESSAY FORMAT
Part I – Introduction
Write an introduction explaining in your own words what the issue is about. Try to generate interest in the topic
under discussion, and make it clear why the topic is controversial. End your paragraph with a thesis statement. (A
thesis statement is a clear summation of your point of view.)
Part II - The body of the essay
Write 2-3 paragraphs to support your thesis. Each paragraph should introduce one point. Explain the point and give
a specific example wherever possible. You can also give reasons why the point is important or relevant. Be sure to
give connecting words and phrases (links) at the beginning of each paragraph to give a sense of logical flow.
Part III – Qualification
Since the issue is never entirely black or white, you do not want to sound too dogmatic, and so you qualify
(moderate) your position (i.e. you usually explain that under certain circumstances the other side of the issue might
be correct). This may involve a sentence beginning with "but" or "however"...
66

Part IV – Conclusion
You cannot leave the essay without reinforcing your thesis. If you have introduced a qualification into your
argument, you will need to draw the essay back to your thesis. Try to avoid simply repeating what you have said;
find something general to say that makes it clear that you have finished.
More essay writing tips
The introduction and the conclusion can be very general, but the body of the essay must be specific. Do not give a
long list of examples all illustrating the same point. Stick to the one point-one example method
The examples can be from your own experience or from your reading or knowledge of current affairs, history etc.
Good vocabulary is an asset, but don’t use long words if you are not sure of the meaning

SESSION -11

ANALYSIS OF AN ISSUE: AN INTRODUCTION


In this Analysis of An Issue section, an issue will be presented and you will be asked to analyze it and explain your
views on it together with examples from your own experience, observations or reading.

These ISSUES themselves fall into two categories

(i) One in which two opposite views on an issue are presented and you are asked to take a position in favour of one
of them and defend that position.

(ii) One in which only one view is presented and you are asked whether you agree or disagree with it and called
upon to defend your position.

How to start: :

Do not start pounding on the keyboard of the computer immediately on seeing the question.

Take a few minutes to think about the issue and plan a response before you begin writing. Be sure to organize your
ideas and develop themfully, but leave time to reread your response and make any revisions that you think are
necessary.

The thirty minutes you are allowed to write the essay is not a long time and no one is capable of writing a profound,
thoroughly developed, well-crafted and technically perfect essay in just half an hour. And your essay need not have
all these qualities to earn you a high score of 5, 5.5 or 6.

The opinion in an Analysis of An Issue essay is stated in such broad and general terms that almost anyone can find
something to say either in favour or against it.

The first step in developing your essay is to brainstorm ideas about what to say. There is no pre-set or magic formula
for this process.
You should first take about two minutes to understand the issue and think of points both in favour and against what
is stated. Then jot down on a scrap sheet (which will be given to you at the test centre) these points on either side
of a centre line.

You will probably find that you have more points on one side than on the other. Adopt the side on which you have
more points as your opinion.
67

Remember that there is no right or wrong opinion which decides the score that you will be awarded. What’s needed
is that you should write a cogent and logical essay supporting your opinion.
Then look at your notes and pick the three or four ideas you like best. These should be ideas which you think make
sense, relate to the topic and support your point of view reasonably well.

Put a check mark next to those ideas so that you spot them easily when you start composing your essay. Finally,
decide on the sequence for the ideas. This can be done in several ways. Often the sequence will be obvious. One
idea may be the fountainhead of all others and therefore should clearly come first. Sometimes the ideas may all
reflect historic events that took place in a definite time sequence. If there is such an obvious sequence, use it.

The most emphatic parts of any essay the parts that the reader is most likely to remember or be impressed with -
are the beginning and the end.

How an Issue should be Analysed: :

Begin the essay with a brief introductory paragraph that sets forth your point of view clearly and, if you like, also
suggests the nature of the ideas you will be using to defend it. The side you are proposing to take on the issue
should be brought out clearly in the first paragraph itself.

Have three or four middle paragraphs to substantiate your view with examples or illustrations. End the essay with a
brief concluding paragraph which summarizes your point of view in a clear, concise and forceful way.

The last step in the process is a quick revision of your essay to eliminate inadvertent errors in spelling and grammar
that you might have committed.

Evaluation of Response: College and university faculty members from various subject matter areas, including
management education, will evaluate the overall quality of your thinking and writing. They will consider how well
you—
ganize, develop, and express your ideas about the issue presented

IntroductiontotheAnalyzingArgument
The "Analyze an Argument" task assesses your ability to understand, analyze and evaluate arguments according to
specific instructions and to convey your evaluation clearly in your writing. Each topic consists of a brief passage in
which the author makes a case for some course of action or interpretation of events by presenting claims backed by
reasons and evidence.
The task is to discuss the logical soundness of the author's case by critically examining the line of reasoning and the
use of evidence. This task requires you to read the argument and instructions carefully. The argument might be read
more than once and make brief notes about points before developing it into a response. While reading the
argument, special attention should be paid to:
 what is offered as evidence, support or proof
 what is explicitly stated, claimed or concluded
 what is assumed or supposed, perhaps without justification or proof
68

 what is not stated, but necessarily follows from or underlies what is stated
In addition, the structure of the argument — the way in which these elements are linked together to form a line of
reasoning; should be considered, i.e., In tracing this line, look for transition words and phrases that suggest the
author is attempting to make a logical connection (e.g., however, thus, therefore, evidently, hence, in conclusion).
An important part of performing well on the Argument task is remembering what is not being asked:
 to discuss whether the statements in the argument are true or accurate.
 to agree or disagree with the position stated.
 to express your own views on the subject being discussed.
Instead, the task taker should evaluate the logical soundness of an argument of another writer and, in doing so, to
demonstrate the critical thinking, perceptive reading and analytical writing skills that university faculty consider
important for success in graduate school.

It is important to address the argument according to the specific instructions. Each task is accompanied by one of
the following sets of instructions:
 Write a response in which you discuss what specific evidence is needed to evaluate the argument and
explain how the evidence would weaken or strengthen the argument.
 Write a response in which you examine the stated and/or unstated assumptions of the argument. Be sure to
explain how the argument depends on these assumptions, and what the implications are for the argument if
the assumptions prove unwarranted.
 Write a response in which you discuss what questions would need to be answered in order to decide
whether the recommendation and the argument on which it is based are reasonable. Be sure to explain how
the answers to these questions would help to evaluate the recommendation.
 Write a response in which you discuss what questions would need to be answered in order to decide
whether the advice and the argument on which it is based are reasonable. Be sure to explain how the
answers to these questions would help to evaluate the advice.
 Write a response in which you discuss what questions would need to be answered in order to decide
whether the recommendation is likely to have the predicted result. Be sure to explain how the answers to
these questions would help to evaluate the recommendation.
 Write a response in which you discuss what questions would need to be answered in order to decide
whether the prediction and the argument on which it is based are reasonable. Be sure to explain how the
answers to these questions would help to evaluate the prediction.
 Write a response in which you discuss one or more alternative explanations that could rival the proposed
explanation and explain how your explanation(s) can plausibly account for the facts presented in the
argument.
69

 Write a response in which you discuss what questions would need to be addressed in order to decide
whether the conclusion and the argument on which it is based are reasonable. Be sure to explain how the
answers to the questions would help to evaluate the conclusion.
"Analyze an Argument" is a critical thinking task requiring a written response. Consequently, the analytical skills
displayed in the evaluation carry great weight in determining your score; however, the clarity with which the idea is
conveyed is also important to the overall score.
Preparingfor theArgumentTask
Since the Argument task is meant to assess analytical writing and informal reasoning skills that are developed
throughout education, it has been designed neither to require any specific course of study nor to advantage
students with a particular type of training.
Although one does not need to know special analytical techniques and terminology, one should be familiar with the
directions for the Argument task and with certain key concepts, including the following:
 alternative explanation — a competing version of what might have caused the events in question that
undercuts or qualifies the original explanation because it too can account for the observed facts
 analysis — the process of breaking something (e.g., an argument) down into its component parts in order to
understand how they work together to make up the whole
 argument — a claim or a set of claims with reasons and evidence offered as support; a line of reasoning
meant to demonstrate the truth or falsehood of something
 assumption — a belief, often unstated or unexamined, that someone must hold in order to maintain a
particular position; something that is taken for granted but that must be true in order for the conclusion to
be true
 conclusion — the end point reached by a line of reasoning, valid if the reasoning is sound; the resulting
assertion
 counterexample — an example, real or hypothetical, that refutes or disproves a statement in the argument
 evaluation — an assessment of the quality of evidence and reasons in an argument and of the overall merit
of an argument
An excellent way to prepare for the "Analyze an Argument" task is to practice writing on some of the published
Argument topics. There is no one way to practice that is best for everyone. Some prefer to start practicing without
adhering to the 30-minute time limit. If you follow this approach, take all the time you need to evaluate the
argument. Regardless of the approach you take, consider the following steps:
 Carefully read the argument and the specific instructions — you might want to read them more than once.
 Identify as many of the argument's claims, conclusions and underlying assumptions as possible and evaluate
their quality.
 Think of as many alternative explanations and counterexamples as you can.
 Think of what specific additional evidence might weaken or lend support to the claims.
70

Tipsfor theArgumentTask
One is free to organize and develop response in any way one thinks will effectively communicate one’s evaluation of
the argument. The response may, but need not, incorporate particular writing strategies learned in English
composition or writing-intensive college courses.
For example, raters will see some essays at the 6 score level that begin by briefly summarizing the argument and
then explicitly stating and developing the main points of the evaluation. The raters know that a writer can earn a
high score by developing several points in an evaluation or by identifying a central feature in the argument and
developing that evaluation extensively.
The format and organization that support and enhance the overall effectiveness of evaluation should be identified.
This means using as many or as few paragraphs as you consider appropriate for your response, e.g., create a new
paragraph when your discussion shifts to a new point of evaluation.
Similarly, examples can be used to help illustrate an important point in the evaluation or move the discussion
forward. However, one has to remember that it is the critical thinking and analytical writing that is being assessed,
not the ability to come up with examples. What matters is not the form the response takes, but how insightfully the
evaluation of the argument and how articulately it was communicated to academic raters within the context of the
task.
SampleArgumentTask
Following is a sample Argument task that you might see on the test:
In surveys Mason City residents rank water sports (swimming, boating and fishing) among their favorite recreational
activities. The Mason River flowing through the city is rarely used for these pursuits, however, and the city park
department devotes little of its budget to maintaining riverside recreational facilities. For years there have been
complaints from residents about the quality of the river's water and the river's smell. In response, the state has
recently announced plans to clean up Mason River. Use of the river for water sports is therefore sure to increase. The
city government should for that reason devote more money in this year's budget to riverside recreational facilities.
Write a response in which you examine the stated and/or unstated assumptions of the argument. Be sure to explain
how the argument depends on the assumptions and what the implications are if the assumptions prove
unwarranted.
Strategies for this Topic
This argument cites a survey to support the prediction that the use of the Mason River is sure to increase and thus
recommends that the city government should devote more money in this year's budget to the riverside recreational
facilities.
In developing your evaluation, you are asked to examine the argument's stated and/or unstated assumptions and
discuss what the implications are if the assumptions prove unwarranted. A successful response must discuss both
the argument's assumptions AND the implications of these assumptions for the argument. A response that does not
fully address the task may not receive a 5 or 6.
Though responses may well raise other points, some assumptions of the argument, and some ways in which the
argument depends on those assumptions, include:
71

 The assumption that people who rank water sports "among their favorite recreational activities" are actually
likely to participate in them. (It is possible that they just like to watch them.) This assumption underlies the
claim that use of the river for water sports is sure to increase after the state cleans up the Mason River and
that the city should for that reason devote more money to riverside recreational facilities.
 The assumption that what residents say in surveys can be taken at face value. (It is possible that survey
results exaggerate the interest in water sports.) This assumption underlies the claim that use of the river for
water sports is sure to increase after the state cleans up the Mason River and that the city should for that
reason devote more money to riverside recreational facilities.
 The assumption that Mason City residents would actually want to do water sports in the Mason River. (As
recreational activities, it is possible that water sports are regarded as pursuits for vacations and weekends
away from the city.) This assumption underlies the claim that use of the river for water sports is sure to
increase after the state cleans up the Mason River and that the city should for that reason devote more
money to riverside recreational facilities.
 The assumption that the park department's devoting little of its budget to maintaining riverside recreational
facilities means that these facilities are inadequately maintained. This assumption underlies the claim that
the city should devote more money in this year's budget to riverside recreational facilities. If current
facilities are adequately maintained, then increased funding might not be needed even if recreational use of
the river does increase.
 The assumption that the riverside recreational facilities are facilities designed for people who participate in
water sports and not some other recreational pursuit. This assumption underlies the claim that the city
should devote more money in this year's budget to riverside recreational facilities.
 The assumption that the dirtiness of the river is the cause of its being little used and that cleaning up the
river will be sufficient to increase recreational use of the river. (Residents might have complained about the
water quality and smell even if they had no desire to boat, swim or fish in the river.) This assumption
underlies the claim that the state's plan to clean up the river will result in increased use of the river for
water sports.
 The assumption that the complaints about the river are numerous and significant. This assumption
motivates the state's plan to clean up the river and underlies the claim that use of the river for water sports
is sure to increase. (Perhaps the complaints are coming from a very small minority; in which case cleaning
the river might be a misuse of state funds.)
 The assumption that the state's clean-up will occur soon enough to require adjustments to this year's
budget. This assumption underlies the claim that the city should devote more money in this year's budget to
riverside recreational facilities.
 The assumption that the clean-up, when it happens, will benefit those parts of the river accessible from the
city's facilities. This assumption underlies the claim that the city should devote more money to riverside
recreational facilities.
72

 The assumption that the city government ought to devote more attention to maintaining a recreational
facility if demand for that facility increases.
 The assumption that the city should finance the new project and not some other agency or group (public or
private).
Should any of the above assumptions prove unwarranted, the implications are:
 that the logic of the argument falls apart or is invalid or is unsound
 that the state and city are spending their funds unnecessarily

SampleEssayResponsesandRater Commentaryfor theArgumentTask


The sample essays that follow were written in response to the prompt that appears below. The rater commentary
that follows each sample essay explains how the response meets the criteria for that score. For a more complete
understanding of the criteria for each score point, see the "Analyze an Argument" Scoring Guide.
In surveys Mason City residents rank water sports (swimming, boating and fishing) among their favorite recreational
activities. The Mason River flowing through the city is rarely used for these pursuits, however, and the city park
department devotes little of its budget to maintaining riverside recreational facilities. For years there have been
complaints from residents about the quality of the river's water and the river's smell. In response, the state has
recently announced plans to clean up Mason River. Use of the river for water sports is therefore sure to increase. The
city government should for that reason devote more money in this year's budget to riverside recreational facilities.

Write a response in which you examine the stated and/or unstated assumptions of the argument. Be sure to
explain how the argument depends on the assumptions and what the implications are if the assumptions prove
unwarranted.
Note: All responses are reproduced exactly as written, including errors, misspellings, etc., if any.
Essay Response — Score 6
While it may be true that the Mason City government ought to devote more money to riverside recreational
facilities, this author's argument does not make a cogent case for increased resources based on river use. It is easy
to understand why city residents would want a cleaner river, but this argument is rife with holes and assumptions,
and thus, not strong enough to lead to increased funding.
Citing surveys of city residents, the author reports city resident's love of water sports. It is not clear, however, the
scope and validity of that survey. For example, the survey could have asked residents if they prefer using the river
for water sports or would like to see a hydroelectric dam built, which may have swayed residents toward river
sports. The sample may not have been representative of city residents, asking only those residents who live upon
the river. The survey may have been 10 pages long, with 2 questions dedicated to river sports. We just do not know.
Unless the survey is fully representative, valid, and reliable, it can not be used to effectively back the author's
argument.
73

Additionally, the author implies that residents do not use the river for swimming, boating, and fishing, despite their
professed interest, because the water is polluted and smelly. While a polluted, smelly river would likely cut down on
river sports, a concrete connection between the resident's lack of river use and the river's current state is not
effectively made. Though there have been complaints, we do not know if there have been numerous complaints
from a wide range of people, or perhaps from one or two individuals who made numerous complaints. To
strengthen his/her argument, the author would benefit from implementing a normed survey asking a wide range of
residents why they do not currently use the river.
Building upon the implication that residents do not use the river due to the quality of the river's water and the smell,
the author suggests that a river clean up will result in increased river usage. If the river's water quality and smell
result from problems which can be cleaned, this may be true. For example, if the decreased water quality and aroma
is caused by pollution by factories along the river, this conceivably could be remedied. But if the quality and aroma
results from the natural mineral deposits in the water or surrounding rock, this may not be true. There are some
bodies of water which emit a strong smell of sulphur due to the geography of the area. This is not something likely
to be afffected by a clean-up. Consequently, a river clean up may have no impact upon river usage. Regardless of
whether the river's quality is able to be improved or not, the author does not effectively show a connection between
water quality and river usage.
A clean, beautiful, safe river often adds to a city's property values, leads to increased tourism and revenue from
those who come to take advantage of the river, and a better overall quality of life for residents. For these reasons,
city government may decide to invest in improving riverside recreational facilities. However, this author's argument
is not likely significantly persuade the city goverment to allocate increased funding.
Rater Commentary for Essay Response — Score 6
This insightful response identifies important assumptions and thoroughly examines their implications. The essay
shows that the proposal to spend more on riverside recreational facilities rests on three questionable assumptions,
namely:
 that the survey provides a reliable basis for budget planning
 that the river’s pollution and odor are the only reasons for its limited recreational use
 that efforts to clean the water and remove the odor will be successful
By showing that each assumption is highly suspect, this essay demonstrates the weakness of the entire argument.
For example, paragraph 2 points out that the survey might not have used a representative sample, might have
offered limited choices, and might have contained very few questions on water sports.
Paragraph 3 examines the tenuous connection between complaints and limited use of the river for recreation.
Complaints about water quality and odor may be coming from only a few people and, even if such complaints are
numerous, other completely different factors may be much more significant in reducing river usage. Finally,
paragraph 4 explains that certain geologic features may prevent effective river clean-up. Details such as these
provide compelling support.
74

In addition, careful organization ensures that each new point builds upon the previous ones. For example, note the
clear transitions at the beginning of paragraphs 3 and 4, as well as the logical sequence of sentences within
paragraphs (specifically paragraph 4).
Although this essay does contain minor errors, it still conveys ideas fluently. Note the effective word choices (e.g.,
"rife with . . . assumptions" and "may have swayed residents"). In addition, sentences are not merely varied; they
also display skillful embedding of subordinate elements.
Since this response offers cogent examination of the argument and conveys meaning skillfully, it earns a score of 6.
Essay Response — Score 1
The statement assumes that everyone in Mason City enjoys some sort of recreational activity, which may not be
necessarily true. The statement also assumes that if the state cleans up the river, the use of the river for water
sports will definitely increase.
Rater Commentary for Essay Response — Score 1
The brevity of this two-sentence response makes it fundamentally deficient. Sentence 1 states an assumption that is
actually not present in the argument, and sentence 2 correctly states an assumption but provides no discussion of its
implications. Although the response may begin to address the assigned task, it offers no development. As such, it is
clearly "extremely brief ... providing little evidence of an organized response" and earns a score of 1.

SESSION- 12
EXERCISES
1. The president of Grove College has recommended that the college abandon its century-old tradition of all-female
education and begin admitting men. Pointing to other all-female colleges that experienced an increase in
applications after adopting coeducation, the president argues that coeducation would lead to a significant increase
in applications and enrollment. However, the director of the alumnae association opposes the plan. Arguing that all-
female education is essential to the very identity of the college, the director cites annual surveys of incoming
students in which these students say that the school's all-female status was the primary reason they selected Grove.
The director also points to a survey of Grove alumnae in which a majority of respondents strongly favored keeping
the college all female.
Write a response in which you discuss what questions would need to be answered in order to decide whether the
recommendation and the argument on which it is based are reasonable. Be sure to explain how the answers to these
questions would help to evaluate the recommendation.

2. The following appeared in a letter to the editor of a Batavia newspaper.


"The department of agriculture in Batavia reports that the number of dairy farms throughout the country is now 25
percent greater than it was 10 years ago. During this same time period, however, the price of milk at the local
Excello Food Market has increased from $1.50 to over $3.00 per gallon. To prevent farmers from continuing to
75

receive excessive profits on an apparently increased supply of milk, the Batavia government should begin to regulate
retail milk prices. Such regulation is necessary to ensure fair prices for consumers."
Write a response in which you discuss what questions would need to be answered in order to decide whether the
recommendation is likely to have the predicted result. Be sure to explain how the answers to these questions would
help to evaluate the recommendation.

3. The following appeared in a newsletter offering advice to investors.


"Over 80 percent of the respondents to a recent survey indicated a desire to reduce their intake of foods containing
fats and cholesterol, and today low-fat products abound in many food stores. Since many of the food products
currently marketed by Old Dairy Industries are high in fat and cholesterol, the company's sales are likely to diminish
greatly and company profits will no doubt decrease. We therefore advise Old Dairy stockholders to sell their shares,
and other investors not to purchase stock in this company."
Write a response in which you discuss what questions would need to be answered in order to decide whether the
advice and the argument on which it is based are reasonable. Be sure to explain how the answers to these questions
would help to evaluate the advice.

4. The following recommendation appeared in a memo from the mayor of the town of Hopewell.
"Two years ago, the nearby town of Ocean View built a new municipal golf course and resort hotel. During the past
two years, tourism in Ocean View has increased, new businesses have opened there, and Ocean View's tax revenues
have risen by 30 percent. Therefore, the best way to improve Hopewell's economy—and generate additional tax
revenues—is to build a golf course and resort hotel similar to those in Ocean View."
Write a response in which you examine the stated and/or unstated assumptions of the argument. Be sure to explain
how the argument depends on these assumptions and what the implications are for the argument if the assumptions
prove unwarranted.

5. The following is taken from a memo from the advertising director of the Super Screen Movie Production
Company.
"According to a recent report from our marketing department, during the past year, fewer people attended Super
Screen-produced movies than in any other year. And yet the percentage of positive reviews by movie reviewers
about specific Super Screen movies actually increased during the past year. Clearly, the contents of these reviews
are not reaching enough of our prospective viewers. Thus, the problem lies not with the quality of our movies but
with the public's lack of awareness that movies of good quality are available. Super Screen should therefore allocate
a greater share of its budget next year to reaching the public through advertising."
Write a response in which you discuss what questions would need to be answered in order to decide whether the
recommendation and the argument on which it is based are reasonable. Be sure to explain how the answers to these
questions would help to evaluate the recommendation.
76

6. The following appeared in a business magazine.


"As a result of numerous complaints of dizziness and nausea on the part of consumers of Promofoods tuna, the
company requested that eight million cans of its tuna be returned for testing. Promofoods concluded that the
canned tuna did not, after all, pose a health risk. This conclusion is based on tests performed on samples of the
recalled cans by chemists from Promofoods; the chemists found that of the eight food chemicals most commonly
blamed for causing symptoms of dizziness and nausea, five were not found in any of the tested cans. The chemists
did find small amounts of the three remaining suspected chemicals but pointed out that these occur naturally in all
canned foods."
Write a response in which you discuss what questions would need to be addressed in order to decide whether the
conclusion and the argument on which it is based are reasonable. Be sure to explain how the answers to the
questions would help to evaluate the conclusion.

7. The following appeared in a memo from the vice president of marketing at Dura-Socks, Inc.
"A recent study of Dura-Socks customers suggests that our company is wasting the money it spends on its patented
Endure manufacturing process, which ensures that our socks are strong enough to last for two years. We have
always advertised our use of the Endure process, but the new study shows that despite the socks' durability, our
customers, on average, actually purchase new Dura-Socks every three months. Furthermore, customers surveyed in
our largest market—northeastern United States cities—say that they most value Dura-Socks' stylish appearance and
availability in many colors. These findings suggest that we can increase our profits by discontinuing use of the
Endure manufacturing process."
Write a response in which you discuss what questions would need to be answered in order to decide whether the
recommendation and the argument on which it is based are reasonable. Be sure to explain how the answers to these
questions would help to evaluate the recommendation.

8. The following is a letter to the editor of an environmental magazine.


"In 1975 a wildlife census found that there were seven species of amphibians in Xanadu National Park, with
abundant numbers of each species. However, in 2002 only four species of amphibians were observed in the park,
and the numbers of each species were drastically reduced. There has been a substantial decline in the numbers of
amphibians worldwide, and global pollution of water and air is clearly implicated. The decline of amphibians in
Xanadu National Park, however, almost certainly has a different cause: in 1975, trout—which are known to eat
amphibian eggs—were introduced into the park."
Write a response in which you discuss what specific evidence is needed to evaluate the argument and explain how
the evidence would weaken or strengthen the argument.
77

9. The following is a letter to the editor of an environmental magazine.


"Two studies of amphibians in Xanadu National Park confirm a significant decline in the numbers of amphibians. In
1975 there were seven species of amphibians in the park, and there were abundant numbers of each species.
However, in 2002 only four species of amphibians were observed in the park, and the numbers of each species were
drastically reduced. One proposed explanation is that the decline was caused by the introduction of trout into the
park's waters, which began in 1975. (Trout are known to eat amphibian eggs.)"
Write a response in which you discuss one or more alternative explanations that could rival the proposed explanation
and explain how your explanation(s) can plausibly account for the facts presented in the argument.

10. The following appeared in a memo at XYZ company.


"When XYZ lays off employees, it pays Delany Personnel Firm to offer those employees assistance in creating
résumés and developing interviewing skills, if they so desire. Laid-off employees have benefited greatly from
Delany's services: last year those who used Delany found jobs much more quickly than did those who did not.
Recently, it has been proposed that we use the less expensive Walsh Personnel Firm in place of Delany. This would
be a mistake because eight years ago, when XYZ was using Walsh, only half of the workers we laid off at that time
found jobs within a year. Moreover, Delany is clearly superior, as evidenced by its bigger staff and larger number of
branch offices. After all, last year Delany's clients took an average of six months to find jobs, whereas Walsh's clients
took nine."
Write a response in which you discuss what specific evidence is needed to evaluate the argument and explain how
the evidence would weaken or strengthen the argument.

SESSION-13
COMPETENCY -2 (B)
SYNTACTIC RELATION
Sentence Corrections
How to approach sentence correction questions
It would be very easy to read the choices and pick the one that sounds best, but the sentences are often very
confusing, and most sentence correction questions test more than one grammar point. Therefore, there is a need
for a systematic approach:

1. Read the sentence carefully and try to spot an error.


2. After identifying the error, eliminate the answer choices that contain the same error. (This often removes
two or three choices.)
3. Go though the remaining choices to find a second error and fix that.
4. If an error is spotted consider option A.
Finally read the whole sentence with the option you are selecting in place.
78

Examples

1. Trying to keep her balance on the icy surface, the last competitor's ski-tip caught the pole and somersaulted into
the soft snow.
A. the last competitor's ski-tip caught the pole and somersaulted into the soft snow.
B. the ski-tip of the last competitor caught the pole and somersaulted in the soft snow.
C. the last competitor caught the pole with the tip of her ski, and somersaulted into the soft snow.
D. the last competitor caught the pole with her ski-tip, which made her somersault into the soft snow.
E. the last competitor somersaulted into the soft snow when the tip of her ski was caught by the pole.

Correct Answer: C
Explanation:
After the comma we need the name of the person. In answer D, the use of which is incorrect. Answer E
unnecessarily uses a passive construction.
2. The temperature dropped suddenly last night, which will mean that the shoots emerging from the soil will be
killed by the frost.
A. which will mean that the shoots emerging from the soil will be killed by the frost.
B. which will mean that the frost will kill the shoots emerging from the soil.
C. and this will mean that the shoots emerging from the soil will be killed by the frost.
D. and the resulting frost will kill the shoots that are emerging from the soil.
E. and as a result, the shoots will be killed by the frost, emerging from the soil.

Correct Answer: D
Explanation: The word which does not have an antecedent noun, and so the construction is incorrect in A and B.
Similarly, in C this does not have a noun to which it refers. D is the best choice. In E the phrase emerging from the
soil is dangling.
3. The impostor eluded detection for so long because she conducted herself as though she were a licensed
practitioner.
A. as though she were a licensed practitioner.
B. as though she was a licensed practitioner.
C. like she was a licensed practitioner.
D. like as if she was a licensed practitioner.
E. as if she was a practitioner with a license.

Correct Answer: A
Explanation: The sentence is correct because we change was to were in situations that are counter to fact or highly
unlikely.
79

4. Being abandoned by our friends is the cause of great sorrow for us.
A. Being abandoned by our friends is the cause of great sorrow for us.
B. Our being abandoned by our friends is the cause of great sorrow.
C. Being abandoned by our friends, we feel great sorrow.
D. Abandoned by our friends, sorrow is the result.
E. We feel great sorrow when our friends abandon us.

5. Among the many reasons for his defeat in the election was his arrogant assumption that his constituents were
incapable of understanding economic conditions, and his unwarranted attack on his chief opponent.
A. was his arrogant assumption that his constituents were incapable of understanding economic conditions
B. were his arrogant assumption that his constituents were incapable of understanding economic conditions
C. were his arrogant assumptions that his constituents were incapable of understanding economical conditions
D. were his arrogant assumption that his constituents would be incapable of understanding economics
E. was the arrogant assumption that his constituents was incapable of understanding economic conditions

6. More and more holidaymakers are choosing to fly to remote islands in search of the perfect beach; seeking sand,
sun and palm trees, rather than centers of entertainment.
A. ; seeking sand, sun and palm trees, rather than centers of entertainment.
B. ; seeking sad, sun, palm trees and not entertainment.
C. , with sand, sun, palm trees and no entertainment.
D. , they seek sand, sun and palm trees, rather than entertainment centers.
E. ; they seek sand, sun and palm trees, rather than centers of entertainment.

7. The government requires that these forms should be submitted before the end of the financial year.
A. that these forms should be submitted
B. that these forms be submitted
C. for these forms to be submitted
D. these forms submission
E. these forms should be submitted

8. After arduous months of fighting, the sight of the white flag being raised generated as much relief on the victor's
side than it did on the vanquished.
A. as much relief on the victor's side than it did on the vanquished.
B. as much relief among the victors as among the vanquished.
C. as much relief on the victor's side as it did on the vanquished's.
D. relief both on the victor's side as well as on the vanquished's.
E. relief both for the victor and the vanquished side.
80

9. The best way to encourage innovative thinking is not to promise financial rewards for ideas, but to ensure that the
person making the suggestion receives recognition for his contribution.
A. but to ensure that the person making the suggestion receives recognition for his contribution.
B. but to ensure that the person who makes the suggestion will be receiving recognition for his contribution.
C. but rather by ensuring that the person making the suggestion receives recognition for his contribution.
D. but rather ensure that suggestion-maker receives recognition for his contribution.
E. but instead make sure that the suggestion-maker will receive recognition.

10. It ought to be her with whom you share your secrets, not me.
A. her with whom you share your secrets, not me
B. her with whom you share your secrets, not I.
C. she with whom you share your secrets, not me.
D. she with whom you share your secrets, not I.
E. her with who you share your secrets, not me.

Each sentence given belowcontains one or more mistakes. Rewrite the sentences correcting the mistakes.
1. I wouldn’t mind your coming late if you do not wake me on.
2. He is independent from his parents.
3. Sonia is taller of the two girls.
4. Could you please move the bit?
5. There I saw his Mom whomhe said was away.
6. This chair is quite cheap at $20.
7. We do not sell things at credit.
8. As we labour, so shall we reward.
9. I’m too tired that I cannot attend the class.
10. She has been absent since three days.

We use verb tense to indicate the time of the action of the verb. Of course, the basic meat-and-potato tenses are
past, present, and future. Grammar would be very easier if these were the only tenses you had to know. But life is
complicated, and therefore so is grammar. This article examines a variation on these tenses: the progressive
tenses. The progressive tenses emphasize that the action discussed is in process, is happening right at the time
specified. Another way to say it: the emphasis of the progressive tenses is simultaneity.

The Present Progressive


Consider the difference between these two sentences.
1) I walk to work. [simple present tense]
2) I am walking to work. [present progressive tense]
81

This is one of the hardest distinctions for folks learning English as a second language to master, because is absent as
a verb form in many other languages. The first describes a general condition that is true at the present time. The
implication of sentence #1 is that I walk to work every day, that this is repeated and ongoing condition in my life.
Sentence #2 carries the implication that, at the very moment I speak it, I amperforming the act of walking to work:
for example, if a friend called on my cell phone while I was in the process of walking to work, I would say sentence
#2. The emphasis of the present progressive is on the fact that articulating the action and performing the action are
simultaneous.
Notice that the form of the present progressive is the present form of the verb “to be” plus the present participle —-
the –ing participle.
The Past Progressive
Often, when describing a past action, it’s enough to say that the action happened.
3) Yesterday evening, I walked my dog.
That’s the simple past tense. That simply makes clear that the action happened in the past. Sometimes, we want to
make an additional distinction clearer: we want to make clear exactly when the action was happening, or make clear
that two things happened at the same time. For example:
4) At 7:30 pm last night, I was walking my dog.
5) Yesterday evening, as I was walking my dog, my broker called.
The underlined verbs are in the past progressive. In both cases, we are emphasizing that the performance of the
action was simultaneous with something else — with the clock time of 7:30 pm or with the phone-call from the
broker.
The Future Progressive
This is a relatively unusual case, which is somewhat unlikely to appear on the GMAT Sentence Correction. The
distinction is analogous to that in the past tenses. Sometimes, it’s enough to say that an action will take place.
6) Tomorrow, I will visit my grandmother.
This is the simple future tense. This indicates only that, sometime in the 24 hour period of tomorrow, I will perform
the action of visiting my grandmother. Under certain circumstances, we to make further specifications: we want to
indicate either exactly when the action will take place or that two future actions will happen at the same time.
7) Tomorrow at 2:30 pm, I will be visiting my grandmother.
8) Tomorrow afternoon, when my roommate will be practicing for his opera performance, I will be visiting my
grandmother.
The underlined verbs are in the future progressive. In both cases, we are emphasizing that the performance of the
action will be simultaneous with something else: in the first, the action will be simultaneous with a clock time of
2:30 pm; in the second, the two actions, visiting grandma and practicing opera, are simultaneous. In fact, the
subordinate conjunction “when” already emphasizes simultaneity, so some folks would argue that having both verbs
in the future progressive with the word “when” is redundant, and should be emended to one of the following:
82

8a) Tomorrow afternoon, when my roommate practices for his opera performance, I will be visiting my
grandmother.
8b) Tomorrow afternoon, when my roommate practices for his opera performance, I will visit my grandmother.
Here are a couple of Sentence Correction Practice Questions involving progressive tenses.
Practice Questions
1) Mozart himself was a piano virtuoso, and the piano parts of his piano concerti, especially the mature work
composed in 1784 and after, have astonishingly difficult finger work that having demanded incomparable technique
to produce the required elegance and precision.
(A) that having demanded (B) which demanded (C) that had demanded
(D) that demands (E) which is demanding

2) Punctuated equilibrium is a biological theory that regards evolution not as a gradual process by which one species
slowly and continuously transforms into another, rather a process in which species were remaining stable for long
periods and then have dramatic change in isolated short bursts.
(A) rather a process in which species were remaining stable for long periods and then have dramatic change
(B) but as a process in which species remain stable for long periods and change dramatically
(C) but a process in which species remain stable for long periods and then have dramatic change
(D) yet as a process in which species remained stable for long periods and change dramatically
(E) but also as a process in which species were remaining stable for long periods and were changing dramatically
Practice Question Explanations
1) First, the “that-which” distinction, about which you can read more here. As a general rule, when a clause is
separated by commas from the rest of the sentence, it should use “which”, but when it follows the noun modified
without the break of a comma, it should use “that.” That strongly suggests that (B) and (E) are not correct.
Let’s look at the verb tenses:
(A) having demanded = participle, not a verb at all = automatically wrong
(B) demanded = verb in simple past tense (C) had demanded = verb in past perfect tense
(D) demands = verb in simple present tense (E) is demanding = verb in present progressive tense

In this situation, the Mozart piano concertos exist, and they have difficult finger work – in the present, this is the
case. This difficult finger work demands incomparable technique, any time that a pianist sits down to play one of
these concerti. It is a general present condition.
All of this happens at the present time, and could happen today, so the past tense (B) is out.
The past perfect tense indicate an action that happens before another past action, so this is complete
inappropriate. (C) is right out.
83

We do not mean to imply that, right as this sentence is spoken, someone happens to be playing a Mozart concerto
right at that moment. That could be true by coincidence, but it is not the intent of the sentence to emphasize that
simultaneity, so (E) is out.
That leaves (D), the simple present tense, as the verb that most aptly describes any action that is generally true in
present times though not necessarily true at this precise moment. Answer = D.
2) First of all, the conjunction. The correct idiom is “not X but Y.” We need the second half of the sentence to begin
with the word “but. (A) and (D) do not complete this idiom correct, so they are wrong. The construction “not X but
also Y” confuses the “not X but Y” idiom with the “not only X but also Y” idiom —“not X but also Y” does not
construct either correctly, so (E) is wrong.
Furthermore, the X and Y must be parallel. The first part begins “as a gradual process by which …”, so the second
part, after the “but”, must also begin with “as” —only (B) does this, so that’s the correct answer.
Notice also the verb tenses. (D) has a past tense in parallel with a present tense, so that’s wrong. Similarly, (A) has
the past progressive in parallel with the present tense, so that’s also wrong. (E) has both verbs in the past
progressive tense, but that’s unusual, because what we’re discussing is an ongoing process, how evolution continues
to take place in the natural world. Both (B) and (C) have the simple present test, which is correct and which mirrors
the simple present test of “transforms” before the underlined part. (C) does have the awkward wording, “have
dramatic change”, instead of “change dramatically” — as a general rule, if you have a choice — action as noun vs.
action as verb — always choose the latter. Actions should be expressed as verbs as much as possible. Again, this
makes (B) the best answer.
Answer = B.

Exercises
Which of phrases given below each sentence should replace the phrase printed in bold type to make the
grammatically correct? If the sentence is correct as it is, mark 'E' as the answer.
1. The small child does whatever his father was done.
a) has done b) did c) does d) had done e) No correction required
2. You need not come unless you want to.
a) You don't need to come unless you want to b) You come only when you want to
c) You come unless you don't want to d) You needn't come until you don't want to
e) No correction required.
3. There are not many men who are so famous that they are frequently referred to by their short names only.
a) initials b) signatures c) pictures d) middle names e) No correction required
84

4. The man to who I sold my house was a cheat.


a) to whom I sell b) to who I sell c) who was sold to d) to whom I sold e) No correction
5. They were all shocked at his failure in the competition.
a) were shocked at all b) had all shocked at c) had all shocked by
d) had been all shocked on e) No correction required
6. I need not offer any explanation regarding this incident - my behaviour is speaking itself.
a) will speak to itself b) speaks for itself c) has been speaking
d) speaks about itself e) No correction required
7. He is too important for tolerating any delay.
a) to tolerate b) to tolerating c) at tolerating
d) with tolerating e) No correction required
8. The population of Tokyo is greater than that of any other town in the world.
a) greatest among any other b) greater than all otherc) greater than those of any other
d) greater than any other e) No correction required
9. The performance of our players was rather worst than I had expected.
a) bad as I had expected b) worse than I had expected c) worse than expectation
d) worst than was expected e) No correction required
10. Why did you not threw the bag away?
a) did you not throw b) had you not threw c) did you not thrown
d) you did not thrown e) No correction required
11. Shapes of gods and goddess are worshipped by people.
a) Images b) Reflections c) Clay shapes d) Clay toys e) No correction required
12. In addition to enhanced their reputations through strategic use of philanthropy, companies are sponsoring
social initiatives to open new markets.
a) of enhancing their reputation b) to having enhance their reputation
c) to enhancing their reputationd) to have their reputation enhancing e) No correction required
13. The intruder stood quietly for few moments.
a) thus far succeeded for recovery b) so far succeeded in recovery of
c) as for as succeeded in recovery of d) so far succeeded to recover e) No correction required
85

14. He confidentially asked the crowd if they thought he was right and the crowd shouted that they did.
a) that he did b) that they had c) that he is d) that he didn't e) No correction required
15. Why should the candidates be afraid of English Language is not clear.
a) the candidates should be b) do the candidates be c) should be the candidates
d) are the candidates e) No correction required
16. He found the gold coin as he cleans the floor.
a) as he had cleaned b) while he cleans c) which he is cleaning d) while cleaning e) No Error
17. He admired the speed with which he completed the work and appreciating the method adopted by him.
a) appreciate the method being adopted b) appreciated the method adopted
c) ppreciate the method of adoption d) appreciated the method adopting method
e) No correction required
18. Maria unnecessarily picked up a quarrel with Rani and left the party hurried.
a) has picked up b) picked on c) picked d) No correction required e) picking up
19. She cooks, washes dishes, does her homework and then relaxing.
a) relaxing then b) then is relaxing c) relaxing is then d) then relaxes e) No correction required
20. Acquisition of certain specific skills can be facilitated from general awareness, education to novel situations
a) can be facilitated by b) may facilitate through c) can be felicitated with
d) may be felicitated with e) No correction required
21. He never has and ever will take such strong measures.
a) had taken nor will ever take b) had taken and will ever take c) has and never will take
d) had and ever will take e) No correction required
22. Technology must use to feed the forces of change.
a) must be used to feed b) must have been using to feed c) must use having fed
d) must be using to feed e) No correction required
23. Anyone interested in the use of computers can learn much if you have access to a personal computer.
a) they have access b) access can be available c) he or she has access
d) one of them have access e) No correction required
24. They are not beware of all the facts.
a) are not aware for b) are not aware of c) are not to be aware
86

d) must not to be aware for e) No correction required


25. We can not always convey ourselves in simple sentences.
a) cannot always convey b) can not always express c) cannot always express
d) can not always communicate e) No correction required

SESSION -14
SENTENCE RESTRUCTURING
Exercises
1. Created in October, 1972, the Consumer Product Safety Commission, which enacted a host of regulations
affecting a wide range of goods, fromoverly flammable attresses to unhealthy aerosol sprays to fireworks.

A. the Consumer Product Safety Commission, which enacted a host of regulations affecting a wide range of goods
B. The Consumer Product Safety Commission enacted a host of regulations affecting a wide range of goods
C. a host of regulations affecting a wide range of goods enacted by the Consumer Product Safety Commission
D. a host of regulations enacted by the Consumer Product Safety Commission, which affected a wide range of goods
E. when a host of regulations affecting a wide range of goods was enacted by the Consumer Product Safety
Commission

2. The radiation emanating from the Sun has been very nearly constant for millions of years, but the amount of heat
that falls on Earth's surface depends on both the length of the period of daylight as well as on the angle of incidence
of incoming sunlight.
A. depends on both the length of the period of daylight as well as on
B. depends both on the length of the period of daylight as well as
C. depends both on the length of the period of daylight and on
D. both depends on the length of the period of daylight and
E. both depends on the length of the period of daylight and on

3. Because you simultaneously focus two eyes on an object, employing a sophisticated method of determining
distance and perceiving depth, we call our vision binocular.
A. you simultaneously focus two eyes on an object B. you simultaneously focus on an object by two eyes
C. of our simultaneous focus on an object with two eyes
D. of our simultaneously focusing on an object by two eyes
E. we simultaneously focus two eyes on an object
87

4. A gale-force solar storm that hit the Earth on March 13, 1989, sent electrons arcing across power lines, railroad
tracks, and expanses of iron-bearing rock in eastern Canada, where alarmed engineers tried but failed to keep the
Hydro-Quebec power grid operating.
A. tried but failed to keep B. tried but failed in their keeping
C. had tried but were failing in keeping D. were trying but having failed to keep
E. were trying but they had failed in keeping

5. The body heat of reptiles and amphibians is not regulated by internal thermostats, therefore, they raise or lower
their own temperature in that they seek or flee warmth, sometimes merely by moving into or out of direct sunlight.
A. thermostats, therefore, they raise or lower their own temperature in that they seek or flee
B. thermostats, therefore raising or lowering their own temperature by seeking or fleeing
C. thermostats, therefore, in raising or lowering their own temperature by seeking or fleeing
D. thermostats; therefore, they raise or lower their own temperature by seeking or fleeing
E. thermostats; therefore, they raise or lower their own temperature in that they seek or flee

6. Until about 10,000 years ago, humans had lived almost exclusively as hunters and gatherers, harvesting wild
plants and animals, then, people had begun to cultivate domesticated plants, and to grow food.
A. animals, then, people had begun B. animals, people then began
C. animals, so that then, people had begun D. animals; so that then, people began
E. animals; then, people began

7. In 1777, when news reached Colonel Ludington that the town of Danbury, Connecticut, was being looted and
burned by British troops, his daughter Sybil rode forty miles through the night so that the soldiers of her father's
regiment had been alerted to the danger.
A. so that the soldiers of her father's regiment had been alerted
B. so that the soldiers of her father's regiment may be alerted
C. alerting the soldiers of her father's regiment D. and alerting the soldiers of her father's regiment
E. for alerting the soldiers of her father's regiment

8. Two hundred million years ago, during the Jurassic period, all of the land area on Earth, found in one large
continent called Pangea, which eventually broke apart through a process now known as continental drift.
A. all of the land area on Earth, found in one large continent called Pangea, which
B. one large continent, in which all of the land area on Earth was found, called Pangea, and it
C. one large continent called Pangea, in which all of the land area on Earth was found, which
D. one large continent was called Pangea, in which all of the land area on Earth was found and
E. all of the land area on Earth was found in one large continent called Pangea, which
88

9. In 1990, the Fish and Wildlife Service has placed the northern spotted owl on the threatened species list, a native
of Pacific Northwest forests.
A. the Fish and Wildlife Service has placed the northern spotted owl on the threatened species list, a native of Pacific
Northwest forests
B. the Fish and Wildlife Service placed the northern spotted owl, a native of Pacific Northwest forests, on the
threatened species list
C. a native of Pacific Northwest forests, the Fish and Wildlife Service placed the northern spotted owl on the
threatened species list
D. the northern spotted owl has been placed on the threatened species list by the Fish and Wildlife Service, which is
a native of Pacific Northwest forests
E. the northern spotted owl, a native of pacific Northwest forests, placed on the threatened species list by the Fish
and Wildlife Service

10. In the United States at the end of the nineteenth century, public places such as theaters, restaurants, shops, and
banks had installed electric lighting, but electricity was in fewer than one percent of homes and lighting
still provided mainly by candles or gas.
A. electricity was in fewer than one percent of homes and lighting still
B. electricity was in fewer than one percent of homes, where lighting was still
C. fewer than one percent of homes that had electricity, where lighting was still being
D. fewer than one percent of homes with electricity, having lighting that was still
E. fewer than one percent of homes had electricity, where lighting had still been

11. Mainly through the efforts of Jane Goodall, apes were revealed to be not blundering, primitive automatons, but
beings with as complex and subtle social lives like us.
A. as complex and subtle social lives like us B. complex and subtle social lives just like us
C. as complex and subtle social lives as us D. social lives complex and subtle just as ours
E. social lives as complex and subtle as ours

12. In the sixteenth century, the forests were destroyed in Britain, which meant not only a shortage of the most
important building material of the time, but also a fuel shortage.
A. the forests were destroyed in Britain, which meant not only a shortage
B. the forests in Britain were destroyed, which meant a shortage not only
C. the forests were destroyed in Britain, meaning a shortage not only
D. destruction of the forests in Britain meant not only a shortage
E. destruction of the forests in Britain meant a shortage not only
89

13. Britain's first Ice Age cave art—faint engravings of birds, ibexes, and a wild ox—were a recent discovery for a
team of archaeologists in a cave at Creswell Crags in central England.

A. were a recent discovery for B. were recently discovered by C. was a recent discovery that
D. was recently discovered by E. recently discovered by

14. Throughout the first half of the nineteenth century, many businesses remained open on Christmas, and so many
people doing their Christmas shopping on Christmas Day that commercial streets in New York City and Philadelphia
often were nearly impassable.
A. Christmas, and so many people doing B. Christmas, and many people doing
C. Christmas, with many people doing D. Christmas; many people did
E. Christmas; so many people did

15. Before the advent of industrial fishing, some regions of the ocean were too distant or too deep for fishers to
reach, with the fish living there to remain untouched.
A. with the fish living there to remain B. so the fish living there remained
C. so the fish that lived there remaining D. and so the fish that lived there remaining
E. and so with the fish that lived there remaining

16. Because she believed strongly that education was important for women as for men, Abigail Adams was unhappy
that she had not been allowed to receive the classical education accorded to the males of her time.
A. Because she believed strongly that education was B. Because of believing strongly in education as
C. Believing strongly that education was as D. In believing strongly in education as
E. While she believed strongly that education was

17. The eruption of Mount Vesuvius in A. D. 79 sent a column of tephra—dust, smoke, and pumice—nearly
seventeen miles into the air so that it covered the city of Pompeii with over eight feet of pumice.
A. so that it B. and so C. And D. and they E. so that they

18. To survive, the people now known as Paleo-Indians had to be resourceful and resilient, able to survive only if
they develop hunting and gathering techniques suited to their changing environments.
A. able to survive only if they develop B. able to survive only by developing
C. their ability to survive only by developing D. that they are able to survive only by developing
E. that they were able to survive only if they develop
90

19. Around 230 million years ago, the west coast of what is now Europe and the bulge of northwestern Africa began
to collide slowly with the east coast of North America, the coming together of the landmasses to form the
supercontinent Pangea.
A. the coming together of the landmasses to form the supercontinent Pangea
B. the landmasses coming together to form the supercontinent Pangea
C. and with the landmasses coming together to form the supercontinent Pangea
D. the supercontinent Pangea to be formed from the coming together of the landmasses
E. forming the supercontinent Pangea by the landmasses coming together

20. As executives of the newspaper seek to make the paper more competitive and to broaden an advertising base
heavily dependent on business-related advertising, they are exploring the addition of a Saturday edition to its
weekday schedule.
A. As executives of the newspaper seek to B. As executives of the newspaper, seeking that they
C. As executives of the newspaper, seeking to D. With executives of the newspaper seeking to
E. With executives of the newspaper seeking that they

COMPETENCY-3
SIMPLE EQUATIONS

1. A class starts at 10 a.m and lasts till 1.27 p.m. Four periods are held during this interval. After
each period, 5min was given free to the students. The exact duration of each period is :
a.42min b. 48 min c.51min d.53min
2. A boy was asked to multiply a number by 25. He instead multiplied the number by 52 and got
the answer 324 more than the correct answer. The number to be multiplied was :
a.12 b.15 c.25 d.32
3 . The number of girls in a class is 5 times the number of boys. Which of the following cannot
be the total of children in the class?
a.24 b.30 c.35 d.42
4. A boy read 3/8th of a book on one day and 4/5th of the remainder on another day. If there were
30 pages unread, How many pages did the book contain?
a.240 b.300 c.600 d.250
5. In an examination, a student scores 4 marks for every correct answer and loses 1 mark for
every wrong answer. If he attempts in all 60 questions and secures 130 marks, the number of
questions he attempts correctly, is :
a.35 b.38 c.30 d.40
6. The price of 10 chairs is equal to that of 4 tables. The price of 15 chairs and 2 tables together
is Rs.4000. The total price of 12chairs and 3 tables is:
a. Rs . 3000 b. Rs . 3750 c. Rs . 3840 d. Rs . 3900
7. The cost of ten mangoes and nine apples is Rs.104. The cost of twenty-seven mangoes and twenty-five apples is
Rs.285. What is the cost of three mangoes and two apples ?
a.Rs.24 b.Re.30 c.Rs.27 d.Rs.32
8. A man has some hens and cows. If the number of heads be 48 and the number of feet equals 140, then the
number of hens will be:
a.22 b.23 c.24 d.26
91

9. In a groups of cows and ducks, the number of legs are 24 more than twice the number of heads. What is the
number of cows in the group?
a.6 b.8 c.10 d.12
10. Twenty years from now, Sreedhar will be Six times as old as he was twenty years ago. What
is the present age of Sreedhar ?
a. 20 years b. 24 years c. 28 years d. 32 year
11. The age of a father is Six times the age of his Son. Twenty years from now, father’s age will
be twice as old as son’s age. What is the father’s present age ? (in years)
a. 25 years b. 40 years c. 30 years d. 20 years
12. Mrudhula gave birth to twins when she was twenty years old. After X years, the sum of the
ages of Mrudhula and her twin children is 50 years. What is the value of X ?
a. 5 b. 10 c. 15 d. 20
13. Ramesh is thrice as old as Suresh. Two years hence, Ramesh will be twice as old as Suresh. Find the Ramesh’s
present age (in Years)
a.2 b.3 c.4 d.6
14.Twenty seven years ago, a man’s age was half his wife’s present age. Sum of their present ages is 90 years. What
are the respective present ages of the man and his wife?
a.48,42 b.52,38 c.54,36 d.46,44

15. Shankar has an amount of Rs.7000 in the denominations of Rs.500 and Rs.100. How many
Rs.500 notes does he have, if he has a total of 22 notes with him ?
a.10 b.12 c.11 d.16

16. David gets on the elevator at the 11th floor of a building and rides up at the rate of 57 floors
per min. At the same time, Albert, gets on the elevator at the 51th floor of the same building
and rides down at the rate of 63 floors per min. If they continue travelling at these rates, then
at which floor will their paths cross?
a.19 b.28 c.30 d.37
17. The taxi charges in a city comprise of a fixed charge, together with the charge of the distance
charged. For a journey of 16km, the charges paid are Rs.156 and for a journey of 24km, the
charges paid are Rs.204.What will a person have to pay for travelling a distance of 30km?
a.Rs.236 b .Rs.240 c.Rs.248 d. Rs.252
18. Nalini has an amount of Rs.20 in coins of denominations of 50 paise and Rs.1. If she has a total of 30 coins with
her, how many Rs.1 coins does she have ?
a.20 b.10 c.15 d.30
19. A fraction is such that the numerator is five less than the denominator. Also four times the numerator is one
more than the denominator. Find the fraction.
a.4/9 b.3/8 c.2/7 d.7/12
20. The denominator of a fraction is two more than its numerator, If one is added to the denominator, the fraction
becomes ¾. Find the fraction.
a.7/9 b.5/7 c.11/13 d.9/11

RATIO PROPORTION & VARIATION


1.If a: b = 2:3, b : c = 4: 5, then a : b: c =
a. 8 : 12 : 15 b. 2 : 3 : 5 c. 2: 4: 5 d. 2: 7 : 5
2. Two whole numbers whose sum is 72 cannot be in the ratio:
a.5 :7 b. 3 :5 c. 3 : 4 d. 4 :5

3. If a carton containing a dozen mirrors is dropped, which of the following cannot be the ratio of broken mirrors to
un broken mirrors?
a. 2 : 1 b. 3 : 1 c. 3 : 2 d. 7 : 5
92

4. The ratio between two numbers is 3 : 4 and their L. C. M is 180. The first number is :
a.60 b.45 c. 20 d.15

5. The ages of two friends are in the ratio 6 : 5 if sum of their ages is 66. After how many years the ratio will become
7: 6?
a.11 b.6 c.10 d.12
6. A sum Rs.427 is to be divided among A,B and C such that 3 times A’s share , 4 times B’s
share and 7 times C’s share are all equal. The share of C is
a.Rs.84 b. Rs.147 c.Rs.196 d. Rs.240

7. The ratio of males and females in a group is 6 : 7. If 4 females join the group and 3 males
leave the group then the ratio of females to males in the becomes 4 : 3. What is the original
number of males and females in the group respectively?
a.48,56 b.54,63 c.42,49 d.36,42

8. A man divides his property so that his son’s share to his wife’s and the wife’s share to his
daughter are both in the ratio 3 : 1. If daughter gets Rs. 10000 less than the son, find the total
worth of the property?
a. Rs.12500 b. Rs.16250 c. Rs.18250 d.Rs.15500

9. I have one rupee coins, fifty paise coins and twenty-five paise coins. The number of coins are
in the ratio 5/2 : 3 : 4. If the total amount is Rs. 200. Find the number of one rupee coins?
a. 90 b.85 c.100 d.105

10. A mixture contains alcohol and water in the ratio 4 : 3. If 5litres of water is added to the
mixture, the ratio becomes 4 : 5. Find the quantity of alcohol in the given mixture?
a.10l b.12l c.14l d.13l

11. The ratio of present ages of two brothers is 1 : 2 and 5 years back, the ratio was of their ages
was 1 : 3.What will be the ratio of their ages after 5 years?
a.1 : 4 b.2 : 3 c. 3 :5 d. 5 : 6

12. The ratio between the ages of P and Q is 5 : 7 respectively. If the difference between Q’s
present age and P’s age after 6years is 2, what is the total of P’s and Q’s presents ages?
a.48years b.52years c.56years d. none of these

13. An alloy is to contain copper and zinc in the ratio 9 : 4. The zinc required to be melted with
24 kg of copper is:
a.10 2/3kg b.10 1/3kg c.9 2/3kg d.9kg

14. 60 kg of an alloy A is mixed with 100kg of alloy B. A has lead and tin in the ratio 3 : 2 and
the alloy B has tin and copper in the ratio 1 ; 4,then the amount of tin in the new alloy is:
a.36kg b.44kg c.53kg d.80kg

15.Gold is 19 times as heavy as water and copper is 9 times as heavy as water. In what ratio
should these be mixed to get an alloy 15 times as heavy as water?
a.1 :1 b. 2 :3 c. 1 :2 d. 3 : 2

16. 15litres of mixture contains 20% alcohol and the rest water. If 3l of water be mixed to the
mixture then the percentage of alcohol in the new mixture would be:
a. 15% b.16 2/3% c.17% d. 18 ½%
93

17. If the incomes of A and B are in the ratio of 3 : 4 and their expenditures in the ratio of 4 : 5 .
Find the ratio of their savings, given that B saves a third of his income.
a.13 : 20 b. 20 : 13 c.3 : 5 d.4 : 5

18. Zinc and copper are melted together in the ratio 9 : 11. What is the weight of melted
mixture, if 28.8kg of zinc has been consumed in it?
a.58kg b.60kg c.64kg d.70kg

19. x varies inversely as square of y. Given that y = 2 for x = 1. The value of x for y = 6 will be equal:
a.3 b.9 c.1/3 d.1/9

20. A is directly proportional to B when C is constant and inversely proportional to the square of
C when B is constant . In a particular instance when A = 18, B = 25, and C = 10, find A if C =
9 and B = 18
a. 24 b. 16 c. 32 d.12

PERCENTAGES
1. In a fraction, if numerator is increased by 40% and denominator is increased by 80%, then
what fraction of the original is the new fraction?
a.1/2 b.7/9 c.7/18 d. data inadequate

2. A scored 11% marks and failed by 60 marks. B scored 28% marks and obtained 8 marks more
than those required to pass. The approximate pass percentage is
a.42% b.26% c.28% d.46%

3. In a city 40% of the people are illiterate and 60% are poor. Among the rich, 10% are illiterate.
What percentage of the poor population is illiterate?
a.36% b.40% c.60% d.None of these

4. At an election involving two candidates , 68 votes were declared invalid. The winning
candidate secure 52% and wins by 98 votes. The total number of votes polled.
a.2382 b.2450 c.2518 d.None of these

5. A batsman scored 110 runs which included 3 boundaries and 8 sixes. What percent of his total
score did he make by running between the wickets.
a.45% b.500/11 c.600/11 d.55%

6. Ramesh invests 50% in raw material, 30% in machinery, 10% advertisement and has Rs.1372
can left with him. What is his total investment?
a.Rs.15,240 b.Rs.6,860 c.Rs.13,720 d.Rs.15,578

7. A’s salary is 40% of B’s salary which is 25% of C’s salary. What percentage of C’s salary is
A’s salary?
a.5% b.10% c.15% d.20%

8. Two numbers are respectively 25/2 % and 25% more than a third number. The first number as
a percentage of the second number is
a.50 b.60 c.75 d.90

9. If the price of a book is first decreased by 25% and then increased by 20% , then the net
change in the price will be
a. No change b.5% increase c.5%decrease d.10%decrease
94

10. A bag contains 600 coins of 25paise denomination and 1200 coins of 50p denominations. If
12% of 25p coins and 24% of 50p coins are removed, the percentage of money removed from
the bag is nearly
a.15.6% b.17.8% c.21.6% d.30%

11. The total population of a village is 5000. The number of males and females increases by 10%
and 15% respectively and consequently the population of the village becomes 5600.What
was the number of males in the village ?
a.2000 b.2500 c.3000 d.4000

12. The population of a town was 1,60,000 three years ago. If it is increased by 3%, 2.5% and
5% respectively in the last three year, then the present population is
a.1,77,000 b.1,77,366 c.1,77,461 d.1,77,596

13. A’s weight is 10% less than that of B and C’s weight is 10% more than that of B. By what
percent is A’s Weight less than that of C?
a. 20% b. 18 2/11% c. 1 9/11% d. 16 2/3%

14. From the salary of an officer, 10% is deducted as house rent, 20% of the rest he spends on
conveyance, 20%of the rest he pays as income tax and 10% of the balance, he spends clothes.
Then, he is left with Rs.15,552. Find his total salary?
a.Rs.25,000 b.Rs.30,000 c.Rs.35,000 d.Rs.40,000

15. In an examination, A Scored 25% less than B. By what percent did B score more than A?
a. 25% b. 20% c. 33 1/3% d. 36 2/3%

16. The price of a car is Rs.3,25,000. It was insured to 85% of its price. The car was damaged
completely in an accident and the insurance company paid 90% of the insurance. What was
the difference between the price of the car and the amount received?
a.Rs.32,500 b.Rs.48,750 c.Rs.76,375 d.81,250

17. The tax on a commodity is diminished by 15% and its consumption increases by 10%. Find
the effect on revenue?
a. increases by 6.5% b. increases by 5.5% c. decreases by 6.5% d. decreases by 5.5%

18. If the price of sugar increases by 25%, find howmuch percent its consumption be reduced so
as not to increase the expenditure?
a. 20% b. 25% c. 30% d. 22%

19. A reduction of 20% in the price of sugar enable purchaser to obtain 2 1/2kg more for Rs. 160.
Find original price of sugar?
a. Rs. 12 b. Rs.15 c. Rs.16 d. Rs.18

20. The price of a commodity rises from Rs.6 per kg to Rs. 7.50 per kg. If the expenditure cannot
increase the percentage of reduction in consumption
a. 15% b. 20% c. 25% d. 30%
95

PROFIT & LOSS


1. A sells a bicycle to B at a gain of 10% and B again sells it to C at a profit of 5%. If C pays
Rs. 462 to B, what is the cost price of the bicycle for A?
a. Rs. 400 b. Rs. 425 c. Rs. 442 d. Rs. 435

2. A sells a bicycle to B at a gain of 5% and B again sells it to C at a profit of 10%. Find the
resultant profit percent?
a.33/2 % b. 31/2 % c. 23/2 % d. 35/2 %

3 . A sold two horses for Rs. 990 each, gaining 10% on the one and losing 10% on the other,
Find his total gain or loss percent?
a. no loss no profit b. 1% loss c. 1% profit d. cannot decide

4. A dishonest dealer sells his goods at the cost price and still earns a profit of 25% by under
weighing. What weight does he use for a kg?
a. 750g b. 800g c. 825g d. 850g

5. If 11 oranges are bought for Rs.10 and sold 10 or Rs. 11. What is gain or loss percent?
a. 21% profit b. 21% loss c. 25% profit d. 25% profit

6. If the selling price of 12 articles is equal to the cost price of 18 articles. What is the profit
percent?
a. 40% b. 60% c. 45% d. 50%

7. If S. P is doubled then profit triples. Find the profit percent?


a. 66 2/3 b. 100 c. 105 1/3 d. 120

8. If a person makes a profit of 10% on ¼ th of the quantity and a loss of 20% on the rest. What
is his average percent profit or loss?
a.12.5% profit b. 12.5% loss c. 11.25% loss d.11.75% profit

9. Profit after selling an article for Rs. 425 is same as the loss after selling it for Rs. 355. The
cost price is
a. Rs.385 b. Rs. 390 c. Rs.395 d. Rs. 400

10. A buys two books for Rs. 410 and sells one at a loss of 20% and the other at a gain of 25%. If
both the books are sold at the same price. Find the c.p of two books?
a. Rs. 250, Rs. 160 b. Rs. 240, Rs. 170 c. Rs. 150, Rs. 260 d. none of these

11. The retail price of a water geyser is Rs. 1265. If the manufacturer gains 10%, the dealer
gains 15% and retailer gains 25%, then the cost price of the product is
a. Rs. 800 b. Rs. 900 c. Rs. 700 d. Rs. 600

12. There would be 10% loss if a toy is sold at Rs. 10.80 per piece. At what price should it be
sold to earn a profit of 20%?
a. Rs.12 b. Rs. 12.96 c. Rs. 14.40 d. Rs. 12.40

13. Ravi sells an article at a gain of 12 ½ %. If he had sold it at Rs. 22.50 more, he had have
gained 25%. The C.P of the article is
a. Rs. 162 b. Rs. 140 c. Rs.196 d. Rs. 180
96

14. A trader charges 20% over the C.P. He allowed a 10% discount on the listed price. What is
the net percentage gain?
a.15% b. 12% c. 8% d.10%

15. Find a single discount equivalent to three successive discounts of 10%, 20% and 30%?
a. 49.6% b. 46. 9% c. 50% d.36 .2%

16. A shop keeper sold an article for Rs. 6750 after giving a discount of 10% on marked price.
He would have earned a profit of 50%, had there been no discount. What was the actual
percentage of profit?
a.36% b. 40 % c. 35% d. none of these

17. Two shop keepers sell machines at the list price. The first allows two successive discounts of
16% and 30% and the second 20% and 26%. Which discount series is more advantageous to
the purchaser?
a. 1st shop keeper b. 2nd shop keeper c. both are equal b. can’t decide

18. The marked price of a watch is Rs.1600. The shopkeeper gives successive discounts of 10%
and k% to the customer. If the customer pays Rs. 1224 for the watch, find the value of k?
a.10% b. 20 % c. 25% d.15%

19. A shopkeeper sells the goods at 44% loss on cost price but uses 30% less weight. What
percentage profit or loss?
a. 20% profit b. 20% loss c. 25% profit d. 22% loss

20. A man buys 2 dozen bananas at Rs. 16 per dozen. After selling 18 bananas at the rate of
Rs. 12 per dozen, the shopkeeper reduced the rate to Rs. 4 per dozen. The percent loss is:
a. 25.2% b. 32.4% c. 36.5% d. 37.5%

SIMPLE & COMPOUND INTEREST


1. If the S.I on Rs. 3650 after 3 years is Rs. 1533, then the rate of interest is
a. 15% b.20% c.14% d. 12%

2. If Rs. 5000 is invested at 8% p.a, the amount after one year is


a. Rs.5,800 b. Rs.5,400 c. Rs 6,400 d. Rs. 6,800

3 . The S.I on sum of money is 1/9 of the sum and the no.of years is equal to the rate percent per
annum. Find the rate percent?
a. 10/3 % b.12/5% c.11% d. 10%

4. . A sum of money doubles it self in 10 years at S. I. What is the rate of interest?


a. 12% b.20% c.14% d. 10%

5. A sum was put at S. I at a certain rate for 2 years. Had it been put at 3% more rate, it would
have fetched Rs. 300 more. Find the sum?
a. Rs.5,800 b. Rs.5,400 c. Rs 5,000 d. Rs. 6,500

6. A sum of money amounts to Rs. 9800 after 5 years and Rs. 12,005 after 8 years at the same
rate of interest. The rate of interest per annum is:
a. 5 % b. 8 % c. 12 % d. 15 %
97

7. Find C.I on the sum 80000 for 2 years at the rate of 5% p.a?
a. Rs.8,800 b. Rs.8,200 c. Rs 8,400 d. Rs. 8,800

8. A sum of money placed at compound interest doubles itself in 5 yrs. It will amount to 8 times
itself at the same rate of interest in:
a. 7 yrs b. 10 yrs c. 15 yrs d.20 yrs

9. A sum of money amounts to Rs. 6690 after 3 yrs and to Rs. 10, 035 after 6 yrs on compound
interest. Find the sum?
a. Rs.4460 b. Rs.4400 c. Rs.4560 d. Rs.4800

10. A certain sum amounts to Rs.2300 in 3 years and to Rs 2,500 in 5 years at simple interest.
Find the sum and the rate of interest?
a. Rs.1200,6% p.a. b. Rs.1800,5% p.a. c. Rs.2000, 5% p.a. d. Rs.1500,6% p.a.
11. The rate of interest on a sum of money is 4% p.a for the first 2years, 6% p.a for the next 4
years and 8% p.a for the period beyond 6years. If the S.I accrued by the sum for a total
period of 9 years is Rs. 1,120, then the sum is in rupees
a. 2400 b.2200 c.2000 d. 2500

12. A sum of Rs. 600 amounts to Rs 720 in 4 yrs at simple interest.If the interest rate is increased
by 2 %, it would amount to how much? (in rupees)
a. 648 b.768 c. 726 d. 792

13. What will be the S.I earned on an amount of 16800in 9 months at the rate of 6 ¼% p.a?
a.Rs.787.50 b. Rs.812.50 c. Rs.860 d. Rs.887.50

14. Find the amount by investing Rs.8,000 at 5% p.a compound interest payable half yearly for
one and half years ?
a. Rs. 8715 b. Rs.8615 c. Rs.8815 d. Rs.8615

15. Find the C.I. by investing Rs.1,000 at 40% p.a compounded quarterly for 1 year ?
a. Rs. 464.10 b. Rs.474.10 c. Rs.408 d. Rs.508

16. A sum is invested for 3 years compounded at 5%, 10% and 20% respectively. In 3 years, if
the sum amounts to Rs. 16632, then find the sum?
a. Rs.11000 b. Rs.12000 c. Rs.13000 d. Rs.14000

17. The simple interest and the compound interest on a sumfor a period of two years are Rs 800
and Rs.864 respectively. Find the sum and the rate of interest.
a. 2500,16% b. 3600,8% c. 4000,16% d. 1750,12%

18. What will be difference between S.I and C.I on a sum of Rs. 4500 put for 2 years at 5% p.a?
a. Rs.11.25 b. Rs.12.25 c. Rs.13.35 d. Rs.14.25

19. What is the difference between simple interest and compound interest for 2 years on
Rs.10000 when the rate of interest is 11% for the first year and 12% for second year?
a.Rs.132 b. Rs.123 c. Rs.133 d. Rs.142

20. A man invested 1/3 of his capital at 7%, ¼ at 8% and the remainder at 10%. If his annual
income is Rs.561, the capital is :
a.Rs. 5400 b. Rs.6000 c. Rs.6600 d.Rs.7200
98

COMPETENCY-4
Deductions
Questions based on Deductions are solved using Venn Diagrams. To answer these questions one should have a
critical understanding of the words like ‘all’ ‘some’ etc and one should also know how to represent them in a Venn
diagram. In these questions two, three or four statements are followed by certain conclusions. One has to find the
conclusions that can definitely follow the given statements.
Directions for questions (1 – 5): Each of these questions consists of two statements followed by two conclusions I
and II. Find which of the conclusions logically follows the given statements, even though they are at variance with
commonly known facts. Mark your answer as
(1) If only I follows
(2) If only II follows
(3) If either I or II follows
(4) If neither I nor II follows
(5) If both I and II follows
1. Statements:
All seas are bees
Some teas are bees
Conclusions:
I. All teas are seas
II. Some seas are teas

2. Statements:
All villages are towns
No country is a village
Conclusions:
I. No town is a country
II. No country is a town

3. Statements:
Some wealthy are healthy
Some healthy are not stealthy
Conclusions:
I. All wealthy are stealthy
II. Some wealthy are not stealthy

4. Statements:
All weddings are writings
All weddings are wirings
Conclusions:
I. Some writings are wirings
II. No wiring is a writing

5. Statements:
Some zeroes are not heroes
All zeroes are poor
99

Conclusions:
I. All heroes are poor
II. Some poor are not heroes

Directions for questions (6 – 14): Each of these questions consists of two statements followed by four conclusions.
Find which of the conclusion(s) logically follows the given statements, even though they are at variance with
commonly known facts.
6. Statements:
All kites are flights
All lights are kites
Conclusions:
I. Some flights are lights
II. Some lights are flights
III. All flights are lights
IV. All lights are flights

1. Only I and II follows


2. Only II and III follows
3. Only I and IV follows
4. Only I, II and IV follows
5. All follow

7. Statements:
No pen is a board
No pencil is a board
Conclusions:
I. Some pens are not pencils
II. No pencil is a pen
III. Some pencils are not pens
IV. No pen is a pencil

1 All follows
2 Only II and III follows
3 Only II and IV follows
4 Only I and II follows
5 None follows

8. Statements:
Some different are difficult,
No deterrent is different
Conclusion:
I. Some different are deterrent.
II. Some difficult are not different.
III. Some difficult are not deterrent.
IV. Some different are not deterrent.

1. Only 1 follows.
2. Only IV follows.
3. Only III and IV follow.
4. Only III follows.
5. None follows.
100

9. Statements:
All cats are dogs,
No dog is a rat.
Conclusions:
I. some dogs are cats.
II. No cat is a rat.
III. No rat is a cat.
IV. All dogs are cats.

1. Only I and II follow.


2. Only I, II and III follow.
3. Only III and IV follow.
4. Only II and IV follow.
5. Only III follows.

10. Statements:
Some crazy are lazy,
Some lazy are not hazy.
Conclusions:
I. Some hazy are lazy.
II. Some crazy are hazy.
III. No hazy is crazy.
IV. All lazy are crazy.

1. Only II follows.
2. Only III follows.
3. Only IV follows.
4. Only either II or III follows.
5. Only I and either II or III follow.

11. Statements:
All plants are trees
All trees are saplings
Conclusions:
I. Some saplings are plants.
II. Some trees are plants
III. All plants are saplings
IV. Some saplings are trees

1. Only I, II and III follows.


2. Only II, III and IV follows.
3. Only I, II and IV follows.
4. Only I, III and IV follows
5. All follow.

12. Statements:
All actors are dancers
All singers are dancers
Conclusions:
I. Some actors are singers
II. All actors are singers
III. All dancers are singers
IV. No actor is a singer
101

1 Only I and III follows


2 Only I follows
3 Only II and III follows
4 Only I or IV follows
5 None follows.

13. Statements:
Some bookies are rookies
All rookies are techies
Conclusions:
I. All rookies are bookies
II. Some techies are rookies
III. Some bookies are not techies
IV. Some techies are bookies

1 Only I and II follow


2 Only II and IV follow
3 Only II and III follow
4 Only III and IV follows
5 Only I, II and III follow

14. Statements:
No Maruti is a Fiat
Some Fiats are Toyotas
Conclusions:
I. Some Marutis are not Toyotas
II. Some Fiats are not Toyotas
III. All Marutis are Toyotas
IV. No fiat is a maruti

1 Only I, II and III follow


2 Only I and II follow
3 Either I or III and IV follow
4 Only II and IV follows
5 All follow

Directions for questions (15 – 25): In each of these questions consists of three or four statements followed by four
conclusions are given. Find which of the conclusion(s) logically follows the given statements, even though they are at
variance with commonly known facts

15. Statements:
Some arguments are arrangements
All arrangements are agreements
Some agreements are achievements

Conclusions:
I. All arguments are agreements
II. Some agreements are arguments
III. Some arguments are achievements
IV. Some arrangements are achievements
102

1 Only I and III follows


2 Only I, II and III follows
3 Only II and IV follows
4 Only II follows
5 None of these

16. Statements:
No Port is a harbour
Some ports are capitals
All harbours are aerodromes
Conclusions:
I. Some aerodromes are not ports
II. No port is an aerodrome
III. No harbor is a capital
IV. Some capitals are not harbours

1 Only I and IV follows


2 Only II and IV follows
3 Only III and IV follows
4 Only IV follows
5 None follow

17. Statements:
Some rubies are not diamonds
All sapphires are emeralds
No emerald is a diamond
Conclusions:
I. Some rubies are not sapphires
II. Some rubies are not emeralds
III. No diamond is a sapphire
IV. Some sapphires are not diamonds

1 Only I and III follows


2 Only IV follows
3 Only III and IV follows
4 Only III follows
5 None of these
18. Statements:
All adjectives are adverbs
Some adverbs are pronouns
All pronouns are nouns
Conclusions:
I. Some adjectives are nouns
II. Some adjectives are pronouns
III. Some adverbs are nouns
IV. Some adjectives are not pronouns

1 Only I and IV follows


2 Only III follows
3 Only I and III follows
4 Only II, III and IV follows
5 All follow
103

19. Statements:
No boxer is a shooter
Some shooters are surfers
All wrestlers are shooters
Conclusions:
I. No boxer is a wrestlers
II. Some boxers are not wrestlers
III. Some surfers are not boxers
IV. Some wrestlers are not boxers

1 Only I and II follows


2 Only II and III follows
3 Only III and IV follows
4 Only I, III and IV follows
5 All follow

20. Statements:
Some jacks are hearts
No heart is a spade
All spades are clubs
Conclusions:
I. Some clubs are not hearts
II. Some clubs are jacks
III. Some spades are jacks
IV. Some hearts are clubs

1 Only II and I follows


2 Only I follows
3 Only III and IV follows
4 Only IV and I follows
5 None of these

21. Statements:
All roads are routes
All routes are networks
All ways are networks
Conclusions:
I. All roads are ways
II. No road is network
III. Some roads are ways
IV. Some ways are roads

1 Only I and II follows


2 Only III and IV follows
3 Only II and IV follows
4 None follows
5 None of these
22. Statements:
Some batsmen are bowlers
All keepers are batsmen
No coach is a keeper
104

Conclusions:
I. No coach is a bowler
II. No keeper is a bowler
III. Some bowlers are coaches
IV. Some batsmen are coaches

1 Only I and II follows


2 Only II and III follows
3 Only I and IV follows
4 None follows
5 None of these

23. Statements:
No clock is a lock
Some docks are not blocks
No rock is a block
All locks are rocks

Conclusions:
I. Some rocks are locks
II. Some locks are not clocks
III. Some rocks are not clocks
IV. Some blocks are docks

1 Only I follows
2 Only II follows
3 Only III follows
4 Only II and IV follow
5 Only I, II and III follow

24. Statements:
All fractions are percentages
All decimals are numbers
No percentage is negative
Some decimals are not negatives
Conclusions:
I. No number is negative
II. Some decimals are fractions
III. No fraction is negative
IV. Some numbers are not negative

1 Only I, II and III follows


2 Only I and IV follows
3 Only III and IV follows
4 Only II and IV follow
5 All follow

25. Statements:
Some ships are boats
No rover is cruise
All rovers are boats
105

Conclusions:
I. Some ships are rovers
II. All boats being cruises is not a possibility
III. No ship is cruise
IV. No ship is rover

1 Only II follows
2 Only II and either I or IV follows
3 Either I or IV follow
4 All follow
5 None follows

Logical Connectives

Directions for the questions (1 – 9)

Each question has a main statement followed by four statements labeled A,B,C,D. Choose the ordered pair of
statements where the first statement implies the second, and the two statements are logically consistent with the
main statement.

1. Either the mouse is of Logitech make or it is of HP make.

(A) The mouse is of Logitech make


(B) The mouse is not of HP make
(C) The mouse is not of Logitech make
(D) The mouse is of HP make.

(1) AB (2)BA (3)DC (4)AD

2. I will buy either a Honda City or a Lancer.

(A) I have bought a Honda City.


(B) I have not bought a Lancer.
(C) I have bought a Lancer.
(D) I have not bought a Honda City.

(1) AB & CD (2) AC Alone (3) DC Alone (4) BA & DC

3. Either the pen is big or the pencil is small.

(A) The pen is big.


(B) The pen is not big.
(C) The pen is small.
(D) The pencil is not small.

(1) AC (2) DB (3) BC (4) AD


106

4. If the kangaroo walks, then the monkey jumps.

(A) The kangaroo did not walk.


(B) The monkey did not jump.
(C) The kangaroo walked.
(D) The monkey jumped.

(1) CA & DB (2) BC & BA (3) BA & CD (4) AB & DC

5. The island is beautiful, if the water is clear.

(A) The island is not beautiful.


(B) The water is clear.
(C) The island is beautiful.
(D) The water is not clear.

(1) CA (2) AD (3) CB (4) DA

6. Unless you catch the thief, the robberies will not stop.

(A) The thief has been caught.


(B) The robberies have stopped.
(C) The thief has not been caught.
(D) The robberies have not stopped.

(1) CD (2) AB (3) DC (4) DA

7. The minister is not re-elected, if the Chief Minister is not re-elected.

(A) The minister is re-elected.


(B) The minister is not re-elected.
(C) The Chief Minister is not re-elected.
(D) The Chief Minister is re-elected.

(1) CA (2) BD (3) BC (4) AD

8. Unless the gangster is arrested, the hostages will be killed.

(A) The hostages will be killed.


(B) The gangster is arrested.
(C) The hostages will not be killed.
(D) The gangster is not arrested.

(1) AD (2) CD (3) CB (4) BA

9. If the Zebra is striped, then the tiger is not striped.

(A) The Zebra is not striped.


(B) The tiger is not striped.
(C) The Zebra is striped.
(D) The tiger is striped.
(1) CB & DA (2) BA & CD (3) Only CD (4) Only BA
107

Directions for the questions (10-17)

Each question has a main statement followed by four numbered statements. From the numbered statements, select
the one that logically follows the main statement.

10. If it is a holiday, then I will go for a picnic or I will visit my uncle’s house.

(1) I will not go for a picnic or I will not visit my uncle’s house, implies that it is not a holiday.
(2) If it is not a holiday, then I will not go for a picnic and I will not visit my uncle’s house.
(3) I will not go for a picnic and I will not visit my uncle’s house, implies that it is not a holiday.
(4) If it is not a holiday, then I will not go for a picnic or I will not visit my uncle’s house.

11. Whenever my mom scolds me, I either hide behind my dad or complain to my grandma.

(1) If I complain to my grandma or I hide behind my dad, then my mommust have scolded me.
(2) If I did not complain to my grandma and I did not hide behind my dad, then my mom must not have
scolded me.
(3) If my mom does not scold me, I will neither hide behind my dad nor complain to my grandma.
(4) None of the above.

12. If the tea is not hot, then I will not go to school and will not have dinner.
(1) If I have gone to school or I have not had dinner, then the tea is not hot.
(2) If I have gone to school and I had dinner, then the tea is hot.
(3) If I have gone to school and I have not had dinner, then the tea is hot.
(4) If I have gone to school or I have had dinner, then the tea is hot.

13. If it is a Sunday, then on that day, there is no college and I go to church.


(1) If there is no college and I do not go to church, then the day is Sunday.
(2) If there is college and I do not go to church, then that day is not a Sunday.
(3) If there is college or I do not go to church, then that day is not a Sunday.
(4) Both 2 & 3.

14. Unless we win the assembly elections, we will lose the Rajya Sahba elections and the Presidential elections
1. We have won the Assembly elections; it means that we will not lose either the Rajya Sahba elections or
the presidential elections.
2. We have not lost the Rajya Sahba elections or we have not lost the Presidential elections, means that we
have won the Assembly elections.
3. We have not lost the Rajya Sabha elections and not lost the Presidential elections, means that we have
won the Assembly elections.
4. Both 2 and 3.
15. If it is not raining, then I will not go for a movie but I will visit my friend’s house.
1. If it is raining, then I will go for a movie but I will not visit my friend’s house.
2. If it is raining, then I will not go for a movie but I will not visit my friend’s house.
3. If I go for a movie or I do not visit my friend’s house, it means that it is raining.
4. If I will not go for a movie and I will visit my friends house, it means that it is raining.
108

16. If it is very hot outside, then I will carry an onion with me and I will return home by lunch time.
1. I will not carry an onion with me or I will not return home by lunch time, means that it is not very hot
outside.
2. It is not very hot outside means that I will not carry an onion with me and I will not return home by
lunch time.
3. I will not carry an onion with me and I will return home by lunch time means that it is not very hot
outside.
4. I will carry an onion with me and I will return home by lunch time means that it is very hot outside.
17. Whenever it rains, I will either carry an umbrella or wear a raincoat
1. It is not raining, means that I will neither carry an umbrella nor wear a raincoat.
2. I am carrying an umbrella or I am wearing a rain coat, implies that it is raining.
3. I am not carrying an umbrella or I am not wearing a raincoat, means that it is not raining.
4. If it is raining but I amnot wearing a raincoat, means that I must be carrying an umbrella.
Directions for the Questions (19 - 23)
Each question below consists a statement followed by four number statements. From the numbered statements
select the one that logically negates the main statement.

19. Sravan will go to the movie, if his parents are not with him.

1. Sravan did not go to the movie and his parents are with him.
2. Sravan’s parents are with him and he went to the movie.
3. Sravan did not go to the movie and his parents are not with him.
4. Sravan went to the movie and his parents are not with him.

20. Ramesh works very hard whenever there is an exam.

1. Ramesh worked very hard and there is no exam


2. Ramesh did not Work hard and there is no exam.
3. Ramesh did not work hard and there is an exam.
4. Both 2 and 3.

21. Either it is a flying saucer or the person is not telling the truth.

1. It is not a flying saucer and the person is not telling the truth.
2. The person is telling the truth and it is not a flying saucer.
3. It is a flying saucer and the person is telling the truth.
4. The person is not telling the truth and it is a flying saucer.

22. Sachin scores a century, unless he is paired with the captain.

1. Sachin is paired with the captain and he did not score a century.
2. Sachin scored a century and he is not paired with the captain.
3. Sachin is paired with the captain and he scored a century.
4. Sachin did not score a century and he is not paired with the captain.
109

23. Bond will buy the car only if it is the costliest and fastest.

1. Bond did not buy the car and it is neither the fastest nor the costliest.
2. Bond bought the car and it is not the costliest or it is not the fastest.
3. The car is the fastest and costliest, and Bond did not buy it.
4. Bond bought the car and it is the fastest and costliest.

Arrangements

Questions (1-3): These questions are based on the following data.

Eight people – Arjun, Bhargav, Chandu, David, Eashwar, Farhaat, Girish and Harish are sitting in a row, and are all
facing the same direction. We have the following information about their seating arrangement.

Arjun is sitting to the left of Farhaat, Who in turn is sitting to the left of David.
Chandu is sitting between Bhargav and harish.
Girish is six places away from Eashwar.
Harish is sitting to the immediate left of Arjun.
Exactly two people are sitting between Bhargav and Arjun, and the number of people sitting to the left of
Arjun is the same as the number of people sitting to the right of Bhargav.

1. Who is sitting at the extreme right of the row?

(a) Girish (b) David (c) Eashwar (d) Cannot be determined

2. If exactly one person is sitting between Farhaat and Grish, then how many persons are sitting
between Chandu and Eashwar?

(a) 3 (b) 2 (c) 1 (d) Cannot be determined

3. If Chandu is sitting to the left of Farhaat, then who is sitting to the immediate left of Harish?

(a) Eashwar (b) Girish (c) David (d) Cannot be determined

Questions (4-8): These questions are based on the following information.

Nine persons A,B,C,D,E,F,G,H and I are seated in a row. The seats are arranged from 1 to 9 from left to right as per
the following restrictions.

C,F and I sit together.


E is to the immediate right of H.
B sits in the fourth seat from the left end and D sits in the third seat from the right end. E is not between B
and D.

4. If G sits to the immediate left of D, then who will sit in the center seat of the row?

(a)B (b)A (c)D (d)F

5. If F sits between C and I, then in how many ways altogether can all of them be seated in the row?

(a)2 (b)1 (c)3 (d)4


110

6. Which of the following is at one of the extreme ends of the row?

(a)C (b)F (c)A (d)E

7. If the positions of B and D are changed such that D takes B’s position and B sits at one of the extreme ends
of the row, then which of the following can be in seats 2,3 and 4, though not in the same order?

(a)C,F and I (b)A,G and D (c)B,D and H (d)G,D and H

8. If I sits at one of the extreme ends and F sits to the immediate left of B, then in how many ways can all be
seated, assuming that the other conditions remain the same?
(a)1 (b)2 (c)3 (d)4

Questions (9-10): These questions are based on the following data.

Eight boys – Jagan, kiran, naveen, prakash, rakesh, suresh, tarun and vinay are sitting around a circular table with
eight chairs. Further, we have the following information about their seating arrangement.
Rakesh is sitting to the immediate right of Vinay.
Tarun is adjacent to both Kiran and Prakash.
Suresh is two places to the left of Vinay.
Suresh is next to Kiran.
Naveen is not next to Prakash.

9. Who is sitting opposite Jagan?

(a)Suresh (b)Naveen c)Kiran (d)Vinay

10. Who is two places to the right of Kiran

(a)Suresh (b)Naveen c)Tarun (d)Rakesh

Questions (11-15): These questions are based on the following data.

Eight persons – A through H are sitting around a circular table. It is know that

F is adjacent to neither G nor B


A is two places away to the left of B. G and B are not adjacent to each other.
D is to the right of F
H is three places away to the right f G.

11. Who is sitting to the left of C?

(a)F (b)B (c)A (d)G (e)Cannot be determined

12. Who is sitting opposite G?

(a)A (b)B (c)C (d)D (e)E


111

13. Who is sitting two places away to the left of D?

(a)H (b)A (c)C (d)D (e)Cannot be determined

14. Who is to the right of A?

(a)C (b)B (c)H (d)E (e)Cannot be determined

15. Who is sitting opposite F?

(a)H (b)A (c)C (d)E (e)None of these

Questions (16-20): These questions are based on the following data.

P, K, R, S, T and U sit around a hexagonal table in such a way that neither P nor R sits opposite to K and neither T nor
U sits opposite to S, whereas R sits to the immediate left of S.

16. If K sits to the immediate right of S, then who sits opposite to S?

(a)T (b)P (c)U (d)None of these

17. If T sits opposite to R, then which of the following must be true?

(a)P is opposite S (b)K and U sit opposite (c)P and U sit opposite (d)None

18. If T sits opposite to K, then which of the following is definitely false?

(a)U and P sit next to each other (b)U sits opposite to R


(c)P sits opposite to S (d)P and U have only one person between them

19. If P sits to the immediate left of R, then

(a)K sits opposite to R (b)P sits opposite to S (c)K sits opposite to S (d)T is opposite to R

20. Which of the following additional pieces of information will determine the seating arrangement completely?

(a)T is opposite to S (b)P is opposite to S (c)T and U are next to each other (d)None

Questions (21-25): These questions are based on the following paragraph

Four men A, B, C, D and four women P, Q, R, S are dancing on a floor, in such a way that they face the center and
form a circle. No two women and no two men are next to each other. A is to the immediate left of R, who is opposite
to Q. P and Q have only C dancing between them. P is dancing opposite to S, who is dancing to the immediate right
of B.

21. If B is the only person dancing between R and S, then who is opposite to him?

(a) A (b) Q (c) C (d) D


112

22. Which of the following is an acceptable arrangement of the dancers in clockwise direction?
(a) C,Q,P,D,S,B,A,R (b) Q,C,P,D,R,A,S,B (c) A,R,D,P,C,Q,B,S (d) D,R,A,S,B,Q,P,C

23. Which of the following persons are opposite each other?


(a) Q and R (b) B and Q (c) C and D (d) A and B

24. Who is the only person dancing between P and R?

(a) B or C (b) S or A (c) A or D (d) D or C

25. If S is to the immediate left of A, then who is opposite C?


(a) A (b) B (c) D (d) M

Questions (26 - 30): These questions are based on the following data.

Eight chairs are arranged in a room. Four of them are exactly at the four corners while the remaining four are placed
against the walls on the four sides in between each pair of chairs in the corners. P,Q,R and S are four boys and
A,B,C,D are four girls who occupy the chairs all of which are facing the center of the room. Q is in a corner chair and
R is in a chair which is not along the same wall as either of the walls adjoining Q. A and C are seated at corners,
diagonally opposite to each other. B does not sit along any wall which is adjacent to the corner where A sits and is
opposite to P. C sits to the immediate right of R, who is between C and D.

26. S must be seated between


(a)C and Q (b)A and D (c)A and Q (d)C and A

27. If S and P interchange their seats, then who is to the immediate left of D?
(a)A (b)P (c)R or C (d)S

28. Which of the following is not one of the correct arrangements of the corner seat occupants, either in clock-
wise or anti clock-wise direction consecutively?
(a)Q,A,D and C (b)A,Q,C, and D (c)D,A,Q and C (d)D,Q,A and C

29. Who sits to the immediate left of A?


(a)Q (b)R (c)D (d)S

30. P sits between


(a)A and D (b)D and C (c)C and Q (d)Cannot be determined

Questions (31-35): These questions are based on the following data.

Seven men A, B, C, D, E, F and G have parked their cars in a row. The cars of E and F should be next to each other.
The cars of D and G should be parked next to each other. Whereas A and B cannot park their cars next to each other.
But B and D must park their cars next to each other and C’s car is parked to the immediate right of G’s car.

31. If A’s car is not next to C’s car, then what is the position of A’s car from the right end of the row?
(a)1st (b)7th (c)5th (d)1st or 7th

32. If F does not park his car next to A’s, which is parked in the left extreme, then which of the following men can
park their cars between the cars of E and G?
(a)G, C and B (b)A, F and C (c)F, G and C (d)F, B and D
113

33. In how many ways can the seven cars be parked in a row?
(a)2 (b)8 (c)7 (d)5

34. If A parks his car to the immediate right of C’s, then which of the following cars can be parked the left extreme
end?
(a)Only E (b)Only F (c)Only E or F (d)E or F or B

35. If E parks his car to the left of F, then which of the following statements must be false?
(a)There are two cars between B’s and G’s (b)B and C’s cars are not parked together
(c)G’s car is the only car in between D’s and C’s (d)A’s car is at the left extreme end

Ordering, Selections and Double Line-up

Questions (1-3): These questions are based on the following information.

Six boys – Vicky, Nicky, Kiran, Ricky, Danny and Ravi – are comparing their heights. We have the following
information about their heights.

Only one boy is taller than Vicky


Nicky is taller than only one boy.
The number of boys shorter than Kiran is the same as the number of boys taller than Ricky
No two boys are of the same height.
Danny is shorter than Ravi, who in turn is shorter than Ricky.

1. How many boys are taller than Kiran?


(a)5 (b)3 (c)2 (d)None of the above

2. Who is the fourth shortest boy?


(a)Ravi (b)Danny (c)Kiran (d)Ricky

3. The Boy, who is taller than Nicky but shorter than Ravi, is
(a)Kiran (b)Danny (c)Ricky(d)Cannot be determined

Questions (4-5): These questions are based on the following information.

In a class the top six rankers – Sudha, Chandran, Hema, Bindu, Ramya and Tara – are comparing their ranks. We
have the following information about their ranks.

Sudha’s rank is better than Chandran’s but worse than Hema’s


Ramya’s Rank is worse than Bindi’s, whose rank is better than Tara’s
Hema is not the first ranker.
Ramya’s rank is worse than Sudha’s and Tara’s rank is better than Chandran’s

4. If Ramya has got the fourth rank, then what is Tara rank?
(a)5 (b)3 (c)2 (d)cannot be determined

5. If Tara has got the second rank, then who has got the fifth rank?
(a)Chandran (b)Ramya (c)Sudha (d)Cannot be determined

Questions (9-10): These questions are based on the following data.


114

A team of three people is to be selected from six people Dinesh, Rajan, Karan, Anish, Varun and Charan – Subjected
to the following conditions.

If Dinesh is selected , then Rajan must not be selected.


At most one of karan and Anish can be selected.
At least one of Varun and Charan must be selected.

6. In How many ways can the teambe selected?


(a)6 (b)8 (c)10 (d)12

7. If Anish and Charan are selected, then who cannot be in the team?
(a)Dinesh (b)Rajan (c)Varun (d)None of the above

Questions (8-10): These questions are based on the following information.

A team of four people is to be selected girls – Chandana, Jaya, Kamala, Malini, Sanjana, Tanuja and Vandana –
Subjected to the following conditions.

Unless Kamala is selected, Malini must be selected.


All the three of chandana , Tanuja and vandana cannot be selected together.
At least one of Jaya and Sanjana must be selected.
At most one of Chandana and Malini can be selected.

8. If chandana is selected, then who must be selected for the team?


(a)Kamala (b)Jaya (c)Sanjana (d)None of the above

9. If exactly one of Chandana and Jaya is selected, then which of the following can be the other three members
of that group?
(a)sanjana, Tanuja and Vandana (b)Malini, Tanuja and Vandana
(c)Kamala, Sanjana and Tanuja (d)Any of the above

10. If Sanjana is not to be selected, then in how many ways can the required team be selected?
(a)2 (b)4 (c)6 (d)None of the above

Questions (11-15): These questions are based on the following information.

Each of four Friends Abhinay, Bhanu, Chandra, and Dhanush are living in one of the four different floors 1St to 4th
(bottom to top) of a building. Each of thembelongs to a different state among Andhra Pradesh (A.P), Uttar Pradesh
(U.P), Himachal Pradesh (H.P), and Madhya Pradesh (M.P).
The information known is as follows:
The person belongs to Andhra Pradesh stays in the 1st floor.
Chandra stays in 4th floor.
Bhanu belongs to Himachal Pradesh.
The person who stays at the 2nd floor does not belong to Madhya Pradesh.
Neither Abhinay nor Bhanu stays at the 2nd floor.

11. Who stays in the 3rd floor?


(a) Abhinay (b) Bhanu (c) Dhanush (d)Cannot be determined

12. Who belongs to Madhya Pradesh?

(a) Bhanu (b) Abhnay (c)Chandra (d)Cannot be determined


115

13. Which of the following is the correct combination of the person, the floor in which he lives and the city to
which he belongs?
(a) 3rd – Dhanush – Madhya Pradesh (b) 2nd – Dhanush – Himachal Pradesh
(c)1st – Abhinay – Uttar Pradesh (d) 3rd – Bhanu – Himachal Pradesh

14. From which state does the person who stays at the 4th floor belong to?
(a)Uttar Pradesh (b) Madhya Pradesh (c) Himachal Pradesh (d) Andhra Pradesh

15. At which floor does the person who belongs to Uttar Pradesh stay?
(a) 2nd floor (b) 3rd floor (c) 4th floor (d) 1st floor

Questions (16-20): These questions are based on the following information.

In a school, five students A, B, C, D and E secured top five ranks in each of the four different categories among –
Attendance, Academics, Discipline, and Cultural Activities. No two of them got the same rank in any category and
none of them got the same rank in any two categories. Further it is known that
In each category, D got a better rank than A.
C did not get the first rank in Discipline.
B and E got the fourth and the fifth ranks in Discipline respectively, C got the fourth rank in Attendance.
A got a better rank than C in Academics.

16. Who got the third rank in cultural activities?


(a)A (b)C (c)E (d)D

17. In Which category did B get the top rank?


(a)Attendance (b)Academics (c)Discipline (d)Cannot be determined

18. What is the sum of the ranks of D in Attendance and that of A in Discipline?
(a)6 (b)5 (c)4 (d)7

19. Who among the following did not get the 4th rank in any of the categories?
(a)B (b)A (c)D (d)E

20. What is the sum of the ranks of C in all the categories put together?
(a)14 (b)12 (c)13 (d)11

Source of Content and References


https://www.englishgrammar.org
http://www.world-english.org
http://www.indiabix.com
http://www.majortests.com
http://www.mindtools.com
http://www.readingcompresions.com

S-ar putea să vă placă și